300+ Basic Surgical Background Objective Questions and Answers

Basic Surgical Background Multiple Choice Questions :-

1. Skeletal muscle breakdown produces predominantly liberation of which two amino acids?
A. Lysine.
B. Tyrosine.
C. Alanine.
D. Glutamine.
E. Arginine.
Answer: CD

2. In “catabolic” surgical patients, which of the following changes in body composition do not occur?
A. Lean body mass increases.
B. Total body water increases.
C. Adipose tissue decreases.
D. Body weight decreases.
Answer: A

3. The hormonal alterations that follow operation and injury favor accelerated gluconeogenesis. This new glucose is consumed by which of the following tissues?
A. Central nervous system.
B. Skeletal muscle.
C. Bone.
D. Kidney.
E. Tissue in the healing wound.
Answer: ADE

4. Cytokines are endogenous signals that stimulate:
A. Local cell proliferation within the wound.
B. The central nervous system to initiate fever.
C. The production of “acute-phase proteins.”
D. Hypoferremia.
E. Septic shock.
Answer:ABCD

5. The characteristic changes that follow a major operation or moderate to severe injury do not include the following:
A. Hypermetabolism.
B. Fever.
C. Tachypnea.
D. Hyperphagia.
E. Negative nitrogen balance.
Answer: D

6. Shock can best be defined as:
A. Hypotension.
B. Hypoperfusion of tissues.
C. Hypoxemia.
D. All of the above.
Answer: B

7. Which of the following statements about continuous cardiac output monitoring are true?
A. Continuous cardiac output monitoring may unmask events not detected by intermittent cardiac output measurements.
B. Continuous cardiac output monitoring by the thermodilution method requires continuous infusion of fluid injectate at a constant rate and temperature.
C. The major advantage of the Fick method over the thermodilution method of calculating cardiac output is that it is noninvasive, requiring only the determination of oxygen consumption by respiratory gas analysis.
D. The technique of thoracic electrical bioimpedance utilizes sensors to determine stroke volume by detecting changes in resistance to a small, applied alternating current.
Answer: AD

8. Which of the following statements regarding cytokines is incorrect?
A. Cytokines act directly on target cells and may potentiate the actions of one another.
B. Interleukin 1 (IL-1) is a major proinflammatory mediator with multiple effects, including regulation of skeletal muscle proteolysis in patients with sepsis or significant injury.
C. Platelet-activating factor (PAF) is a major cytokine that results in platelet aggregation, bronchoconstriction, and increased vascular permeability.
D. Tumor necrosis factor alpha (TNF-a), despite its short plasma half-life, appears to be a principal mediator in the evolution of sepsis and the multiple organ dysfunction syndrome because of its multiple actions and the secondary cascades that it stimulates.
Answer: C

9. True statements concerning hypoadrenal shock include which of the following?
A. Adrenocortical insufficiency may manifest itself as severe shock refractory to volume and pressor therapy.
B. The presence of hyperglycemia and hypotension may suggest the diagnosis of shock due to adrenocortical insufficiency.
C. Hydrocortisone does not interfere with the serum cortisol assay and should be given to hemodynamically unstable patients suspected of having hypoadrenal shock.
D. The rapid adrenocorticotropic hormone (ACTH) stimulation test should be performed to help establish the diagnosis of acute adrenocortical insufficiency.
Answer: AD

10. All of the following are true about neurogenic shock except:
A. There is a decrease in systemic vascular resistance and an increase in venous capacitance.
B. Tachycardia or bradycardia may be observed, along with hypotension.
C. The use of an alpha agonist such as phenylephrine is the mainstay of treatment.
D. Severe head injury, spinal cord injury, and high spinal anesthesia may all cause neurogenic shock.
Answer: C

BASIC SURGICAL BACKGROUND Objective Questions
BASIC SURGICAL BACKGROUND MCQs

11. True statements regarding eicosanoids include which of the following?
A. Prostaglandins and thromboxanes are synthesized via the cyclo-oxygenase pathway.
B. The vasoconstricting, platelet-aggregating, and bronchoconstricting effects of thromboxane A 2 are balanced by the actions of prostacyclin, which produces the opposite effects.
D. The principal prostaglandins have a short circulation half-life and exert most of their effects locally.
Answer: ABD

12. Which of the following statements about delivery-dependent oxygen consumption are true?
A. Below the critical oxygen delivery (D(overdot)O 2crit), one would expect to see a decrease in the lactate-pyruvate ratio.
B. D(overdot)O 2crit may be increased in patients with sepsis.
C. A desirable goal in the treatment of shock is to achieve delivery-independent oxygen consumption.
D. The oxygen extraction ratio remains constant as long as oxygen delivery remains above D(overdot)O 2crit.
Answer: BC

13. All of the following may be useful in the treatment of cardiogenic shock except:
A. Dobutamine.
B. Sodium nitroprusside.
C. Pneumatic antishock garment.
D. Intra-aortic balloon pump.
Answer: C

14. Which of the following statements concerning monitoring techniques in the intensive care unit are true?
A. Pulmonary artery and pulmonary capillary wedge pressure readings should be made at end inspiration, to minimize ventilatory artifacts.
B. Continuous SvO 2 monitoring based on the technique of reflectance spectrophotometry has been shown to be accurate and reliable.
C. Direct measurement of gastric intramucosal pH can be provided by gastrointestinal tonometry.
D. Hyperlactatemia may be seen in a number of clinical conditions not associated with tissue hypoxia, including liver disease and hypermetabolic states.
Answer: BD

15. An 18-year-old man shot once in the left chest has a blood pressure of 80/50 mm. Hg, a heart rate of 130 beats per minute, and distended neck veins. Immediate treatment might include:
A. Administration of one liter of Ringer’s lactate solution.
B. Subxiphoid pericardiotomy.
C. Needle decompression of the left chest in the second intercostal space.
D. Emergency thoracotomy to cross-clamp the aorta.
Answer: AC

16. Which of the following statements are true of the multiple organ dysfunction syndrome (MODS)?
A. The “two-hit” model proposes that secondary MODS may be produced when even a relatively minor second insult reactivates, in a more amplified form, the systemic inflammatory response that was primed by an initial insult to the host.
B. The systemic inflammatory response syndrome (SIRS), shock due to sepsis or SIRS, and MODS may be regarded as a continuum of illness severity.
C. Prolonged stimulation or activation of Kupffer cells in the liver is thought to be a critical factor in the sustained, uncontrolled release of inflammatory mediators.
D. The incidence of MODS in intensive care units has decreased owing to increased awareness, prevention, and treatment of the syndrome.
Answer: ABC

17. All of the following statements about hemorrhagic shock are true except:
A. Following hemorrhagic shock, there is an initial interstitial fluid volume contraction.
B. Dopamine, or a similar inotropic agent, should be given immediately for resuscitation from hemorrhagic shock, to increase cardiac output and improve oxygen delivery to hypoperfused tissues.
C. The use of colloid solutions or hypertonic saline solutions is contraindicated for treatment of hemorrhagic shock.
D. In hemorrhagic shock, a narrowed pulse pressure is commonly seen before a fall in systolic blood pressure.
Answer: BC

18. Which of the following statements about septic shock are true?
A. A circulating myocardial depressant factor may account for the cardiac dysfunction sometimes seen with shock due to sepsis or SIRS.
B. A cardiac index (CI) of 6 liters per minute per square meter of body surface, a pulmonary capillary wedge pressure of 15 mm. Hg, and a systemic vascular resistance index (SVRI) of 800 dynes-sec/(cm 5-m 2) is a hemodynamic profile consistent with septic shock.
C. An increase in SvO 2 in septic patients may be explained by the finding of anatomic arteriovenous shunts.
D. Results of human trials employing antimediator therapy, such as antiendotoxin antibodies, IL-1 receptor antagonist, and tumor necrosis factor (TNF) antibodies, have confirmed animal studies that demonstrate a significant improvement in survival with the use of such agents.
Answer: AB

19. Which of the following statements are true of oxidants?
A. In addition to their pathophysiologic roles in inflammation, injury, and infection, oxidants also have physiologic roles.
B. Oxidants may be generated from activated neutrophils and during reperfusion following a period of ischemia.
C. The deleterious effects of oxidants include lipid peroxidation and cell membrane damage, oxidative damage to DNA, and inhibition of adenosine triphosphate (ATP) synthesis.
D. The mechanism of ischemia-reperfusion injury involved the catalytic production of superoxide anion (O 2•) by the enzyme xanthine oxidase.
Answer: ABCD

20. Which of the following statements about the role of the gut in shock and sepsis are true?
A. Selective decontamination of the digestive tract with the use of oral antibiotics has been shown to reduce nosocomial pneumonias and to improve mortality rates.
B. Enteral nutrition, as compared with parenteral nutrition, preserves the villus architecture of the gut.
C. Gut dysfunction may be an effect of shock, but it may also contribute to the development of MODS by the mechanism of bacterial translocation.
D. As compared with parenteral nutrition, enteral nutrition is associated with a reduction in septic morbidity.
Answer: BCD

21. Which of the following statements about head injury and concomitant hyponatremia are true?
A. There are no primary alterations in cardiovascular signs.
B. Signs of increased intracranial pressure may be masked by the hyponatremia.
C. Oliguric renal failure is an unlikely complication.
D. Rapid correction of the hyponatremia may prevent central pontine injury.
E. This patient is best treated by restriction of water intake.
Answer: A

22. Which of the following statements about total body water composition are correct?
A. Females and obese persons have an increased percentage of body water.
B. Increased muscle mass is associated with decreased total body water.
C. Newborn infants have the greatest proportion of total body water.
D. Total body water decreases steadily with age.
E. Any person’s percentage of body water is subject to wide physiologic variation.
Answer: CD

23. Which of the following statements about extracellular fluid are true?
A. The total extracellular fluid volume represents 40% of the body weight.
B. The plasma volume constitutes one fourth of the total extracellular fluid volume.
C. Potassium is the principal cation in extracellular fluid.
D. The protein content of the plasma produces a lower concentration of cations than in the interstitial fluid.
E. The interstitial fluid equilibrates slowly with the other body compartments.
Answer: B

24. Which of the following statements are true of a patient with hyperglycemia and hyponatremia?
A. The sodium concentration must be corrected by 5 mEq. per 100 mg. per 100 ml. elevation in blood glucose.
B. With normal renal function, this patient is likely to be volume overloaded.
C. Proper fluid therapy would be unlikely to include potassium administration.
D. Insulin administration will increase the potassium content of cells.
E. Early in treatment adequate urine output is a reliable measure of adequate volume resuscitation.
Answer: D

25. Which of the following statements about respiratory acidosis are true?
A. Compensation occurs by a shift of chloride out of the red blood cells.
B. Renal compensation occurs rapidly.
C. Retention of bicarbonate and increased ammonia formation are normal compensatory mechanisms.
D. Narcotic administration is an unusual cause of respiratory acidosis.
E. The ratio of bicarbonate to carbonic acid is less than 20:1.
Answer: CE

26. Which of the following statements are true of elevated–anion gap metabolic acidosis?
A. Hypoperfusion from the shock state rarely produces an elevated anion gap.
B. Retention of sulfuric and phosphoric acids may lead to this form of acidosis.
C. Copious diarrhea does not produce this condition.
D. Rapid volume expansion may produce this form of acidosis.
E. Use of lactated Ringer’s solution is inappropriate in the treatment of lactic acidosis.
Answer: BC

27. Which of the following is true of loss of gastrointestinal secretions?
A. Gastric losses are best replaced with a balanced salt solution.
B. Potassium supplementation is unnecessary in replacement of gastric secretions.
C. Bicarbonate wasting is an unusual complication of a high-volume pancreatic fistula.
D. Balanced salt solution is a reasonable replacement fluid for a small bowel fistula.
E. A patient with persistent vomiting usually requires hyperchloremic replacement fluids.
Answer: DE

28. Which of the following statements regarding hypercalcemia are true?
A. The symptoms of hypercalcemia may mimic some symptoms of hyperglycemia.
B. Metastatic breast cancer is an unusual cause of hypercalcemia.
C. Calcitonin is a satisfactory long-term therapy for hypercalcemia.
D. Severely hypercalcemic patients exhibit the signs of extracellular fluid volume deficit.
E. Urinary calcium excretion may be increased by vigorous volume repletion.
Answer: ADE

29. Which of the following statements about normal salt and water balance are true?
A. The products of catabolism may be excreted by as little as 300 ml. of urine per day.
B. The lungs represent the primary source of insensible water loss.
C. The normal daily insensible water loss is 600 to 900 ml.
D. Excessive cell catabolism causes significant loss of total body water.
E. In normal humans, urine represents the greatest source of daily water loss.
Answer: CE

30. Which of the following is/are not associated with increased likelihood of infection after major elective surgery?
A. Age over 70 years.
B. Chronic malnutrition.
C. Controlled diabetes mellitus.
D. Long-term steroid use.
E. Infection at a remote body site.
Answer: C

31. Which of the following are not determinants of a postoperative cardiac complication?
A. Myocardial infarct 4 months previously.
B. Clinical evidence of congestive heart failure in a patient with 8.5 gm. per dl. hemoglobin.
C. Premature atrial or ventricular contractions on electrocardiogram.
D. A harsh aortic systolic murmur.
E. Age over 70 years.
Answer: B

32. Rank the clinical scenarios in order of greatest likelihood of serious postoperative pulmonary complications.
A. Transabdominal hysterectomy in an obese woman that requires 3 hours of anesthesia time.
B. Right middle lobectomy for bronchogenic cancer in a 65-year-old smoker.
C. Vagotomy and pyloroplasty for chronic duodenal ulcer disease in a 50-year-old who had chest film findings of old, healed tuberculosis.
D. Right hemicolectomy in an obese 60-year-old smoker.
E. Modified radical mastectomy in a 58-year-old woman who is obese.
Answer: BDCAE

33. Rank the following laboratory tests and procedures in terms of their relative value to a 65-year-old woman who is to undergo elective resection of a sigmoid cancer.
A. Carcinoembryonic antigen (CEA).
B. Blood urea nitrogen (BUN).
C. Electrocardiogram (ECG).
D. Hemoglobin concentration (Hgb).
E. Serum creatinine (Cr).
F. Arterial blood oxygen tension (PaO 2) on room air.
G. Serum sodium concentration (Na+).
Answer: CDFEBAG

34. Which of the following statements regarding whole blood transfusion is/are correct?
A. Whole blood is the most commonly used red cell preparation for transfusion in the United States.
B. Whole blood is effective in the replacement of acute blood loss.
C. Most blood banks in the United States have large supplies of whole blood available.
D. The use of whole blood produces higher rates of disease transmission than the use of individual component therapies.
Answer: B

35. Which of the following statements about the preparation and storage of blood components is/are true?
A. Solutions containing citrate prevent coagulation by binding calcium.
B. The shelf life of packed red blood cells preserved with CPDA-1 is approximately 35 days at 1؛ to 6؛ C.
C. There are normal numbers of platelets in packed red blood cells stored at 1؛ to 6؛ C for more than 2 days.
D. The storage lesion affecting refrigerated packed red blood cells includes development of acidosis, hyperkalemia, and decreased intracellular 2,3DPG (diphosphoglycerate).
Answer: ABD

36. Which of the following is/are acceptable reasons for the transfusion of red blood cells based on currently available data?
A. Rapid, acute blood loss with unstable vital signs but no available hematocrit or hemoglobin determination.
B. Symptomatic anemia: orthostatic hypotension, severe tachycardia (greater than 120 beats per minute), evidence of myocardial ischemia, including angina.
C. To increase wound healing.
D. A hematocrit of 26% in an otherwise stable, asymptomatic patient.
Answer: AB

37. The transfusion of fresh frozen plasma (FFP) is indicated for which of the following reasons?
A. Volume replacement.
B. As a nutritional supplement.
C. Specific coagulation factor deficiency with an abnormal prothrombin time (PT) and/or an abnormal activated partial thromboplastin time (APTT).
D. For the correction of abnormal PT secondary to warfarin therapy, vitamin K deficiency, or liver disease.
Answer: CD

38. In patients receiving massive blood transfusion for acute blood loss, which of the following is/are correct?
A. Packed red blood cells and crystalloid solution should be infused to restore oxygen-carrying capacity and intravascular volume.
B. Two units of FFP should be given with every 5 units of packed red blood cells in most cases.
C. A “six pack” of platelets should be administered with every 10 units of packed red blood cells in most cases.
D. One to two ampules of sodium bicarbonate should be administered with every 5 units of packed red blood cells to avoid acidosis.
E. One ampule of calcium chloride should be administered with every 5 units of packed red blood cells to avoid hypocalcemia.
Answer: A

39. Hemostasis and the cessation of bleeding require which of the following processes?
A. Adherence of platelets to exposed subendothelial glycoproteins and collagen with subsequent aggregation of platelets and formation of a hemostatic plug.
B. Interaction of tissue factor with factor VII circulating in the plasma.
C. The production of thrombin via the coagulation cascade with conversion of fibrinogen to fibrin.
D. Cross-linking of fibrin by factor XIII.
Answer: ABCD

40. Which of the statements listed below about bleeding disorders is/are correct?
A. Acquired bleeding disorders are more common than congenital defects.
B. Deficiencies of vitamin K decrease production of factors II, VII, IX, and X, protein C, and protein S.
C. Hypothermia below 32؛C rarely causes a bleeding disorder.
D. Von Willebrand’s disease is a very uncommon congenital bleeding disorder.
Answer: AB

41. The evaluation of a patient scheduled for elective surgery should always include the following as tests of hemostasis and coagulation:
A. History and physical examination.
B. Complete blood count (CBC), including platelet count.
C. Prothrombin time (PT) and activated partial thromboplastin time (APTT).
D. Studies of platelet aggregation with adenosine diphosphate (ADP) and epinephrine.
Answer: A

42. Which of the following statements regarding the transmission of infectious agents through blood transfusions is/are true?
A. The transmission rates for human immunodeficiency virus (HIV) have been decreasing progressively since the early 1980s.
B. The transmission rates of hepatitis have been decreasing steadily since the 1980s.
C. Cytomegalovirus (CMV) is the infectious agent most commonly transmitted in blood.
D. Severely immunocompromised patients (such as patients undergoing transplantation) should receive specially screened blood products.
Answer: ABCD

43. The most common cause of fatal transfusion reactions is:
A. An allergic reaction.
B. An anaphylactoid reaction.
C. A clerical error.
D. An acute bacterial infection transmitted in blood.
Answer: C

44. Which of the following statements about the coagulation cascade is/are true?
A. The intrinsic pathway of coagulation is the predominant pathway in vivo for hemostasis and coagulation.
B. The intrinsic pathway beginning with the activation of factor XII is the predominant in vivo mechanism for activation of the coagulation cascade.
C. Deficiencies of factor VIII and IX cause highly significant coagulation abnormalities.
D. Deficiencies of factor XII cause severe clinical bleeding syndromes.
Answer: AC

45. A major problem in nutritional support is identifying patients at risk. Recent studies suggest that these patients can be identified. Which of the following findings identify the patient at risk?
A. Weight loss of greater than 10% over 2 to 4 months.
B. Serum albumin of less than 3 gm. per 100 ml. in the hydrated state.
C. Malnutrition as identified by global assessment.
D. Serum transferrin of less than 220 mg. per 100 ml.
E. Functional impairment by history.
Answer: ABCDE

46. Essential fatty acid deficiency may complicate total parenteral nutrition (TPN). Which of the following statements are true?
A. Essential fatty acid deficiency may be prevented by the administration of 1% to 2% of total calories as fat emulsion.
B. Fat-free parenteral nutrition results in the appearance of plasma abnormalities, indicating essential fatty acid deficiency, within 7 to 10 days of initiation.
C. An abnormal plasma eicosatrienoic-arachidonic acid ratio is always associated with essential fatty acid deficiency.
D. Following initiation of fat-free parenteral nutrition, dry, scaly skin associated with a maculopapular rash indicates essential fatty acid deficiency.
Answer: BD

47. It is stated that enteral nutrition is safer than parenteral nutrition. Which of the following may be complications of enteral nutrition?
A. Hyperosmolar, nonketotic coma.
B. Vomiting and aspiration.
C. Pneumatosis cystoides intestinalis.
D. Perforation and peritonitis.
Answer: ABCD

48. It has been suggested that enterocyte-specific fuels be utilized for all patients receiving parenteral nutrition. Theoretically, the benefits of such fuels include:
A. Glutamine increases gut mucosal protein content and wall thickness.
B. Butyrate increases jejunal mucosal protein content and wall thickness.
C. The short-chain fatty acids—butyrate, propionate, and acetate—are useful in supporting ileal mucosal protein content and thickness.
D. The use of glutamine-enriched solutions for parenteral nutrition for patients with chemotherapy toxicity or radiation enteritis is without hazards.
Answer: NONE IS ENTIRELY TRUE

49. Essential amino acids have been advocated as standard therapy for renal failure. Which of the following statements are true?
A. Increased survival from acute renal failure has been reported with both essential and nonessential amino acid therapy of patients in renal failure.
B. Essential amino acids retard the rise of blood urea nitrogen (BUN) secondary to decreased urea appearance.
C. Essential amino acids and hypertonic dextrose are a convenient form of therapy for hyperkalemia.
D. Essential amino acids decrease BUN and creatinine to the same degree as solutions containing excessive nonessential amino acids.
Answer: BC

50. A modified amino acid solution with increased equimolar branched-chain amino acids and decreased aromatic amino acids has been proposed for patients with hepatic insufficiency. Which of the following statements is/are true?
A. This formulation is proposed for the use of patients with fulminant hepatitis.
B. Nitrogen balance is achieved in such patients with amounts of 40 gm. of amino acids per 24 hours.
C. The use of 80 to 100 gm. of such solutions is associated with hepatic encephalopathy.
D. In some studies of surgical patients, improvements in mortality have been reported.
Answer: D

51. In the nutritional support of patients with cancer, which of the following statements is/are true?
A. Nutritional support benefits the patient’s lean body mass but does not enable the tumor to grow.
B. In experimental animals, the growth of implanted tumors is directly proportional to the amount of calories and protein supplied.
C. Prospective randomized trials of nutritional support utilizing chemotherapy and radiation therapy have revealed benefits to patients receiving total parenteral nutrition.
D. Studies of nutritional support for patients with cancer about to undergo surgery revealed decreased morbidity and mortality, especially morbidity from sepsis.
Answer: B

52. Glucose overload results in increased CO 2 production. Which of the following statements are true?
A. In patients with respiratory insufficiency, administration of glucose as a principal calorie source is contraindicated.
B. In patients with pulmonary infection and sepsis, calorie support should consist of 95% fat and 5% glucose.
C. In Askanazi’s study, increased CO 2 production and difficulty in weaning was associated only with pronounced overfeeding.
D. CO 2 production should be measured in most patients who are supported by respirators in intensive care units and are receiving nutritional support.
Answer: C

53. Hepatic abnormalities have been noted in adults since the beginning of hyperalimentation. Which of the following statements are true?
A. Hepatic steatosis appears to be associated with an overload of glucose.
B. Hepatic steatosis is usually associated with abnormalities in hepatic enzymes.
C. Hyperbilirubinemia is inevitably associated with hepatic steatosis.
D. Abnormalities in the portal insulin-glucagon ratio are thought to be causative of hepatic steatosis in experimental animals.
Answer: AD

54. Which of the following statements about the presence of gallstones in diabetes patients is/are correct?
A. Gallstones occur with the same frequency in diabetes patients as in the healthy population.
B. The presence of gallstones, regardless of the presence of symptoms, is an indication for cholecystectomy in a diabetes patient.
C. Diabetes patients with gallstones and chronic biliary pain should be managed nonoperatively with chemical dissolution and/or lithotripsy because of severe complicating medical conditions and a high operative risk.
D. The presence of diabetes and gallstones places the patient at high risk for pancreatic cancer.
E. Diabetes patients with symptomatic gallstones should have prompt elective cholecystectomy, to avoid the complications of acute cholecystitis and gallbladder necrosis.
Answer: E

55. Intensive insulin therapy:
A. Prevents the aggressive development of atherosclerosis in diabetic patients.
B. Is not associated with unawareness of hypoglycemia.
C. Improves peripheral neuropathy.
D. Improves established retinopathy and nephropathy.
E. Is indicated in all patients with non–insulin-dependent diabetes mellitus (NIDDM).
Answer: C

56. Which of the following statements about hypertension in diabetes patients is/are correct?
A. Hypertension worsens the macrovascular disease of diabetes patients.
B. Hypertension accelerates the progression of diabetic nephropathy.
C. Hypertension is associated with sodium retention in diabetes patients.
D. Angiotensin-converting enzyme (ACE) inhibitors should be used in all patients with chronic hyperglycemia, regardless of the presence of hypertension.
E. Diuretics, are not indicated in the treatment of hypertension in diabetes patients.
Answer: ABCDE

57. What is the major determinant in an individual patient’s risk for perioperative complications?
A. The surgical procedure.
B. The length of the surgical procedure.
C. The anesthetic technique (e.g., general, regional).
D. The length of anesthesia.
E. All of the above.
Answer: A

58. Which of the following are considered routine intraoperative monitors?
A. Temperature probe.
B. Electrocardiogram.
C. Capnograph.
D. Blood pressure cuff.
E. Foley catheter.
Answer: ABD

59. Muscle relaxants can be used for which of the following?
A. To facilitate intubation.
B. To provide optimal surgical conditions.
C. To optimize ventilator support.
D. To provide sedation.
Answer: ABC

60. Local anesthetics:
A. Inhibit transmission of nerve impulses by increasing sodium membrane permeability and the displacement of ionized calcium.
B. Are classified as amides or esters.
C. Produce peripheral vasodilation.
D. Are weak acids.
Answer: BC

61. Absolute indications for a double-lumen endotracheal tube during thoracic surgery are:
A. Massive hemorrhage from one lung.
B. Unilateral lung infection.
C. Facilitation of surgical exposure.
D. Unilateral bronchopulmonary lavage.
E. All of the above.
Answer: ABD

62. Determinants of cerebral blood flow include:
A. Preoperative neurologic dysfunction.
B. Arterial CO 2 tension.
C. Arterial O 2 tension.
D. Systemic arterial pressure.
E. All of the above.
Answer: BCD

63. Discharge criteria following ambulatory surgery include:
A. Ability to eat solid food.
B. Stable vital signs.
C. Ability to ambulate.
D. Ability to have protective airway reflexes.
Answer: BCD

64. Advantages of patient-controlled analgesia (PCA) include:
A. Immediate medication delivery.
B. Less contact with nursing staff.
C. Rapid onset of analgesia.
D. Patient control over pain medication.
E. All of the above.
Answer: ACD

65. Advantages of epidural analgesia include:
A. Earlier mobilization after surgery.
B. Earlier return of bowel function.
C. Shorter hospitalizations.
D. Decreased stress response to surgery.
E. All of the above.
Answer: E

66. Ketorolac:
A. Is a nonsteroidal anti-inflammatory (NSAID) approved for intravenous, intramuscular, and oral administration.
B. Can be used indefinitely for postoperative analgesia.
C. Can cause renal dysfunction.
D. May decrease surgical blood loss.
Answer: AC

67. Factors that decrease collagen synthesis include all of the following except:
A. Protein depletion.
B. Infection.
C. Anemia.
D. Advanced age.
E. Hypoxia.
Answer: C

69. Which of the following is/are true of the actions of transforming growth factor beta (TGF-b) during wound repair?
A. Increased matrix and proteoglycan synthesis.
B. Inhibition of proteases.
C. Stimulation of plasminogen inhibitor.
D. Chemotaxis for fibroblasts and macrophages.
E. Autoinduction of TGF-b.
Answer: ABDE

70. In contrast to adult wound healing with scar formation, which of the following are characteristic of scarless fetal skin repair?
A. Matrix rich in hyaluronic acid.
B. Increased inflammatory response.
C. Increased production of TGF-b.
D. No collagen deposition.
E. Minimal angiogenesis.
Answer: AE

71. Which of the following cell types are not crucial for healing a clean, incisional wound?
A. Macrophage.
B. Platelet.
C. Fibroblast.
D. Polymorphonuclear leukocyte.
E. Myofibroblast.
Answer: DE

72. Which of the following is/are not a substrate or cofactor for prolyl hydroxylase?
A. Alpha-ketoglutarate.
B. Ascorbate.
C. Biotin.
D. Oxygen.
E. Copper.
Answer: CE

73. Which of the following is an adhesion glycoprotein?
A. Fibronectin.
B. Tenascin.
C. Laminin.
D. Hyaluronic acid.
E. Collagen type IV.
Answer: ABC

74. Which of the following is/are true concerning wound fibroblasts?
A. Fibroblasts synthesize and secrete collagen molecules.
B. Wound fibroblasts are derived from blood-borne precursor cells.
C. Fibroblasts migrate to a wound along fibrin strands, which are used as a scaffold.
D. Large amounts of fibrin or blood clot can act as a physical barrier to fibroblast penetration, which delays normal wound healing.
Answer: ACD

75. Which of the following is/are true?
A. Because of its thickness, the tensile strength of a healing wound on the eyelid is much less than one on the thick skin of the back.
B. By 2 days, the experimental burst strength of skin is minimal since collagen has been formed in the wound but has not yet cross-linked.
C. Wound strength reaches a plateau by 3 weeks.
D. Wounds rarely, if ever, regain the strength of intact tissues.
Answer: BD

76. Which of the following interfere with normal collagen formation or cross-linking?
A. Beta-aminopropionitrile.
B. Iron chelators.
C. Vitamin C depletion.
D. Proline analogs (e.g., cis-hydroxyproline).
E. D-Penicillamine.
Answer: ABCDE

77 Which of the following statement(s) is/are true concerning the cell plasma membrane?
a. The plasma membrane is composed of amphipathic molecules
b. The hydrophobic core of the lipid bilayer of the cell membrane contains specialized transport proteins which maintain the intracellular ionic milieu different from the extracellular fluid
c. Plasma membrane proteins extend externally and bear phospholipid moieties which contribute to the cell coat
d. The membrane proteins of nerve cells are highly voltage-dependent
Answer: a, b, d

78 Which of the following statement(s) is/are true concerning water movement across cell membranes?
a. Water moves only actively through cell membrane transport proteins
b. For most cells of the body, the transmembrane hydrostatic pressure is 0
c. Water distribution is determined entirely by solute distribution
d. Specialized cells such as the glomerulus of the kidney actively transport water to maintain hydrostatic pressure
Answer: b, c

79 The transport of proteins out of the cell is termed exocytosis. Which of the following statement(s) is/are true concerning this process?
a. Secretory vesicles fuse with the plasma membrane
b. The process can occur in either a constitutive or regulated process
c. A regulated secretion is triggered by a stimulus, most likely a hormone or a neurotransmitter
d. A decrease in cytoplasmic calcium occurs as part of the secretion process
Answer: a, b, c

80 Which of the following statement(s) is/are true concerning the cell function of phagocytosis?
a. Phagocytosis is a mechanistically distinct process of endocytosis performed by special cells to take up larger particles such as bacteria or erythrocytes
b. Lymphocytes are the primary blood cell involved with this process
c. The process involves a coating of the cytoplasmic surface known as clathrin
d. Phagocytosis is performed only by white blood cells and tissue macrophages
Answer: a

81 A striking feature of living cells is a marked difference between the composition of the cytosol and the extracellular milieu. Which of the following statement(s) concerning the mechanisms of maintenance of these differences is/are true?
a. The cell membrane is able to maintain a 10,000 fold gradient between the extracellular concentration of ionized calcium and the intracellular concentration
b. The key to these differences is the fact that the plasma membrane is normally impermeable to sodium, potassium and calcium
c. The selectivity of biologic membranes is highly consistent and seldom changes
d. The selectivity of cell membranes relates only to ions and not organic compounds

Answer: a

82 Which of the following statement(s) is/are true concerning DNA?
a. DNA is contained only in the nucleus of the cell
b. DNA strands are encoded by the sequence of four bases—adenine, guanine, cytosine and uridine
c. The basic unit of information of DNA is the intron, a sequence of three bases
d. There are an infinite number of possible codons
Answer: a

83 Which of the following statement(s) is/are true concerning cell membrane receptors?
a. The largest family of cell surface receptors are the G-protein-linked receptors
b. Activities of the G-protein involve binding and hydrolysis of ATP
c. The G protein receptor generates an intracellular messenger commonly through the use adenylate cyclase
d. Tyrosine kinase receptors are considered G-protein-linked receptors
Answer: a, c

84 Which of the following statement(s) is/are true concerning cellular ion channels?
a. Ion channels are transmembrane proteins that form pores that can conduct ions across the plasma membrane
b. Ion channels are formed by membrane-spanning peptides that are arranged so that polar moieties line a central core
c. Ion channel proteins undergo conformational changes between open states and closed states
d. Ion channels can be blocked
Answer: a, b, c, d

85 Which of the following statement(s) is/are true concerning carrier proteins?
a. Carrier proteins are distinguished by three types of mechanisms: carrier-type, channel-type, and conduction-type
b. Conformational changes in the membrane protein occur between the conducting and the nonconducting states
c. A channel-type carrier protein has two states—closed and open
d. Carrier-type transport proteins are equally accessible from either side of the membrane
Answer: b, c

86 Which of the following statement(s) is/are true concerning translation of the mRNA message to protein synthesis?
a. An adaptor molecule, tRNA, recognizes specific nucleic acid bases and unites them with specific amino acids
b. Covalent attachment of tRNA to amino acids is energy dependent
c. The formation of a peptide bond between the growing peptide chain and the free amino acid occurs in the free cytoplasm
d. Complete protein synthesis takes hours
Answer: a, b

87 Cell regulation can be thought of as the effector side of cell communication. Most commonly cell regulation occurs by means of extracellular chemical messengers. Which of the following statement(s) is/are true concerning these messengers?
a. Paracrine regulation involves a messenger which is produced and acts systemically
b. The extracellular signal or stimulus is received by a receptor on or in the target cell
c. Neurocrine regulation depends on a physical connection between the neuron and the target cell
d. Most hormones, local mediators, and neurotransmitters readily cross the cell plasma membrane
Answer: b, c

88 Proteins that are destined to be secreted from the cells must pass through a series of organelles. These organelles include:
a. Endoplasmic reticulum
b. Golgi apparatus
c. Mitochondria
d. Lysosomes
Answer: a, b, d

89 The best understood intracellular messenger is cyclic AMP (cAMP). Which of the following statement(s) concerning this intracellular messenger is/are correct?
a. Intracellular cyclic AMP is constantly degraded by a specific enzyme, cAMP phosphodiesterase
b. Most of the actions of cAMP are mediated by activation of protein kinase A
c. Intracellular levels of cAMP are relatively stable and change solely in response to activation of adenylate cyclase
d. cAMP is the only cyclic nucleotide active as an intracellular messenger
Answer: a, b

90 The activities of the cytoskeleton is dependent on which of the following types of filaments?
a. Microtubules
b. Intermediate filaments
c. Actin filaments
d. None of the above
Answer: a, b, c

91 Intracellular organelles involved with protein synthesis include:
a. Mitochondria
b. Endoplasmic reticulum
c. Golgi complex
d. Lysosomes
Answer: b, c

92 An important step in protein synthesis is transcription. Which of the following statement(s) is/are true concerning this process?
a. The first step in gene transcription involves separating the double helix of DNA by an enzyme known as DNA polymerase
b. The initial product of DNA transcription is called heterogeneous nuclear RNA which codes directly for proteins
c. After processing is complete, the mRNA is exported from the nucleus to the cytoplasm
d. Only one protein can be produced from an initial mRNA strand
Answer: c

93 There are two properties of the cell necessary to maintain nonequilibrium cellular composition; the first is selectivity and the second is energy conversion. Which of the following statement(s) is/are true concerning energy converting transport?
a. The site of energy conversion and transport in the plasma membrane involves the phospholipid component
b. The Na+-K++-ATPase derives energy from hydrolysis of extracellular ATP
c. In some systems, energy inherent in the transmembrane ion gradient can be used to drive transport of a second species
d. Examples of species transported via secondary active transport include hydrogen ions, calcium, amino acids and glucose
Answer: c, d

94 Which of the following statement(s) is/are correct concerning cell junctions?
a. The major occluding junction is the tight junction or zonula occludens
b. Tight junctions are usually located near the basal pole of the cell
c. Desmosomes are button-like points of attachment which serve to weld together adjacent cells
d. Gap junctions are a type of cell junction specialized for cell communication
Answer: a, c, d

95 Examples of ion channel blockers include:
a. Tetrodotoxin
b. Amiloride
c. Xylocaine
d. None of the above
Answer: a, b, c

96 Most hormone receptors are localized on the cell membrane and transduce hormone binding into altered levels of intracellular messengers. A limited number of intracellular receptors do exist. Which of the following statement(s) is/are true concerning intracellular receptors?
a. The messengers or hormones must by lipophilic
b. These intracellular receptors generally regulate protein synthesis
c. The intracellular receptors are located entirely in the nucleus of the cell
d. A heat-shock protein serves as an inhibitor protein blocking the DNA-binding domain of the steroid receptor
Answer: a, d

97 Altering the amino acid profile in total parenteral nutrition solutions can be of benefit in certain conditions. Which of the following conditions are associated with a benefit by supplementation with the amino acid type listed?
a. Acute renal failure and essential amino acids
b. Hepatic failure and aromatic amino acids
c. Short gut syndrome and glutamine
d. Chronic renal failure and essential amino acids
Answer: a, c

98 Under certain circumstances, the gut may become a source of sepsis and serve as the motor of systemic inflammatory response syndrome. Microbial translocation is the process by which microorganisms migrate across the mucosal barrier to invade the host. Which of the following mechanisms can promote bacterial translocation?
a. An increased number of gut bacteria
b. Altered intestinal mucosal permeability
c. Decreased host defense mechanisms
d. Lack of enteral feeding

Answer: a, b, c, d

99 Translocation is promoted in three general ways: 1) altered permeability of the intestinal mucosa as caused by shock, sepsis, distant injury, or cell toxins; 2) decreased host defense (secondary to glucocorticoid administration, immunosuppression, or protein depletion; and 3) an increased number of bacteria within the intestine. Because many factors that facilitate bacteria translocation occur simultaneously in surgical patients, these effects may be either additive or cumulative. In addition, many patients in Surgical Intensive Care Units do not generally receive enteral feedings and therefore current parenteral therapy results in gut atrophy which further promotes translocation.
Which of the following statement(s) is/are true concerning nutritional support of the injured patient?
a. The goal of nutritional support is maintenance of body cell mass and limitation of weight loss to less than 25% of preinjury weight
b. Under-nutrition may compromise the patient’s available defense mechanisms
c. Nutritional support is an immediate priority for the trauma patient
d. Fifty percent of non-nitrogen caloric requirements should be provided in the form of fat
Answer: b

100 Which of the following statement(s) is/are true concerning body fuel reserves?
a. The largest fuel reserve in the body is skeletal muscle
b. Fat provides about 9 calories/gram
c. Free glucose and glycogen stores are a trivial fuel reserve
d. Body protein is a valuable storage form of energy
Answer: b, c

101 Which of the following statement(s) is/are true concerning the indications and administration of nutritional support to cancer patients?
a. Preoperative nutritional support should be provided to all patients with cancer
b. To be effective, preoperative nutrition must be given for at least two weeks preoperatively
c. Parenteral nutrition is the preferred route of feeding for all cancer patients
d. Standard total parenteral nutrition solutions maintain integrity of the small bowel
e. None of the above
Answer: e

The role of nutritional support in the cancer patient remains an important component of overall therapy. Preoperative nutritional support should be given only to those patients who do not require an emergency operation and who have severe weight loss (> 15% of pre-illness body weight) and a serum albumen < 2.9 mg%. Preoperative nutrition (enteral or parenteral) should not be given for longer than 7 to 10 days. Enteral nutrition is always the preferred route of feeding cancer patients if the GI tract is functional. There are several benefits of using the bowel lumen for nutrient delivery. The trophic effects of enteral feeding on small bowel mucosa have been well described. The integrity of the mucosal lining is maintained and it may provide an effective barrier to intraluminal enteric organisms which might otherwise translocate into the systemic circulation. Atrophic changes may be seen in the intestinal epithelium after several days of bowel rest; this atrophy is not reversed by currently available total parenteral nutrition solutions.

102 Which of the following hormones can be expected to be released as part of the stress response?
a. Antidiuretic hormone (ADH)
b. Aldosterone
c. Insulin
d. Epinephrine
nswer: a, b, d

Several important responses occur in response to stress. The body immediately attempts to compensate for a reduction in circulating blood volume in order to maintain adequate organ perfusion. Afferent nerve signals are also initiated which stimulate the release of both antidiuretic hormone (ADH) and aldosterone. The pain and fear associated with the stress response lead to excessive production to catecholamines which also increase metabolic rate, stimulate lipolysis, hepatic glycolysis, and gluconeogenesis. Glucagon, which has a potent glycogenolytic and gluconeogenic effect in the liver, is also released. This hormone has the exact opposite effect of insulin, which promotes glucose storage and uptake by the cells.

103 Cytokines which play an important role in the metabolic response to injury include:
a. Tumor necrosis factor—a (TNF)
b. Interleukin-1 (IL-1)
c. Interleukin-6 (IL-6)
d. Interferon-g
Answer: a, b, c, d

TNF or cachetin is considered the primary mediator of the systemic effects of endotoxin, producing anorexia, fever, tachypnea, and tachycardia at low doses and hypotension, organ failure, and death at higher doses. TNF is produced primarily by macrophages, but lymphocytes, Kupffer cells, and a number of other cell types have been identified as sources of TNF. IL-1, like TNF, has a variety of pro-inflammatory activities. IL-6 is now recognized as a primary mediator of altered hepatic protein synthesis known as acute-phase protein synthetic response. Glucocorticoid hormones augment the cytokine effects on acute phase protein synthesis. Interferons are a family of proteins which are readily identified for their ability to inhibit viral replication in infected sells. IFN-g has the ability to upregulate the number of TNF receptors on various cell types.

104 A 16-year-old boy suffers a mid-gut volvulus with massive loss of small intestine. Which of the following statement(s) is/are true concerning his nutritional requirements and management?
a. If at least 18 inches of residual small intestine survives, the patient may tolerate some form of enteral nutrition
b. A nutritional regimen consisting of supplemental glutamine, growth hormone, and a modified high carbohydrate, low fat diet may be beneficial in this patient
c. The regimen described above may decrease the cost of care
d. TPN needs will increase after discontinuation of growth hormone
Answer: a, b, c

Prior to the availability of TPN, most patients developing short bowel syndrome from either surgery or catastrophic event died. In selected patients, however, with residual small intestine (at least 18 inches), post-resectional hyperplasia may develop with time such that they can tolerate enteral feeds. Recent studies have demonstrated the requirement for TPN could be decreased or even eliminated in patients with short-gut syndrome by providing a nutritional regimen consisting of supplemental glutamine, growth hormone, and a modified high carbohydrate, low fat diet. There was a marked improvement in absorption of nutrients with this combination of therapy and a decrease in stool output. In addition, TPN requirements were reduced by 50% as were costs associated with the care of these individuals. Discontinuation of the growth hormone did not increase TPN needs in patients once they had undergone successful gut rehabilitation.

105 A number of changes in trace mineral metabolism are noted during sepsis. Which of the following change(s) may be observed in a septic or trauma patient?
a. Plasma iron levels are noted to decrease
b. Plasma copper levels are noted to decrease
c. Plasma serum zinc levels may decrease
d. Administration of iron is appropriate
Answer: a, c

Changes in the balance of magnesium, inorganic phosphate, zinc, and potassium generally follow alterations in nitrogen balance. Although the iron-binding capacity of transferrin is usually unchanged in early infection, iron disappears from the plasma, especially during severe pyogenic infection; similar alterations are observed in serum zinc levels. The administration of iron to the infected host, especially early into the disease, is contraindicated, however, because increased serum iron concentrations may impair resistance. Unlike iron and zinc, copper levels generally rise, and the increased plasma concentrations can be ascribed almost entirely to the levels of the ceruloplasmin produced by the liver.

106 A 17-year-old patient involved in an automobile accident is paralyzed with multiple peripheral extremity injuries. Nutritional support is instituted with a transnasal feeding catheter. Which of the following statement(s) is/are true concerning the patient’s management?
a. Feeding into the stomach results in stimulation of the biliary/pancreatic axis which is probably trophic for small bowel
b. Gastric secretions will dilute the feedings increasing the risk of diarrhea
c. The major risk in this patient is tracheobronchial aspiration
d. Placement of the feeding catheter through the pylorus into the first portion of the duodenum reduces the risk of regurgitation and aspiration
Answer: a, c, d

The use of transnasal feeding catheters for intragastric feeding or for duodenal intubation are popular adjuncts for providing nutritional support by the enteral route. The stomach is easily accessed by passage of a soft flexible feeding tube. Intragastric feeding provides several advantages for the patient. The stomach has the capacity and reservoir for bolus feedings. Feeding into the stomach results in stimulation of the biliary/pancreatic axis which is probably trophic for the small bowel. Gastric secretions will have a dilutional effect on the osmolarity of the feedings, reducing the risk of diarrhea. The major risk of intragastric feeding is the regurgitation of gastric contents resulting in aspiration into the tracheobronchial tree. This risk is highest in patients who have an altered sensorium or who are paralyzed. The placement of the feeding tube through the pylorus into the fourth portion of the duodenum reduces the risk of regurgitation and aspiration of feeding formulas.

107 Although TPN has major beneficial effects to the patient and specific organ systems, TPN has a downside which is related to intestinal disuse. Which of the following statement(s) is/are true concerning the effects of TPN on the GI tract?
a. Patients receiving TPN have an accentuated systemic response to endotoxin challenge compared to enterally fed volunteers
b. TPN can result in disruption of intestinal microflora
c. In experimental models, bacterial translocation from the gut is increased
d. Effects of TPN on the gut may lead to multiple organ failure
nswer: a, b, c, d

A number of studies have examined the effects of TPN on intestinal function and immunity. Although most of these studies have been done in animal models, TPN has consistently been shown to have some detrimental effects. In rats, TPN results in significant disruption of the intestinal microflora and bacterial translocation of the gut to the mesenteric lymph nodes. In addition, when stresses such as a burn injury, chemotherapy, or radiation are introduced into these models, animals on TPN have a much higher mortality. The body of literature suggests that TPN under certain circumstances may predispose patients to an increase in gut-derived infectious complications. In a study in human volunteers, individuals receiving TPN had an accentuated systemic response to endotoxin challenge compared to enterally fed volunteers. This study is consistent with impairment of gut barrier function during parenteral feedings which may promote the release of bacteria and/or cytokines leading to pronounced systemic responses and possibly multiple organ failure.

108 Total body mass is composed of an aqueous component and a nonaqueous component. The nonaqueous component is made up of which of the following?

a. Liver
b. Tendons
c. Skeletal muscle
d. Extracellular fluid
e. Adipose tissue
Answer: b, e

The nonaqueous portion of total body mass is made up of bones, tendons, and mineral mass as well as adipose tissue. The aqueous component contains the body cell mass which is made up of skeletal muscle, intraabdominal and intrathoracic organs, skin, and circulating blood cells. Also contributing to the aqueous portion is the interstitial fluid and intravascular volume.

109 Fatty acids are a major energy source for the body. Which of the following statement(s) is/are true concerning the use of fatty acids as an energy source?
a. Fatty acids are stored in adipocytes as triglycerides
b. Hormone-sensitive lipase is present only in adipose tissue
c. Fatty acids are released into the circulation traveling freely in plasma
d. Approximately 25% of total nonprotein caloric needs supplied via total parenteral nutrition should be in the form of fat
Answer: a, b, d

In most tissues, fatty acids are readily oxidized for energy. They are especially important energy sources for the heart, liver and skeletal muscle. In adipose tissue, fatty acids may be re-esterified with glycerol and stored as triglycerides in adipocytes. Stored fat is mobilized during starvation and stress. Hormone-sensitive lipase, present only in adipose tissue, catalyzes the breakdown of stored triglycerides into glycerol and fatty acids. The fatty acids that are produced are released in the circulation. The major lipids in plasma do not circulate in a free form, thus free fatty acids must be bound to albumin. During stress, the activity of hormone-sensitive lipase is increased which leads to mobilization of fat stores. However, fat remains an important fuel source for critically ill patients and as a rule the amount of fat administered to patients receiving total parenteral nutrition should comprise about 5–30% of total nonprotein caloric needs.

110 Which of the following metabolic effects may be observed in patients with sepsis?
a. Increased gluconeogenesis
b. Accelerated proteolysis
c. Increased lipolysis
d. Impaired gut metabolism of glutamine
Answer: a, b, c, d

A number of metabolic responses to sepsis have been defined. Glucose production is increased in infected patients which appears to be additive to the augmented gluconeogenesis that occurs following injury. Accelerated proteolysis, increased nitrogen excretion and prolonged negative nitrogen balance also occur following infection with a response pattern similar to that described with injury. Severe infection is often associated with a hypercatabolic state that initiates marked changes in interorgan glutamine metabolism. This process results in accelerated muscle proteolysis and net skeletal muscle glutamine release. The bulk of glutamine is taken up by the liver at the expense of the gut. It appears that sepsis can impair gut metabolism of glutamine. Fat is a major fuel oxidized in infected patients, and increased metabolism of lipids from peripheral fat stores is especially prominent during a period of inadequate nutritional support.

111 Which of the following statement(s) is/are true concerning protein/amino acid metabolism in man?
a. The major source of amino acids is breakdown of circulating proteins
b. The recommended daily allowance for protein may triple in critically ill patients
c. Urinary nitrogen losses will approach 0 in the face of protein starvation
d. Negative nitrogen balance refers to a decrease in nitrogen taken into the body versus the amount of nitrogen lost
Answer: b, d

About 15% of the total body weight is made up of proteins, about half of which are intracellular and half extracellular. In man and other animals, dietary protein is the source of most amino acids. Intestinal absorption is the only physiological pathway by which the body obtains exogenous amino acids. Digestion of ingested protein provides free amino acids that are absorbed by the small intestine and transported to the liver where they can be incorporated into new proteins or other biosynthetic products. Excess amino acids are degraded and their carbon skeleton is oxidized to produce energy or it is incorporated into glycogen or into free fatty acids. In addition to the metabolism of dietary amino acids, the existing proteins in the cell are continuously recycled, such that total protein turnover in the body is about 300 g/day. Vertebrates cannot reutilize nitrogen with 100% efficiency; therefore, obligatory nitrogen losses occur, mainly in the urine. Urinary nitrogen losses will diminish when individuals are fed a protein-free diet, but will never become 0 because of the body’s inability to completely reutilize nitrogen. In stressed patients, this ability to adapt to starvation is compromised such that proteolysis of body proteins continues at a substantial rate. This increases the amount of obligatory nitrogen losses which are accentuated by the catabolic disease states. This results in a negative nitrogen balance in which the amount of nitrogen taken in by the patient is exceeded by the amount of nitrogen lost in the urine, stool, skin, wounds, and fistula drainage.

112 Which of the following statement(s) concerning intravenous nutritional support is/are true?
a. Concentrations of glucose no higher than 5% should be used to avoid peripheral vein sclerosis
b. A major disadvantage of the peripheral technique is limited caloric delivery
c. If total parenteral nutrition is required, access to the superior vena cava via the external jugular vein is the most suitable site
d. Venous thrombosis is an uncommon complication for long-term central vein catheterization
Answer: b

Although peripheral access can be used for intravenous nutrition, the major disadvantage of this technique is limited caloric delivery to meet catabolic demands within tolerated fluid limits. Infusion of glucose (up to 10%), amino acid solutions, and fat emulsions can be administered peripherally but these solutions must be nearly isotonic to avoid peripheral vein sclerosis. The preferred method of access for total parenteral nutrition is into the superior vena cava by cutaneous cannulation of the subclavian vein. Alternative sites include the internal and external jugular vein but the catheter exiting from the neck region makes it more difficult to secure and maintain a sterile dressing. Complications from long-term central venous catheterization include venous thrombosis and venous catheter-related infection. Thrombosis of central vessels is a complication which is often overlooked. The clinical suspicion of subclavian vein thrombosis is only about 3%, whereas studies that use phlebography or radionucleotide venography indicate the incidence is as high as 35%.

113 Sepsis causes a marked metabolic response. Which of the following statement(s) is/are true concerning the metabolic response to sepsis?
a. Oxygen consumption is increased in the face of infection
b. In a patient with a maximal metabolic rate secondary to trauma, the presence of infection will increase the rate further
c. Metabolic rate increases at a rate of approximately 10% for each increase of 1°C in central temperature
d. The extent of increase in oxygen consumption relates to the severity of the infection
Answer: a, c, d

Oxygen consumption is usually elevated in the infected patient. The extent of this increase is related to the severity of the infection, with peak elevations reaching 50% to 60% above normal. If the patient’s metabolic rate is already elevated to a maximal extent because of severe injury, no further increase will be observed. In patients with only a slightly accelerated rate of oxygen consumption, the presence of infection will cause a rise in metabolic rate added to the preexisting state. A portion of the increase in metabolism may be ascribed to increase in reaction rate associated with fever. Calculations suggest that the metabolic rate increases 10% to 13% for each elevation of 1°C in central temperature.

114 Interleukin-6 is recognized as the cytokine primarily responsible for the alteration in hepatic protein synthesis recognized as the acute phase response. Which of the following statement(s) is/are true concerning acute phase protein response to surgical stress?
a. Glucocorticoid hormones inhibit this response
b. Proteins such as albumin and transferrin which serve in serum transport are generally increased in this response
c. Examples of acute phase proteins include fibrinogen and C-reactive protein
d. In general, the physiologic role of acute phase proteins are to reduce the systemic effects of tissue damage
Answer: c, d

IL-6 is now recognized at the cytokine primarily responsible for the alteration in hepatic synthesis recognized as the acute phase response. Glucocorticoid hormones augment this response. The primary metabolic component of the acute phase response is a qualitative alteration in hepatic protein synthesis with resulting alteration in plasma protein composition. Characteristically, proteins which act as serum transport in binding molecules, (albumin, transferrin) are reduced in quantity and acute phase proteins (fibrinogen, C-reactive proteins) are increased. Acute phase proteins are elaborated for the purpose of reducing the systemic effects of tissue damage. Many act as anti-proteases, opsonins, or coagulation and wound healing factors that generally inhibit the tissue destruction that is associated with the local initiation of inflammation.

115 A 59-year-old trauma patient has suffered multiple septic complications including severe pneumonia, intraabdominal abscess, and major wound infection. He has now developed signs of multisystem organ failure. Which of the following statement(s) is/are true concerning necessary changes to be made in his nutritional management?
a. Carbohydrate load should be reduced in the face of respiratory failure
b. In patients with renal failure, protein intake should be diminished
c. During hemodialysis protein intake should be limited to the same extent
d. In patients with hepatic failure, carbohydrate load should be increased
Answer: a, b

The most severe complication of sepsis is multiple system organ dysfunction syndrome, which may result in death. The development of organ failure requires changes in the nutritional requirements and creates special feeding problems. A problem associated with systemic infection is oxygenation and elimination of carbon dioxide. Most of the enteral and parenteral formulas used to provide nutritional support for critically ill patients contain large amounts of carbohydrate, which generate large amounts of carbon dioxide following oxygenation. Such a large CO2 load may worsen pulmonary function or may delay weaning from the respirator. If this factor becomes a problem, the carbohydrate load should be reduced to 50% of metabolic requirements and fat emulsion administered to provide additional calories. When renal failure becomes progressive, the use of hemodialysis minimizes the effect of uremia superimposed on the metabolism of sepsis. Metabolic studies in patients with acute and chronic renal failure have limited the intake of nonessential amino acids, in an attempt to lower urea production. Proteins of high biologic value, but in much smaller quantities than usually given, are administered along with adequate calories, usually in the form of glucose. When enteral feedings are not feasible, a central venous infusion of an essential amino acid solution and hypertonic dextrose provides calories and a small quantity of nitrogen to reduce protein catabolism while simultaneously controlling the rise in BUN. During dialysis, protein intake is liberalized, but the BUN should still be maintained below 100 mg/dl. Hepatic dysfunction is a common manifestation of septicemia. The carbohydrate load is usually reduced to consist of no more than 5% of metabolic requirements, and the additional calories should be provided as fat emulsion. If encephalopathy develops, protein load should also be reduced0.

116 Which of the following statement(s) is/are true concerning the role of glutamine in total parenteral nutrition?
a. Glutamine is an essential amino acid
b. Glutamine appears to be of primary benefit in critical illness
c. Glutamine is included in most standard TPN solutions
d. Glutamine is the primary energy source for intestinal mucosal cells of the small bowel and colon
Answer: b

Glutamine is the most studied gut-specific nutrient. Glutamine has been classified as a nonessential or nutritionally dispensable amino acid since glutamine can be synthesized in adequate quantities from other amino acids and precursors. Glutamine is not included in most nutritional formulas and has been eliminated from TPN solutions because of its relative instability and short half life compared to other amino acids. With few exceptions, glutamine is present in oral enteral diets but only at relatively low levels characteristic of the concentration in most animal and plant stores (about 7% of total amino acids). Several recent studies, however, have demonstrated that glutamine may be an essential amino acid during critical illness, particularly as it relates to supporting the metabolic requirements of the intestinal mucosa. These studies demonstrate that dietary glutamine is not required during states of health but appears to be beneficial when glutamine depletion is severe and/or when intestinal mucosa is damaged by insults such as chemotherapy or radiation therapy. The addition of glutamine to enteral diet reduces the incidence of gut translocation but these improvements are dependent upon the amount of supplemental glutamine and the type of insult studied. Glutamine-enriched TPN partially attenuates villous atrophy that develops during parenteral nutrition. The use of intravenous glutamine in patients appears to be safe and effective in its ability to maintain muscle glutamine stores and improve nitrogen balance. In contrast to glutamine, short chain fatty acids are primary energy source for colonocytes.

117 Which of the following are determinants of the host response to surgical stress?
a. Gender
b. Age
c. Nutritional status
d. Body composition
Answer: a, b, c, d

The pattern of physiologic changes elicited in response to surgical stress results from the specific interaction of an individual patient with a stressful stimulus. Several factors specific to the patient may determine the nature of the host response to stress. Body composition is a major determinant of the metabolic responses observed during surgical illness. Post-traumatic nitrogen excretion is directly related to the size of the body protein mass. A strong relationship between protein depletion and postoperative complications has been demonstrated in nonseptic, nonimmunocompromised patients undergoing elective major gastrointestinal surgery. Protein-depleted patients have significantly lower preoperative respiratory muscle strength and vital capacity, increased incidence of postoperative pneumonia, and longer postoperative hospital stay. Impaired wound healing and respiratory, hepatic, and muscle function in protein-depleted patients awaiting surgery has also been reported. Many of the changes in the metabolic responses to surgical illnesses that occur with aging can be attributed to alterations in body composition and to long-standing patterns of physical activity. Fat mass tends to increase with age and muscle mass tends to decrease. Loss of strength that accompanies immobility, starvation and acute surgical illness may have marked functional consequences. Furthermore, the prevalence of cardiovascular and pulmonary diseases increase with age. Thus, the delivery of oxygen to tissues may be impaired in the elderly. Finally, observed differences in metabolic responses of men and women generally reflect differences in body composition. Lean body mass is lower in women than in men; and this difference is thought to account for the net loss of nitrogen after major elective abdominal surgery generally being lower in women than in men.

118 In contrast to a patient undergoing an elective operation, which of the following statement(s) is/are true concerning a patient who has suffered a multiple trauma?
a. Basal metabolic rates are similar
b. The patient is highly sensitive to insulin
c. Utilization of the amino acids, glutamine and alanine, is similar to their composition in skeletal muscle
d. Fat and protein stores are rapidly depleted
Answer: b, d

The degree of hypermetabolism is generally related to the severity of injury. Patients with long-bone fractures have a 15–25% increase in metabolic rate, whereas metabolic rates in patients with multiple injuries increases by 50%. These metabolic rates in trauma patients are contrasted with those in postoperative patients, who rarely increase their BMR by more than 10–15% following operation. Studies have shown that uninjured volunteers are able to dispose of exogenous glucose load much more readily than injured patients. Other studies have demonstrated a failure to suppress hepatic glucose production in trauma patients during glucose loading or insulin infusion. Thus, profound insulin resistance occurs in injured patients. Skeletal muscle is the major source of nitrogen that is lost in the urine following extensive injury. Although it is recognized that amino acids are released by muscle in increased quantities following injuries, it has only been recently appreciated that the composition of amino acid reflux does not reflect the composition of muscle protein. The release is skewed towards glutamine and alanine, each of which comprise about one-third of the total amino acids released by skeletal muscle. To support hypermetabolism, stored triglyceride is mobilized at an accelerated rate. Although mobilization and use of free fatty acids are accelerated in injured subjects, if unfed, severely injured patients rapidly deplete their fat and protein stores.

119 A 47-year-old patient undergoing a complicated laparotomy for bowel obstruction develops a postoperative enterocutaneous fistula. Which of the following statement(s) is/are true concerning parenteral nutritional support in the postoperative period?
a. Oral intake can result in severe dehydration, electrolyte abnormalities, and perifistula skin injury
b. Total parenteral nutrition increases the spontaneous closure rate of intestinal fistula
c. Total parenteral nutrition decreases mortality rate in patients with intestinal fistulas
d. The use of TPN better prepares the patient for surgery if surgical intervention proves necessary
Answer: a, b, d

Patients with gastrointestinal-cutaneous fistulas represent the classical indication for TPN. In such patients, oral intake of food almost invariably results in increased fistula output with associated metabolic disturbances, dehydration, skin breakdown, and death. Several comprehensive reviews have concluded that TPN clearly impacts on the treatment course of the disease in patients with GI fistulas. The following conclusions can be drawn from studies evaluating the use of TPN in patients with enterocutaneous fistula. First, TPN increases spontaneous closure rate of enterocutaneous fistulas but does not markedly decrease the mortality rate in patients with fistulas. Second, if spontaneous closure of the fistula does not occur, patients are better prepared for operative intervention because of the nutritional support they have received. Finally, certain fistulas are associated with a lower rate of spontaneous closure than others and should be treated more aggressively surgically after a defined period of nutritional support (unless closure occurs).

120 Appropriate guidelines for the use of TPN in cancer patients include:
a. Long-term TPN in patients with rapid progressive tumor growth unresponsive to other therapy
b. Mildly malnourished patients undergoing surgery for a curable cancer
c. Preoperatively administered TPN prior to surgery or other therapy in patients with severe malnutrition
d. Patients in whom treatment toxicity precludes the use of enteral nutrition
Answer: c, d

As a general rule, the most important factor to consider when making decisions about the use of TPN in patients with cancer is the response of the tumor to antineoplastic therapy. Appropriate guidelines would include the following: Short-term TPN is indicated in severely malnourished patients or in those in whom gastrointestinal or other toxicities preclude adequate enteral intake for seven days or a longer period. TPN is not indicated in well nourished or mildly malnourished patients undergoing therapy or surgery who would be expected to be able to resume adequate nutrition in approximately seven days. Long-term TPN is indicated in patients in whom treatment associated toxicities preclude the use of enteral nutrition and represent the primary impediment to the restoration of performance status. These patients should be expected to be responding to anti-tumor therapy. Long-term TPN is not indicated with rapidly progressive tumor growth which is unresponsive to such therapy.

121 Which of the following statements(s) is/are true concerning human energy requirement?
a. In normal subjects, less than 5% of basal energy requirement is spent on cardiac output and the work of breathing
b. Mechanical ventilation can decrease the energy expenditure for normal respiration
c. For a 70 kg male, average resting energy consumption is almost 1500 kcal/day
d. Similar increases in energy expenditures are associated with elective surgery and trauma or thermal injury
Answer: a, c

138 Which of the following statement(s) is/are true concerning wound contraction?
a. Wound contraction accounts for similar rates of reduction of wound size regardless of their location
b. The fibroblast, at the cellular level, is the primary force driving wound contraction
c. Excessive wound contraction, when occurring over a joint, may lead to disability
d. Actin microfillaments are found in fibroblasts and may play a role in wound contracture
Answer: b, c, d

Wound contraction is an important event which contrasts healing open wounds and closed incisions. When open wounds contract, the surrounding skin is pulled over the open wound to reduce its size. This can occur much faster than epithelialization. As opposed to other animals, human skin does not have a significant degree of mobility in most sites and specifically on the lower leg, the skin is tightly adherent and less elastic. Therefore, although contraction may account for 90% of reduction of wound size on the perineum, it accounts for, at most, 30–40% of healing of a lower leg ulcer. All healing wounds generate a strong contractile force. When this force is exerted across a joint, it may result in scar contracture which may limit the functional range of motion. At the cellular level, the force which drives wound contraction comes from fibroblasts. Fibroblasts, like muscle cells, contain actin microfilaments. When these filaments increase in number, the cells take a morphologic appearance of myofibroblasts. Myofibroblasts are seen in an increased number in contracting wounds and are felt to play an active role in the process of wound contraction.

139 There are a multitude of various dressings available. Which of the following statement(s) is/are true concerning options for surgical dressings?
a. Hydrocolloids, such as karaya compounds, offer the primary advantage of increased absorptive ability
b. Films, such as Op-site, provide a water impermeable environment to achieve a dry wound
c. Impregnates are fine gauze impregnated with a variety of substances such as antibiotics or moisturizing agents that adhere tightly to the wound and do not require a secondary dressing
d. Absorptive powders and paste are highly useful in debriding necrotic and fibrous material from wounds and absorbing wound serum
Answer: a, d

Although the simplest dressing of gauze and tape combined with the use of antibacterial ointment can achieve moist wound healing in most patients. A multitude of other products are available. These can be classified into films, foams, hydrocolloids, hydrogels, and absorptive powders. Films are semipermeable to water, generally made of polyurethane, and are nonabsorptive. They are useful to achieve a moist wound healing environment over a minimally exudative wound such as split thickness skin graft donor sites. The hydrocolloids deserve special mention because they have achieved widespread use. These agents contain hydrophilic materials such as karaya or carboxymethyl cellulose with an adhesive material and are covered by a semipermeable polyurethane film. The material adheres to the skin surrounding the wound, is highly absorptive, and achieves a moist healing environment. Impregnants are generally fine mesh gauze impregnated with either moisturizing, antibacterial, or bactericidal compounds. They are generally not adherent and require a secondary dressing. They do promote reepithelialization and have a antiinfective effect when combined with antibacterial or bactericidal agents. A variety of absorptive powders and pastes are available which consist of starch copolymers or colloidal hydrophilic particles. These agents have high absorbency for tissue wound fluid and debride necrotic and fibrous material from the wound.

140 Which of the following statement(s) is/are true concerning the remodeling phase of wound healing?
a. Total collagen content increases steadily through this phase
b. The normal adult ratio of collagen is approximately 4:1 of type I to type III collagen.
c. Eventually a scar will achieve the strength of unwounded skin
d. The proteoglycans are responsible for the ground substance of the extracellular matrix
Answer: b, d

The transition from the proliferative phase to the remodeling phase of wound healing is defined by reaching collagen equilibrium. Collagen accumulation within the wound becomes maximal by two to three weeks after wounding. Although supramaximal rates of synthesis and degradation continue throughout remodeling, there is no further change in total collagen content. During the initial phase of wound healing, there is a relative abundance of type III collagen in the wound. With remodeling, the normal adult ratio of 4:1 (type I to type III) collagen is restored. The other important component of the extracellular matrix is the ground substance or proteoglycans. These substances are composed of a protein background with long hydrophilic carbohydrate side chains. The hydrophilic nature of these molecules accounts for much of the water content of scar.
Scars never achieve the degree of order advanced by collagen in normal skin or tendons, but they do increase in strength for six months or more, eventually reaching 70% of the strength of unwounded skin.

141 Which of the following statement(s) is/are true concerning pharmacologic agents used to accelerate wound healing?
a. A number of these agents are now currently approved for use in this country
b. PDGF (platelet-derived growth factor) promotes fibroblast proliferation, chemotaxis, and collagenase synthesis
c. PDGF has been demonstrated in a number of clinical trials to promote healing in chronic wounds
d. Growth hormone functions by promoting fibroblast proliferation and collagen synthesis
Answer: b, c

Currently there are no approved clinical agents that accelerate normal healing. Although a number of clinical trials are in progress, no agents are currently approved. PDGF (platelet-derived growth factor) accelerates wound healing by promoting fibroblast proliferation and chemotaxis and collagenase synthesis. Clinical trials have demonstrated that PDGF has accelerated healing in patients with chronic wounds such as pressure sores and diabetic ulcers. Growth hormone has been successfully used in some situations to reverse the catabolic effect of severe injuries. Wound healing is fundamentally an anabolic event, and in the setting of a severe burn, growth hormone administration significantly accelerates donor site healing, presumably due to its effects in minimizing catabolism.

142 Which of the following statement(s) describe the effects of aging on wound healing?
a. A finer, more cosmetic scar might be expected
b. In vitro studies demonstrate decreased proliferative potential of fibroblasts and epithelial cells
c. Skin sutures should be left in for a longer period of time
d. Wound infection occurs more frequently in elderly patients due to diminished ability to fight infection
Answer: a, b, c

There are important age-dependent aspects of wound healing. The elderly heal more slowly and with less scarring. There is a gradual attenuation of the inflammatory response with age, and decreased wound healing is one of the consequences. In vitro studies have documented an age-dependent decrease in proliferative potential of fibroblasts and epithelial cells. Clinically this will account for the formation of finer scars and improved cosmetic appearance in the elderly. Sutures should be left in place longer to allow for the slow regain of tensile strength in the aged. This can also be done without concern for formation of suture marks as slower epithelialization occurs along the sutures. There is no evidence to suggest that wound infections occur more commonly in elderly patients.

143 Reconstitution of the epithelial barrier (epithelialization) begins within hours of the initial injury. Which of the following statement(s) is/are true concerning the process of epithelialization?
a. Bacteria, protein exudate, and necrotic tissue all will compromise this process
b. Epithelial cells exhibit contact proliferation
c. Epithelialization occurs only from the margins of the wound
d. Visible scarring can occur only when the injury extends deeper than the superficial dermis
Answer: a, d

The initial step of epithelialization involves epithelial cells from the basal layer of the wound edge flattening and migrating across the wound, completing wound coverage within 24–48 hours in a co-opted surgical wound. Epithelial cells exhibit contact inhibition. That is, they will continue to migrate across an appropriate bed until a single continuous layer is formed. Epithelial cell migration occurs by a process in which the epithelial cells send out pseudopods, attaching to the underlying extracellular matrix by integrin receptors. Bacteria, large amounts of protein exudate from leaky capillaries, and necrotic tissue all compromise this process delaying epithelialization. In the case of open wounds, epithelialization results from migration of epithelial cells from remaining dermal appendages, sweat glands, and hair follicles, if the dermis is not completely destroyed. In a full thickness injury, the entire dermis is destroyed or removed. Epithelialization therefore occurs only at the margins of a wound, at a dermal rate of 1–2 mm/day.
Visible scarring occurs only when the injury extends deeper than the superficial dermis. Superficial abrasions and burns usually heal without scar, while deeper abrasions and burns may scar significantly. Whenever the dermis is incised, a scar will form.

144 Scar formation is part of the normal healing process following injury. Which of the following tissues has the ability to heal without scar formation?
a. Liver
b. Skin
c. Bone
d. Muscle
Answer: c

Every tissue in the body undergoes reparative processes after injury. Bone has the unique ability to heal without scar and liver has the potential to regenerate parenchyma, the only organ that has maintained that ability in the adult human. Although liver does regenerate, it often heals with scar (cirrhosis) as well. With these exceptions, all other mature human tissues heal with scar.

145 Which of the following factors have been demonstrated to promote wound healing in normal individuals?
a. Vitamin A supplementation
b. Vitamin C supplementation
c. Vitamin E application to the wound
d. Zinc supplementation
e. None of the above
Answer: e

Several important systemic factors or conditions influence wound healing. Interestingly, there are no known systemic conditions that lead to enhanced or more rapid wound healing. Overall nutrition as well as adequate vitamins play an important role in wound healing. Vitamin A is involved in the stimulation of fibroplasia, collagen cross-linking, and epithelialization. Although there is no conclusive evidence in humans, vitamin A may be useful clinically for steroid-dependent patients who have problematic wounds or who are undergoing extensive surgical procedures. Vitamin C is a necessary cofactor in hydroxylization of lysine and proline in collagen synthesis and cross-linkage. The utility of vitamin C supplementation in patients who otherwise take in a normal diet has not been established. Vitamin E is applied to wounds and incisions empirically by many patients. The evidence to support this practice is entirely anecdotal. In fact, large doses of vitamin E have been found to inhibit wound healing. Zinc and copper are also important cofactors for many enzyme systems that are important to wound healing. Deficiency states are seen with parenteral nutrition but are rare and readily recognized and treated with supplements. Overall, vitamin and mineral deficiency states are extremely rare in the absence of parenteral nutrition or other extreme dietary restrictions. There is no evidence to support the concept that supranormal provision of these factors enhance wound healing in normal patients.

146 Which of the following statement(s) is/are true concerning excessive scarring processes?
a. Keloids occur randomly regardless of gender or race
b. Hypertrophic scars and keloid are histologically different
c. Keloids tend to develop early and hypertrophic scars late after the surgical injury
d. Simple reexcision and closure of a hypertrophic scar can be useful in certain situations such as a wound closed by secondary intention
Answer: d

True keloids are uncommon and occur predominantly in dark skinned people with a genetic predisposition for keloid formation. In most cases, the gene appears to be transmitted as an autosomal dominant pattern. The primary difference between a keloid and a hypertrophic scar is that a keloid extends beyond the boundary of the original tissue injury. It behaves as a tumor and extends into or invades the normal surrounding tissue creating a scar that is larger than the original wound. Histologically, keloids and hypertrophic scars are similar. Both contain an overabundance of collagen. Although the absolute number of fibroblasts is not increased, the production of collagen continually out paces the activity of collagenase, resulting in a scar of ever increasing dimensions. Hypertrophic scars respect the boundaries of the original injury and do not extend into normal unwounded tissue. There is less of a genetic predisposition, but hypertrophic scars also occur more frequently in Orientals and the Black population. They are often seen on the upper torso and across flexor surfaces. Some improvement in a keloid can be obtained with excision followed by intra-lesional steroid injection. However, the resulting scar is unpredictable and potentially worse. Reexcision and closure should, however, be considered for hypertrophic scars, if the condition of closure can be improved. This is especially pertinent for wounds that originally healed by secondary intention or that are complicated by infection. Keloids typically develop several months after the injury and rarely, if ever, subside. Hypertrophic scars usually develop within the first month after wounding and often subside gradually.

147 Which of the following statement(s) is/are true concerning the vascular response to injury?
a. Vasoconstriction is an early event in the response to injury
b. Vasodilatation is a detrimental response to injury with normal body processes working to avoid this process
c. Vascular permeability is maintained to prevent further cellular injury
d. Histamine, prostaglandin E2 (PGE2) and prostacyclin (PGI2) are important mediators of local vasoconstriction
Answer: a

After wounding, there is transient vasoconstriction mediated by catecholamines, thromboxane, and prostaglandin F2 (PGF2a). This period of vasoconstriction lasts for only five to ten minutes. Once a clot has been formed and active bleeding has stopped, vasodilatation occurs in an around the wound. Vasodilatation increases local blood flow to the wounded area, supplying the cells and substrate necessary for further wound repair. The vascular endothelial cells also deform, increasing vascular permeability. The vasodilatation and increased endothelial permeability is mediated by histamine, PGE2, and prostacyclin as well as growth factor VEGF (vascular endothelial cell growth factor). These vasodilatory substances are released by injured endothelial cells and mast cells and enhance the egress of cells and substrate into the wound and tissue.

148 Which of the following statement(s) concerning laboratory studies used in monitoring a patient with intravenous heparinization is/are correct?
a. The platelet count should be followed because of the risk of heparin-associated thrombocytopenia
b. The prothrombin time should be observed if prolonged treatment is necessary
c. The activated partial thromboplastin time (aPTT) should be maintained at approximately 1.5 times normal
d. The serum creatinine should be measured daily to allow adjustments in dose based on renal function
Answer: a, c

In monitoring the effect of heparin, an activated partial thromboplastin time (aPTT) of 1.5 control or a thrombin clotting time (TCT) of 2 times control reflects adequate anticoagulation. The prothrombin time remains normal. Heparin-associated thrombocytopenia from an immune mechanism is a potential complication of the use of this anticoagulant. Therefore any patient undergoing heparin therapy should have a platelet count determined every other day after the fourth day of therapy or earlier if he or she is known to have been exposed to heparin in the past. Heparin is not excreted through the kidneys or the liver but is cleared through the reticuloendothelial system. Therefore the dose of heparin need not be adjusted in cases of liver or renal dysfunction.

149 Which of the following statement(s) is/are true concerning heparin-associated thrombocytopenia?
a. Heparin-associated thrombocytopenia occurs only in the face of over anticoagulation with heparin
b. Severe thrombocytopenia (platelet count less than 100,000) is seen in less than 10% of patients treated with heparin
c. Heparin-associated thrombocytopenia is due to the aggregation of platelets and may result in thrombosis or embolic episodes
d. Heparin-associated thrombocytopenia may be seen within hours of initiation of heparin therapy
Answer: b, c

Heparin-associated thrombocytopenia occurs in 0.6% to 30% of patients who receive heparin, although severe thrombocytopenia (platelet counts less than 100,000) is seen in fewer than 10% of patients treated with heparin. It is caused by a plasma factor, most likely a heparin-dependent platelet antibody, that causes aggregation of platelets when exposed to heparin. Activation of platelets in this setting results in thrombocytopenia, thrombosis and embolic episodes, which can lead to death. Both bovine and porcine heparin have been associated with this syndrome, which usually begins 5 to 15 days after initiating heparin therapy. Even trivial exposure with heparin such as coating on pulmonary artery catheters or low rate infusion into arterial catheters may cause this syndrome.

150 Antithrombin III deficiency is a commonly observed hypercoaguable state. Which of the following statement(s) is/are true concerning this condition?
a. A patient with this deficiency usually presents with thrombosis while on heparin or exhibits an inability to become adequately anticoagulated with heparin
b. This deficiency may be either congenital or acquired
c. Thrombotic episodes are related to predisposing events such as operations, childbirth, and infections
d. Treatment involves acutely the administration of fresh frozen plasma followed by long-term treatment with Coumadin
Answer: a, b, c, d

BASIC SURGICAL BACKGROUND Objective type Questions with Answers

151 Mini-dose heparin has been shown to be useful in the prophylaxis of postoperative venous thrombosis. Mechanism(s) by which low-dose heparin is/are thought to protect against venous thrombosis include:
a. Enhancement of antithrombin III activity
b. A decrease in thrombin availability
c. Inhibition of platelet aggregation and subsequent platelet release action
d. A mild prolongation of activated partial thromboplastin time
Answer: a, b, c

Low-dose heparin is thought to protect against venous thrombosis through three different mechanisms. First, antithrombin III activity with its inhibition of activated Factor X is enhanced by only trace amounts of heparin; second, there is a decrease in thrombin availability that prevents its activation and thus its fibrin-stabilizing effect; and third, small doses of heparin may inhibit the second wave of platelet aggregation and subsequent platelet release reaction. The standard doses of heparin administered (5000 units bid) does not affect aPTT.

152 Tests of coagulation are used to monitor anticoagulation treatment and detect intrinsic abnormalities in coagulation. Which of the following statement(s) is/are true concerning coagulation tests?
a. Prothrombin time (PT) measures both the intrinsic and extrinsic clotting pathways and fibrinogen
b. Activated partial thromboplastin time (aPTT) can be used to monitor both oral anticoagulation with Warfarin and intravenous anticoagulation with heparin
c. Thrombin clotting time (TCT) is a measurement of the time it takes for exogenously administered thrombin to turn plasma fibrinogen into fibrin clot
d. Whole blood activated clotting time (ACT) is a measurement of the ability of whole blood to clot and is used to monitor heparin levels intraoperatively during cardiovascular and peripheral vascular operations
Answer: a, c, d

Coagulation tests include prothrombin time (PT), which measures the intrinsic and extrinsic pathways of fibrinogen production and is the most common method for measuring a level of oral anticoagulant therapy. The activated partial thromboplastin time (aPTT) identifies the abnormalities of the contact and intrinsic phases of coagulation. Values of aPTT have variably been shown to correlate with heparin dosages and serum heparin levels and are therefore most commonly used in monitoring heparin therapy. It is of no value in long-term management of patients on oral Warfarin therapy. Thrombin clotting time (TCT) is the measure of the time it takes for exogenously administered thrombin to turn plasma fibrinogen into fibrin clot. It is extremely sensitive to levels of heparin and is an excellent measure of measuring the level of heparin-induced anticoagulation. The beauty of the TCT is that it is not specific for any disease condition; thus it may be used to differentiate factor deficiencies from the presence of heparin, or to separate lupus anticoagulant from abnormalities in fibrinogen levels. The whole blood activated clotting time (ACT) is a measurement of the ability of whole blood to clot, and as such, is an available technique for monitoring heparin levels intraoperatively. The ACT responds in a linear fashion to increasing heparin dosage and correlates well with the observed clinical anticoagulation. Adequate anticoagulation for extracorporeal circulation is defined as an ACT of 480 seconds or more while for peripheral vascular applications, values of 250 seconds or greater are considered appropriate.

153 Thrombolytic therapy has become a useful adjunct in the management of peripheral arterial occlusion. In this setting, direct intraarterial administration rather than intravenous has been advocated to decrease the risk of systemic bleeding. Which of the following true statement(s) concerning the use of thrombolytic agents for arterial occlusion is/are true?
a. A standard technique involves infusing high-dose urokinase, 4000 units per minute for 1–2 hours, directly into the clot by a catheter embedded in the thrombus
b. If progress is made, further fibrinolytic therapy is given at 1000 to 2000 units per minute until clot lysis has occurred
c. The usual infusion time by the above-stated technique is usually in excess of 24 hours
d. Successful clot lysis occurs more frequently in arterial graft occlusions than native arterial occlusions
e. The use of intraoperative thrombolytic therapy may be indicated for situations where complete clot evacuation cannot be accomplished surgically
Answer: a, b, e

The most popular method for intraarterial thrombolytic therapy for arterial occlusion involves passing a guidewire through the thrombus with arteriographic guidance and then infusing high-dose urokinase, 4000 units per minute for 1–2 hours, directly into the clot. If progress is made, further fibrinolytic therapy is given at 1000 to 2000 units per minute for a 6–12 hour period or until clot lysis has occurred. Using this technique, mean infusion time in a recent study was found to be 18 hours and the incidence of bleeding complications was significantly lessened. Selective intraarterial infusion of urokinase was associated with complete clot resolution in 77% of native arterial occlusions versus only 41% with arterial graft occlusion. After thrombolytic therapy has reopened an occluded vessel or graft, radiologic or surgical correction of the lesion responsible for the thrombosis in the first place must be addressed for any hope of long-term success. The use of intraoperative thrombolytic therapy is advocated in those situations where complete clot resolution cannot be accomplished (such as following balloon embolectomy for acute arterial occlusion) or when distal vasculative is occluded and precludes appropriate inflow patency.

155 Fibrinolytic therapy is based on activation of plasminogen, the inactive proteolytic enzyme of plasma that binds to fibrin during the formation of thrombosis. Activation of plasminogen to plasmin results in selective thrombolysis at the fibrin clot surface. Which of the following statement(s) is/are true concerning agents used in thrombolytic therapy?
a. Streptokinase is a bacterial protein which is antigenic in humans, resulting in allergic reactions in up to l5% of cases
b. Tissue plasminogen activator acts directly on plasmin without an intermediate plasmin complex
c. The half-life of urokinase, streptokinase, and TPA all exceed 30 minutes
d. Streptokinase is significantly cheaper than urokinase or TPA
Answer: a, b, d

157 External pneumatic compression has been advocated for the prevention of deep venous thrombosis during operative procedures. Which of the following statement(s) concerning the use of external pneumatic compression devices is/are true?
a. Intermittent pneumatic compression is as effective as low-dose heparin in prevention of venous thrombosis
b. These devices function by compressing the lower extremities therefore augmenting venous return
c. Pneumatic compression devices may also exhibit their antithrombotic effect through stimulating local and systemic fibrinolysis
d. The length of time that intermittent pneumatic compression should be used includes through the operation and for at least several days in the postoperative period
Answer: b, c, d

In many well-controlled studies of venous prophylaxis, intermittent pneumatic compression has been found to be as effective as low-dose heparin therapy. In addition to augmentation of venous return with these devices, local and systemic fibrinolysis appears to be stimulated. Fibrinolytic activities are usually reduced for a 7–10 day period after an operation. Studies have demonstrated that the pneumatic-compression devices may exhibit their antithrombotic effect through prevention of this fibrinolytic shutdown even when applied to the upper extremity. The length of time that intermittent pneumatic compression should be used has not been adequately determined but most data suggest that devices should be used through the operation and for at least five days in the face or prolonged immobilization.

Warfarin interferes with the vitamin K dependent clotting factors II, VII, IX and X, protein C, and protein S. An important complication of warfarin is skin necrosis with patients both with and without protein C deficiency. This syndrome usually involves full thickness skin slough over fatty areas such as the breasts and buttocks. Warfarin therapy should be monitored using the one stage prothrombin time (PT). The PT should be kept at 1.3 to 1.4 control for effective anticoagulation. At higher levels, there is a five-fold increase in the frequency of bleeding complications. Two major complications of Warfarin therapy include recurrent thrombosis and bleeding. It is recommended that Warfarin be continued four months after an initial episode of deep venous thrombosis. Between ten weeks and four to six months after deep vein thrombosis, there is a recurrent thrombosis rate of 8.3 episodes per 1000 patient months. Between four months and three years, recurrences fall to four episodes per 1000 patient-months. At four months, the risks of bleeding complications matches and exceeds the benefit from anticoagulant therapy and thus is the basis for discontinuing warfarin administration at this time.

159 Which of the following statement(s) is/are true concerning the management of a patient with hemophilia A undergoing an elective surgical operation?
a. Concentrates of factor VIII should be given several days prior to elective surgery
b. The half-life of factor VIII concentrates is less than 24 hours
c. A dose of 40–50 IU/kg of factor VIII concentrate should be given prior to the planned surgical procedure
d. Factor VIII concentration administration should be given for the first 24 hours after surgery but may then be stopped if no abnormal bleeding has been observed
e. A new recombinant preparation of factor VIII offers the advantage of being virus-free
Answer: b, c, e

Although the half-life of factor VIII is 2.9 days in normal individuals, the half-life of factor VIII concentrates is 9 to l8 hours. Levels of 80% to 100% of normal should be obtained for surgical bleeding or life-threatening hemorrhage. A dose of 40 to 50 IU/kg of factor VIII should be given with half of this dose then administered every twelve hours. After surgery, transfusion of factor VIII concentrates should be continued for at least ten days. Unfortunately, past use of concentrates of factor VIII obtained from donors has led to a high incidence of HIV infection in the hemophilia population. A new recombinant preparation of factor VIII offers the advantage of being virus-free.

160 Transfusions of blood products can be associated with a number of complications including immediate and delayed hemolytic reactions; nonhemolytic reactions; infectious disease transmission; and complications of massive transfusions. Which of the following statements are true concerning complications of blood transfusions?
a. Immediate hemolytic transfusion reactions are caused by major ABO blood group incompatibility
b. Nonhemolytic transfusion reactions are usually due to RH incompatibility and are therefore more common in women of childbearing age
c. The most common complication of massive blood transfusion is dilutional thrombocytopenia
d. Routine impaired calcium supplementation is necessary during most massive transfusion episodes
Answer: a, c

Immediate hemolytic reactions are usually caused by blood group ABO incompatibility although they may be caused by antigens of other blood group systems on the transfused red blood cells. The clinical manifestations revolve around the antigen on the red blood cell stroma and the antibody in the patient’s serum, and include production of bradykinin, compliment activation, release of vasoactive agents from platelets, and initiation of systemic clotting. Chills and fevers, chest pain and lumbar pain, tachycardia and hypotension in the conscious patient, and often diffuse bleeding in the anesthetized, unconscious patient constitute this syndrome. Although reaction occurs immediately, death related to the syndrome is uncommon, unless associated with a transfusion of more than 100 ml of blood. Death usually occurs from acute renal failure or hemorrhage due to DIC. Nonhemolytic reactions occur with the frequency of 1 to 2% of all transfusions and consist primarily of chills and fevers during the transfusion or in the first 2 to 3 hours after the transfusion is complete. Mechanism of these reactions includes the presence of antibodies to white blood cell antigens in the transfused blood, especially in the multitransfused or multiparous patient. Massive transfusion complications relate to the rate and volume of blood transfused. The most common complication is dilutional thrombocytopenia. Factor deficiency of the labile factors V and VIII rarely is of sufficient magnitude to result in problems with hemostasis. For hypocalcemia to occur with massive transfusion, citrated blood must be administered, one unit every five minutes. Routine empiric calcium supplementation is unnecessary during most massive transfusion episodes. Conversely, hypothermia is clearly a problem, especially when associated with massive transfusion during complex intraoperative procedures such as thoracoabdominal aneurysm resection.

161 A 67-year-old male with advanced cholangiocarcinoma develops gram-negative sepsis. Excessive bleeding is noted around vascular catheters and from needle puncture sites. The diagnosis of disseminated intervascular coagulation (DIC) is considered. Which of the following laboratory test(s) is/are indicative of DIC?
a. Decreased platelet count
b. Decreased fibrinogen level
c. Normal prothrombin time
d. Elevated fibrin split products
Answer: a, b, d

Disseminated intravascular coagulation (DIC) is the primary form of acute thrombosis. Causes of this syndrome include abruptio placenta, gram-positive and gram-negative sepsis, endotoxemia, malignant tumors, pelvic operations, certain snake bites, hematologic malignancies, and hepatic failure. Blood coagulation is activated by the release of tissue factor into the circulation, which activates factor VII of the extrinsic pathway to VIIa, leading to massive thrombin production and fibrin generation. This in turn activates the fibrinolytic system, leading to bleeding in the later stages of the syndrome due to consumption of coagulation factors, depletion of fibrinogen, and unchecked plasma activities. Laboratory values in DIC usually include a decline in the platelet count and fibrinogen level, along with an elevation of fibrin split products.

162 Which of the following substances, not normally present in the circulation, trigger the initiating events in the hemostatic process?
a. Thrombin
b. Platelet factor 3
c. Tissue factor
d. Collagen
Answer: c, d

The initiating agents for hemostasis involve two substances that are not normally present in the circulation—collagen and tissue factor. Tissue factor is released from injured cells, beginning the activation of the extrinsic pathway of coagulation, while disruption of the protective endothelial barrier of blood vessels exposes the underlying collagen to the activation of platelets. In the bloodstream, tissue factor complexes with factor VII which then activates factor X to factor Xa. At the same time, activated platelets change from their discoid shape with their procoagulant phospholipid (termed platelet factor 3) buried on the inner side of the surface membrane to a spreading shape to allow for the externalization of platelet factor 3 activity. Activated factor X, activated factor V, ionized calcium and factor II (prothrombin) then assemble on the platelet phospholipid surface to form the so-called prothrombinase complex which catalyzes the formation of thrombin.

163 Bleeding complications are frequently associated with fibrinolytic therapy. Which of the following statement(s) concerning complications of fibrinolytic therapy is/are true?
a. Careful monitoring of prothrombin time and aPTT time are necessary to avoid bleeding complications
b. A level of serum fibrinogen less than 100 mg/dl is associated with an increased risk of bleeding
c. Recent (less than 10 days) major surgery is a contraindication to systemic but not regional fibrinolytic therapy
d. A patient with a cerebrovascular event occurring less than two months ago can be treated with fibrinolytic therapy if head CT scan is normal
Answer: b

Fibrinolytic therapy induces a hemostatic defect through a combination of factors. Hypofibrinogenemia and fibrin degradation products inhibit fibrin polymerization and, in combination with a decrease in the clotting factors V and VIII, prolong the ability of blood to clot. However, coagulation tests in general do not correlate well with bleeding complications. A level of fibrinogen less than 100 mg/dl is associated with an increased risk of bleeding. Absolute contraindications to thrombolytic therapy include active internal bleeding, recent (less than 2 months) cerebral vascular accident, and documented left heart thrombosis. Recent (less than 10 days) major surgery, obstetric delivery, organ biopsy, or major trauma is considered a major relative contraindication to either regional or systemic thrombolytic therapy.

164 Which of the following statement(s) is/are true concerning the results of a National Institute of Health Consensus Conference on venous thrombosis and low-dose heparin prophylaxis?
a. The odds of developing deep venous thrombosis with low-dose heparin prophylaxis decreases by 67%
b. The risk of pulmonary embolism is decreased by almost 50%
c. There is no increase in mortality from other causes found in patients treated with low-dose heparin
d. There was no difference in the incidence of bleeding complications
Answer: a, b, c

In a metaanalysis of 70 randomized trials in 16,000 patients comparing low-dose heparin prophylaxis with standard therapy, the odds of developing deep venous thrombosis with low-dose heparin prophylaxis decreased 67%, whereas for pulmonary embolism (both fatal and non-fatal), the odds decreased by 47%. For fatal pulmonary embolism, the odds reduction was even greater (64%). No increase in mortality from other causes was found in those patients treated with low-dose heparin. Bleeding complications were more frequent in the heparin-treated patients, with no difference between 5000 units twice daily and 5000 units three times daily. Similarly, the effectiveness of prophylaxis was not influenced by either two or three times daily dosage.

165 Laboratory monitoring of coagulation and anticoagulation includes testing of platelet function. Which of the following statements is/are true concerning tests of platelet function?
a. A platelet count of 50,000/µL or more usually ensures hemostasis
b. Bleeding time assays assessibility of platelets to perform hemostatic plugs and is determined from a sample of blood drawn in an EDTA coated test tube
c. Aspirin therapy can be associated with a bleeding time in the range of 8–15 minutes
d. Tests of platelet aggregation should be part of the standard preoperative evaluation of patients using aspirin
Answer: a, c

166 As thrombin generation proceeds, the body has natural anticoagulant systems opposing further thrombus formation. Natural anticoagulants include:
a. Tissue plasminogen activator (TPA)
b. Antithrombin III
c. Activated protein C
d. Heparin cofactor II
Answer: b, c, d

Just as thrombin generation is the key to coagulation, antithrombin III is the most central anticoagulant proteins. This glycoprotein binds to thrombin, preventing its removal of fibrinoprotein A and B from fibrinogen, prevents the activation of factor V and VIII and the activation and aggregation of platelets. The second line of defense is the activated protein C, which inactivates factors Va and VIIIa. This inactivation reduces the ability of the prothrombinase complex to accelerate the rate of thrombin formation. A third natural anticoagulant is heparin cofactor II. Its concentration in plasma is estimated to be some four-fold lower than antithrombin III, and its action is primarily implicated in the regulation of thrombin formation in extravascular tissues. Tissue plasminogen activator (TPA) is a natural catalyst for the activation of plasminogen to plasmin, the main fibrinolytic enzyme in the body. Therefore, TPA is part of the fibrinolytic system rather than a natural anticoagulant.

167 Infectious disease transmission during blood transfusions is of clinical significance to surgeons and of major importance to patients contemplating surgery potentially associated with the need for blood administration. Which of the following statement(s) is/are true concerning the transmission of infectious disease during blood transfusions?
a. Post-transfusion hepatitis is usually due to hepatitis B
b. Hepatitis and HIV transmission is greatest with the administration of pooled plasma products such as serum albumin
c. The most important cause of post-transfusion disease in immunosuppressed patients is CMV infection
d. The risk of HIV transmission in blood transfusions is significantly less than the risk of hepatitis transmission
Answer: c, d

The most common infectious diseases transmitted during blood transfusions include viral hepatitis, CMV, and HIV infection. Post-transfusion hepatitis in 90% of cases consists of non-A, non-B hepatitis known as hepatitis C. All blood products except for immune serum globulin and albumin can carry and transmit this form of hepatitis. Because heat treatment eliminates the risk of viral transmission, products from pooled plasma that are heat treated such as albumin are not at risk for HIV or hepatitis transmission. CMV transmission exists in three forms—primary, reinfection, and reactivation. Primary exposure results in an IgM response to the virus. Reactivation is most commonly related to pregnancy, transplantation, and immunosuppression, and is the most important cause of post-transfusion disease accompanying immunosuppression of patients. Although the risk of the public concern for transmission of HIV disease associated with blood transfusions has significantly outweighed other infectious disease transmission, the risks of HIV transmission is markedly less than that of hepatitis.

168 There are a number of hypercoaguable states which can be associated with arterial or venous thrombosis and embolic phenomenon. These include:
a. Heparin-associated thrombocytopenia
b. Antithrombin III deficiency
c. Von Willebrand disease
d. Vitamin C deficiency
Answer: a, b

A number of hypercoaguable states are present. These include heparin-associated thrombocytopenia in which a heparin-dependent platelet antibody causes aggregation of platelets when the patient is exposed to heparin. Activation of platelets in this setting results in thrombocytopenia, thrombosis, and embolic episodes. Antithrombin III deficiency accounts for about 2% of venous thrombotic events and has been described in pulmonary embolism, mesenteric venous thrombosis, lower extremity venous thrombosis, arterial thrombosis, and dialysis fistula failure. Von Willebrand’s disease is a hereditary complex coagulation factor deficiency which is manifested by a reduction of factor VIII activity, and the Von Willebrand factor which is an adhesive protein that mediates platelet adhesion to collagen. Severe vitamin C deficiency results in a disorder in soft tissue increasing vascular permeability and fragility resulting in the potential for bleeding disorders.

169 Cytokines with clearly defined actions in acute inflammation and early tissue injury include which of the following?
a. Cysteine-X-Cysteine (C-X-C) chemokines
b. Tumor Necrosis Factor (TNFa)
c. Transforming Growth Factor-b (TGF-b)
d. Interleukin-6 (IL-6)
e. Platelet Derived Growth Factor (PDGF)
Answer: a, b, c, d, e

Polypeptide mediators, such as TNFa and IL-1, are considered “early response” cytokines and are actively involved in the initiation of the cascade of events which precipitate acute inflammation. In addition to being important triggers for the induction of other cytokines important inflammatory network, TNFa and IL-1 appear to be key mediators in promoting the adherence of inflammatory cells to the endothelium. IL-1 is a complex, multifunctional molecule that shares many overlapping biological properties with TNFa. In addition, both IL-1 and TNFa potentiate the effects of one another. The most important function of IL-6 appears to be the regulation of the hepatic acute phase response. Following injury, a number of physiologic changes develop within several hours. IL-6 is one of the primary stimuli for the production of acute phase proteins from the liver. Endotoxin, IL-1, TNFa and PDGF are capable of causing significant induction of IL-6 synthesis.
Over the last decade, at least 12 different C-X-C chemokines have been identified. These include IL-8, one of the most potent mediators of chemotaxis known. TNFa and IL-1 are key molecules for the induction of IL-8, which in turn is important for the induction of neutrophil recruitment and activation.
Similar properties are apparent for other members of this chemokine family.
Platelet activation and degranulation occur during coagulation following injury, leading to the deposition of a number of cytokines into the provisional matrix. These cytokines include transforming growth factor-a, (TGFa), transforming growth factor b (TGF-b), platelet-derived growth factor (PDGF), and neutrophil activating peptide-2 (NAP-2). These cytokines are either important growth factors or chemotaxis for leukocytes, endothelial cells, fibroblasts, and keratinocytes which are key components in the process of tissue repair. Thus, coagulation and platelet activation provide the initial foundation for subsequent cellular recruitment.

170 Which of the following statements regarding transforming growth factor b (TGF-b) are true?
a. TGF-b expression is autoregulated
b. TGF-b enhances collagen synthesis
c. TGF-b inhibits extracellular matrix production
d. TGF-b may inhibit or promote cellular proliferation
Answer: a, b, d

TGF-b appears to be one of the key cytokines in control of tissue repair. TGF-b is strongly chemotactic for neutrophils, T cells, monocytes, and fibroblasts. TGF-b activates inflammatory cells to elaborate fibroblast growth factor, TNFa, IL-1 and increase their synthesis of extracellular matrix proteins. TGF-b also induces both the infiltrating cells and resident cells to produce more TGF-b. This auto-induction amplifies its biological effects at the site of injury and may play an important role in the development of chronic fibrosis in a variety of pathologic states. TGF-b enhances collagen synthesis as well. Lastly, TGF-b may function as a mitogen or growth inhibitor for a wide variety of cell types, including selected cell types of mesenchymal origin. Whether TGF-b stimulates or inhibits proliferation depends on the presence of other growth factors, the concentration of TGF-b, and the cell density. Thus, at low doses, TGF-b stimulates the proliferation of densely plated human marrow fibroblasts, but is inhibitory at high concentrations.

171 Leukocyte activation and adhesion to vascular endothelial cells is a critical step in the inflammatory process. This process is regulated by which of the following molecules?
a. The selectins
b. The b5 integrins
c. The immunoglobulin supergene family
d. Nitric oxide
e. IL-8
Answer: a, c, d, e

The temporal events that initiate and propagate neutrophil recruitment and inflammation include endothelial cell activation and expression of endothelial-derived neutrophil adhesion molecules, neutrophil-endothelial cell adherence, and neutrophil transendothelial migration via established neutrophil chemotactic gradients. There are three major families of adhesion molecules which are expressed on the surface of leukocytes and endothelial cells and are important for leukocyte-endothelial cell interactions. These include the immunoglobin supergene family (ICAM-1, VCAM-1, and PECAM-1), the selectins (E-selectin, P-selectin and L-selectin), and the integrins. The leukocyte b2 integrin adhesion molecule family consists of three members with heterodimeric glycoproteins displayed as a variable alpha and a constant beta chain. Nitric oxide regulates the adhesion process both by direct influence on leukocyte binding as well as by regulation of regional blood flow. IL-8 is one of the most potent mediators of chemotaxis in the C-X-C chemokine family. It serves an important role in neutrophil recruitment and activation, and the continued propagation of the inflammatory response.

172 A 65-year old patient has colon carcinoma metastatic to the liver and lungs. He has had a weight loss of 10 kg. Cytokine-dependent tumor cachexia is attributable to which of the following?
a. Increased glucose uptake and increased glycogen breakdown occur in this circumstance.
b. Suppressed activity of lipoprotein lipase results from TNFa
c. TNFa stimulates lipolysis
d. The differentiation process of pre-adipocytes is impaired
e. Partial reversal of differentiated adipocytes to pre-adipocyte morphology and gene expression occurs
Answer: a, b, c, d, e

Tumor cachexia appears to be mediated by TNFa. Lipopolysaccharide (LPS), as well as other cytokines, activate a variety of inflammatory cells, most importantly macrophages, to produce TNFa. Both the chronic administration of TNFa to rats and implantation of tumors secreting TNFa in mice induce a syndrome of cachexia. In vitro, higher TNFa concentrations alter glucose metabolism in cultured myotubules by increasing glucose uptake and glycogen breakdown. It has also been demonstrated that purified TNFa suppresses lipoprotein lipase activity and stimulates lipolysis in cultured adipocytes. Further, TNFa not only inhibits the differentiation process of preadipocytes, but partially reverses differentiated adipocytes to a preadipocyte morphology and pattern of gene expression. All of these metabolic effects at least partially explain the chronic syndromes of anorexia, weight loss, and cachexia that are associated with both chronic infection and malignancy.

173 Which of the following statements regarding fibroblasts and their function in wound healing are true?
a. IL-1 has both inhibitory and promotional effects on fibroblast growth
b. TNFa stimulates fibroblast collagen synthesis
c. IL-1 and TNFa have opposite effects on the healing of bone
d. In human clinical trials, EGF (epithelial growth factor) has been demonstrated to accelerate epidermal regeneration in cutaneous wounds
Answer: a, d

IL-1 appears to be important in the process of normal wound repair. IL-1 has been shown to stimulate skin fibroblast and keratinocyte growth, as well as fibroblast collagen synthesis and keratinocyte chemotaxis. IL-1 also promotes increased transcription of the matrix degradative enzymes collagenase and stromelysin. These are important and potent tissue degrading proteinases. Other studies have demonstrated that IL-1 inhibits fibroblast growth and matrix synthesis, and stimulates collagenase production. These actions are at least partly due to the ability of IL-1 to upregulate prostaglandin E2 production which results in the down regulation of matrix synthesis. IL-1 has both promoting and inhibiting effects on fibroblast collagen synthesis, therefore, the overall activity in this area is somewhat unclear in comparison to other well-defined fibroblast growth-promoting cytokines. TNFa inhibits fibroblast collagen synthesis, however it also has potent mitogenic effects. The mitogenic response correlates well with an increased stimulation of tyrosine phosphorylation. Both IL-1 and TNFa have similar effects upon bone. Both stimulate cartilage resorption, the release of proteoglycans from cartilage by limited proteolytic degradation, and both inhibit proteoglycan synthesis. Recent studies have also demonstrated that TNFa inhibits fracture healing in experimental animals. This is due to the inhibition of cartilage formation and new bone synthesis, and the inhibition of mesenchymal cell differentiation into chondroblasts. The family of epithelial growth factor (EGF)-like molecules induce mitogenesis and play a role in wound healing. In human clinical trials, EGF has been demonstrated to accelerate epidermal regeneration in cutaneous wounds. In vitro data show that recombinant EGF enhances keratinocyte migration. EGF is also a potent chemoattractant for granulation tissue fibroblasts.

174 Neutrophil chemotaxis is a fundamental aspect of inflammatory injury in conditions such as the Adult Respiratory Distress Syndrome (ARDS). Neutrophil chemotaxis is directly attributable to which of the following molecules?
a. C5a
b. TNFa
c. LPS
d. IL-1
e. ENA-78 (Epithelial Neutrophil Activating Protein)
Answer: a, e

There is a large collection of peptide, polypeptide and lipid mediators which have chemotactic properties. Although TNF a, IL-1 and LPS were initially reported to have direct neutrophil chemotactic activity, recent studies have demonstrated that these molecules are not directly chemotactic for neutrophils. This finding suggests that cytokine networks may be operative in vivo and depend on the initial expression of early response cytokines. This initial interaction is followed by the generation of more distal inflammatory mediators that directly influence neutrophil chemotaxis and activation. There is a particularly important group of novel chemotactic cytokines which share significant homology with the presence of four conserved cysteine amino acid residues. These cytokines in their monomeric forms are all less than 10 kD, are characteristically basic heparin-binding proteins, have specific neutrophil chemotactic activity and display four highly conserved cysteine amino acid residues, with the first two cysteines separated by one non-conserved amino acid residue. Because of their chemotactic properties and the presence of C-X-C cysteine motif, these have been designated the C-X-C chemokine family. Twelve different chemokines have been identified in the last decade. These include IL-8, epithelial neutrophil activating protein (ENA-78), and others. Among the other endogenous chemoattractants are several complement-derived peptides. Perhaps, the most potent of these is the short-lived C5a peptide.

175 Which of the following statements regarding angiogenesis are true?
a. Angiogenesis is a seminal biologic event with clinical relevance limited to its effect upon tumor growth
b. C-X-C chemokines regulate angiogenesis
c. PF-4 has angiogenic properties
d. Sites of atherosclerosis demonstrate chronic angiogenic activity
Answer: b, d

An important component of tissue repair and wound healing is the process of angiogenesis. This normal, physiologic process is a local, transient event which is regulated strictly. A biological imbalance in the production of angiogenic and angiostatic factors contributes to the pathogenesis of several angiogenesis-dependent disorders. These include both malignant and nonmalignant disorders such as rheumatoid arthritis, scleroderma, psoriasis, atherosclerosis, and idiopathic pulmonary fibrosis. Persistent neovascularization in these benign disorders is a prerequisite for the perpetuation of fibroproliferation. IL-8 and potentially other C-X-C chemokines are involved with the angiogenesis process. IL-8 is a potent angiogenic factor. In contrast, another member of the C-X-C chemokine family, PF-4 has angiostatic properties. This suggests that the C-X-C chemokines may function as either angiostatic or angiogenic factors, and the biologic balance that is maintained between these factors may govern overall angiogenic potential in a variety of physiological and pathophysiological states.

176 Which of the following statements regarding IL-1 are correct?
a. While IL-1 and TNFa share many biologic effects, IL-1 appears to be more potent
b. IL-1 expression is in part autoregulated
c. IL-1 inhibits prostaglandin production
d. The ability of IL-1 to upregulate endothelial cell-neutrophil adhesion molecules is relatively limited
Answer: b

IL-1 and TNFa share many biologic properties. In addition, each potentiates the effects of the other one when given concurrently. Overall, IL-1 alone probably has weaker effects than TNFa with respect to the induction of shock; its role is likely to be important with respect to its marked potentiating abilities as it relates to TNFa. IL-1 expression is regulated by a host of factors including IL-2, granulocyte macrophage colony stimulating factor (GM-CSF), transforming growth factor b (TGF-b), TNFa, all of the interferons, and IL-1 itself. Other endogenous stimuli for IL-1 production include antigen-antibody complex, the Fc region of IgG, and C5a; other nonspecific exogenous stimuli include silica particles and UV irradiation.
One of the key proinflammatory features of IL-1-induced inflammation is the stimulation of arachadonic acid metabolism. IL-1 stimulates the release of pituitary stress hormones and increases the synthesis of collagenases, resulting in the destruction of cartilage, bone and other collagen-rich structures. IL-1 stimulates prostaglandin production.
One of the most important properties of IL-1 involves its interaction with the vascular endothelium. This includes the adherence of neutrophils, basophils, eosinophils, monocytes, and lymphocytes to the vascular endothelium via interaction between adhesion molecules on leukocytes and adhesion-receptor complex on the endothelial cells. By inducing the expression of ICAM-1, E-selectin, and VCAM-1 on endothelial cells, IL-1 provides a key step in the extravasation of leukocytes to sites of local inflammation and injury.

177 Which of the following statements regarding TNFa are true?
a. TNFa has a marked procoagulant effect
b. Passive immunization of patients with neutralizing antibodies to TNFa improves survival from multi-organ system failure
c. TNFa upregulates E-selectin expression
d. The most potent known stimulus for TNFa production and release is IL-1
Answer: a, c

TNFa has a marked procoagulant effect on endothelial cells, precipitating intravascular thrombosis. TNFa causes endothelial cells to release procoagulant activity (tissue factor), platelet activating factor, and von Willebrand factor, all of which favor thrombosis. TNFa also down regulates the expression of thrombomodulin, which has the potential to block the assembly of protein C and protein S complexes, further decreasing the anticoagulant properties of the endothelial cell surfaces.
Administration of recombinant TNFa to experimental animals produces a clinical syndrome similar to that seen in septic shock and multi-organ system failure in humans. Passive immunization of animals with neutralizing antibodies against TNFa, prior to the infusion of TNFa or endotoxin, has been shown to prevent the development of this syndrome. No such evidence exists in human patients.
TNFa upregulates a variety of leukocytic adhesion molecules including ICAM-1, PECAM-1, VCAM-1, E-selectin and P-selectin. A variety of exogenous and endogenous factors (including IL-1) are capable of inducing cells to produce TNFa, however the most potent stimulus for TNFa production and release is endotoxin.

178 Which of the following belong to the family of C-X-C chemokines?
a. IL-8
b. IL-10
c. Growth Related Oncogene-a
d. Leukotreine B4
e. b Thromboglobulin
Answer: a, c, e

A particularly important group of novel chemotactic cytokines has been elucidated over the last decade. Twelve are known and are listed below.
C-X-C Chemokines
Connective Tissue Activating Protein III
b-Thromboglobulin
Growth Related Oncogene-a
Growth Related Oncogene-b
Related Oncogene-g
Interleukin-8
Epithelial Neutrophil Activating Protein
Granulocyte Chemotactic Protein-2
Platelet Factor-4
g-Interferon-inducible Protein
Monokine-induced by g-Interferon
Each has unique biologic functions. There appear to be important in vivo cytokine networks involving these molecules which regulate chemotaxis, and other fundamental aspects of inflammation.

179 Which of the following statements regarding the complement system are true?
a. Complement activation yields products which are directly cytotoxic as well as products which act indirectly via activated leukocytes
b. Complement products referred to as anaphylatoxins include C1, C3a, C4a, and C5a
c. The principal role of C5a is in bacterial opsonization
d. The alternative and classical pathways converge proximal to generating the membrane attack complex (C5b-9)
Answer: a, d

The complement system is composed of two different but linked sequences, the classic and alternative pathways. The pathways involve serum proteins that act to amplify the inflammatory-immune response as well as to directly mediate tissue injury. Complement activation by either pathway has been associated with a cascade of events, some of which are mediated directly at a physiologic level by complement products and some of which occur indirectly via activated leukocytes. The direct physiologic effects mediated by C3a and C5a, and to a lesser extent C4a, include increased vascular permeability and contraction of smooth muscle. These are key elements of anaphylaxis. C1 is not an anaphylatoxin as it is the initial complement component which binds to antigen-antibody complex to initiate classical pathway activation. C5a acts principally to alter the behavioral characteristics of leukocytes. Effects include enhanced adherence, enhanced chemotactic activity, release of proteinases, and production of toxic metabolites of oxygen. C3, on the other hand, plays a key role in bacterial opsonization, resulting in enhanced phagocytosis of invading microorganisms. The alternative and classical complement pathways converge at the C5 level proximal to generating the membrane attack complex (C5b-9) (Figure 6-3).

180 Which of the following statements regarding neutrophils are true?
a. The neutrophil undergoes final maturation after release into the circulation
b. Patients with chronic granulomatous disease have a defective neutrophil H-oxidase system
c. Neutrophil killing of bacteria is achieved by oxidants, proteinases and cationic proteins
d. The normal human neutrophil circulates in the blood for 7–10 days
Answer: b, c

The neutrophil is a migratory phagocytic cell that defends the host against bacteria and eliminates necrotic tissue. The neutrophil matures in the bone marrow and is released into the circulation as a fully differentiated cell. It is loaded with granules containing a variety of proteinases, hydrolases, antimicrobial agents and cationic proteins. The cell phagocytoses material and the granules fuse with the phagocytic vacuoles to degrade the foreign material. When the cells are challenged with a large amount of material, the granule contents may be released into the extracellular space where damage to surrounding tissue occurs. The neutrophil normally circulates in the human bloodstream for 7 to 10 hours. Thereafter, neutrophils are thought to exist for 1 to 2 days in the tissues before being cleared from the system. Granule constituents are formed during differentiation and replenishment of spent granules does not occur once the cells are in the circulation. Hence, the neutrophil is a fully differentiated end-cell poised to respond rapidly to stimuli, but it is rapidly spent in the process. Neutrophils have a NADPH-oxidase enzyme system on the plasma membrane which can be activated to produce toxic oxygen species including the superoxide anion (02–). Patients with chronic granulomatous disease (CGD) have a defective NADPH-oxidase system in their neutrophils, and are thus unable to generate 02–. Although neutrophils from patients with CGD are able to phagocytose bacteria, they are unable to kill the intracellular microbes and chronic, unresolved infections result.

181 Which of the following statements regarding the alternative complement pathway are true?
a. C1, C4 and C2 are involved
b. NH3 apparently activates complement via this pathway
c. Factors B and D are involved
d. Endotoxin activates complement via the alternative pathway
Answer: b, c, d

The alternative pathway differs from the classic pathway in that the first steps involving C1, C4 and C2 are bypassed. (See Figure 6-3 previously reproduced.) This pathway can be directly activated by agents other than antigen–antibody complex (e.g., complex polysaccharides like endotoxin and zymosan). Other serum protein factors (e.g., factors B and D) are involved in the activation sequence. Ammonia can attack the thiol-ester, producing amidated C3 and activate the alternative pathway. This leads to membrane attack complex formation (C5b-9) and activation of a number of phagocytic cell functions including toxic oxidant production. This phenomenon may have relevance to several in vivo disease states. In animal models of renal failure, elevated levels of renal vein NH3 have been correlated with impaired renal function and the presence of complement components at the sites of renal injury.

182 Platelet activating factor is:
a. Generated by the action of phospholipase A2 on membrane phospholipids
b. Antiinflammatory in most of its actions
c. Synthesized by endothelial and other cells
d. Exerts a variety of biologic effects which are platelet-independent
Answer: a, c, d

Like the eicosanoids, platelet-activating factor (PAF) is not stored in cells but is rapidly produced during inflammation. PAF exerts a variety of biologic effects that are platelet-independent. The synthesis of PAF is initiated by the activation of phospholipase A2. Activation of phospholipase A2 releases arachidonic acid in addition to lyso-PAF. Hence, PAF synthesis and eicosanoid production are coordinately regulated. PAF is synthesized on activation of a variety of inflammatory cells including platelets, neutrophils, basophils, mast cells, mononuclear phagocytes, eosinophils and vascular endothelium. PAF is a stimulatory agonist for many inflammatory cells, as well as for smooth muscle cells, vascular endothelium and others. PAF enhances the ability of neutrophils to respond to challenge with N-formylpeptides and LTB4. There is considerable overlap and redundancy in the effects produced by PAF and eicosanoids.

183 Platelets have a wide array of functions in inflammation. Which of the following are among these?
a. Synthesis and release of vasoactive eicosanoids
b. Release of chemotactic factors
c. Adherence to and coating of bacterial and tumor cells
d. Increase of vascular permeability
e. Phagocytosis of bacteria
Answer: a, b, c, d

Platelets are anucleated cells derived from megakaryocytes in the bone marrow. Their central role in hemostasis is well known. Platelets possess a wide array of functions in inflammation, including the following:
Synthesis and release of vasoactive eicosanoids
Release of chemotactic factors
Interaction with other inflammatory cells
Interaction with endothelial cells
Adherence to and coating of bacterial and tumor cells
Platelets are not capable of phagocytosis.
Few of the factors released or the functions carried by platelets during inflammation are unique to this cell type. Other inflammatory cells often have the same or similar capabilities. Indeed, some platelet functions may reflect vestigial functions inherited from a primitive precursor inflammatory cell. Platelets serve primarily as an amplifier or modulator of the inflammatory response.

184 Eicosanoids mediate inflammation in a variety of ways. Of the following statements, which are true with regard to this?
a. Eicosanoids are stored in cytoplasmic granules for release after receptor mediated signaling
b. Eicosanoids include prostaglandins, thromboxanes, leukotrienes and lipoxins
c. Eicosanoids generally have a plasma half-life measured in hours
d. Physiologic responses to eicosanoids include vasodilatation, vasoconstriction, increased vascular permeability and both chemotaxis and chemoattractant inhibition
Answer: b, d

The eicosanoids are derived from arachidonic acid (eicosatetraenoic acid) and consist of prostaglandins, thromboxanes, leukotrienes and lipoxins. The eicosanoids are not stored in cells but are rapidly synthesized by cells in response to a variety of stimuli. They have potent effects on vascular and bronchial smooth muscle including vasodilatation, vasoconstriction, bronchodilation and bronchoconstriction. In addition, they directly regulate vascular permeability. LTB4 is a potent, neutrophil chemoattractant whereas lipoxin A4 inhibits other chemoattractants. It appears that eicosanoids are important regulators of the endogenous inflammatory response. The rapid destruction of eicosanoids in the circulation limits their role primarily to that of mediators of local inflammatory changes. The local effects can be substantial. In general, the eicosanoids are rapidly metabolized or are so chemically unstable that they primarily exert their effects near the site of synthesis. Arachidonic acid does not exist in cells but is esterified to membrane phospholipids. Thus, the first step in the production of eicosanoids is phospholipase action, which liberates arachidonic acid. (Figure 6-6)

185 Which of the following statements are true?
a. Eosinophils are the major, if not sole, source of histamine in the blood
b. Basophils are effector cells in allergic reactions by virtue of IgE receptors
c. Mast cells are the major source of tissue histamine except in the stomach and central nervous system
d. Mononuclear phagocytes release a variety of proinflammatory cytokines and growth factors
Answer: b, c, d

Eosinophils constitute 1% to 3% of the leukocyte population of the bloodstream. They also reside in tissues and they exhibit phagocytic capabilities. They are less effective as bactericidal cells than neutrophils, but play a major role in defense against parasites. Eosinophils are primary effectors in allergic reactions by virtue of IgE receptors (which are not found on neutrophils).
Basophils are fully differentiated cells released into the circulation from bone marrow. Basophils are the major, if not sole, source of histamine in the blood. Histamine is a vasoactive amine and the major mediator of the IgE-mediated immediate hypersensitivity response. Histamine release from basophils is induced by complement products as well as by IgE receptors.
Mast cells are formed from bone marrow precursors that differentiate and proliferate in connective tissue. Mast cell granules contain histamine and proteoglycans. They represent the major source of histamine in most tissues except the stomach and central nervous system.
The monocyte–macrophage system consists of phagocytic cells scattered throughout the body. During acute inflammation, monocytes respond to chemoattractants released and are recruited to the site of inflammation. Mononuclear phagocytes respond to inflammatory stimuli by releasing M-CSF, GM-CSF, IL-1, and TNF, in addition to a variety of growth factors. These factors increase the production of mononuclear phagocytes and several of these factors enhance the ability of effector cells to respond to chemotactic stimuli released at the site of injury. Thus, the mononuclear phagocytes are important in initiating and augmenting the cycle of events that result in recruitment and activation of inflammatory cells at sites of inflammation.

186 Cellular injury from oxidants may be manifest by which of the following?
a. Cell membrane lipid peroxidation
b. DNA strand breaks
c. Cytoskeletal disassembly
d. ATP depletion
Answer: a, b, c, d

Free oxygen radicals are chemical species that are intermediates in the normal process of cellular respiration. Oxidants that are free radicals have been implicated as initiators of reactions which lead to a variety of cellular injuries. Oxidants are derived from several sources, notably phagocytes. Among the effects of oxygen free radicals are membrane lipid peroxidation, DNA strand breaks, cytoskeletal disassembly and inhibition of glucose metabolism leading to decreased cellular ATP concentrations. (Figure 6-16)

187 Which of the following acute-phase protein levels are increased in human plasma following acute inflammation?
a. C-reactive protein
b. Serum amyloid
c. a -Proteinase inhibitor
d. Fibrinogen
e. Albumin
Answer: a, b, c, d

The acute-phase response is a series of homeostatic responses of the organism to tissue injury in infection and inflammation. After an inflammatory stimulus occurs, a number of events occur within hours. These reflect altered set-points for various physiologic parameters including thermoregulation (fever), nitrogen balance (negative), and levels of various plasma proteins (increased or decreased). The erythrocyte sedimentation rate, which increases with inflammatory states, is an example of this phenomenon. The increased sedimentation rate is due to increased levels of fibrinogen and some of the other acute-phase reactants in plasma. Some proteins show a large increase (about 1000-fold), some a 4-to 5-fold increase, and others about a 50% increase over resting nonstressed levels.
Note that albumin is an acute-phase reactant. Levels of albumin drop after an inflammatory stimulus, usually 30% to 50% of the level before injury. The reason for the decrease in production is poorly understood.

188 Which of the following statements regarding endothelial cells in acute inflammation are true?
a. Endothelial cells are characterized by phenotypic homogeneity
b. Specific patterns of receptor expression regulate leukocyte adherence
c. Endothelial cell nitric oxide generation regulates regional blood flow and leukocyte adhesion
d. Endothelial cells may be capable of phagocytosis
Answer: b, c, d

Endothelial cells are increasingly recognized to be phenotypically heterogeneous. Specific receptor molecules are expressed at various sites where they help to direct lymphocytes and other leukocytes to their appropriate target organ. In the high endothelial venues, these receptor molecules are known as vascular addressing. Endothelial cells play a major role in regulating vascular tone. This is the result of angiotensin-converting enzyme on the cell surface as well as the production of both endothelia (a potent vasoconstrictor) and nitric oxide (a potent vasodilator). Both play important physiologic roles in determining the distribution of blood flow. In addition, recent evidence suggests that NO may have direct effects upon the expression of a variety of leukocyte adhesion molecules. Under unusual circumstances, endothelial cells can exhibit macrophage-like properties in that they can act as antigen-presenting cells and also phagocytose particles. They may also be a significant source of oxidants in inflammatory reactions after ischemic injury. Endothelial cells are not passive participants in inflammatory processes; rather, they possess the ability to direct and focus many aspects of an inflammatory event.

189 The first line of host defense is the barrier presented to the external environment. Which of the following statement(s) is/are true concerning host barriers?
a. Sebaceous glands secrete chemical compounds that maintain a relatively high pH, providing effective bacterial stasis
b. Within the respiratory tract, ciliary function serves to extrude microorganisms trapped within the mucus secretion layer
c. The low pH within the stomach markedly decreases bacterial content of the upper gastrointestinal tract
d. Gut peristalsis serves to prevent microbial adherence and invasion
Answer: b, c, d

The skin, mucus membranes, and epithelial layers of various organs of the body constitute effective physical barriers against microbial invasion. In certain portions of the body, these barriers have developed ancillary adaptations to increase the effectiveness of the barrier functions. Skin structures such as sebaceous glands secrete chemical compounds that serve to maintain a relatively low pH, providing effective bacterial stasis. Mucus secretion by specialized glands within the bronchi and gut provide a mucus layer that represents a physical and chemical barrier to microbial invasion. Within the respiratory tract, ciliary function serves to extrude microorganisms trapped within this mucus layer. In the alimentary track, the very low pH within the stomach and gut peristalsis both serve to prevent microbial adherence and invasion.

190 Which of the following statement(s) is/are true concerning the antibody response to an invading antigen?
a. All antibodies are composed of one type of heavy and one type of light protein chain
b. The carboxyl terminus of the heavy chain is the antigen binding site
c. Antibody of the immunoglobulin G class is the initial antibody produced in response to an antigenic stimulus
d. Immunoglobulins A, D, and E play an active role in the circulating humoral response
Answer: a

Humoral defenses consist of antibody (immunoglobulin; Ig) and complement. All Ig classes (IgM, IgG, IgA, IgE, IgD) and IgG subclasses are composed of one type (M, G, A, E, D) of heavy and one type (K and g ) of light protein chains that consist of several domains both structurally and functionally. Each Ig molecule contains one or more units that consist of two heavy and two light chains linked by disulfide bonds. The amino terminus of both heavy and light chains contain several hypervariable regions that fold in three dimensions to produce the antigen-binding site. The carboxyl terminus of the heavy chains contain regions that activate complement and bind Fc receptors, by which direct adherence to polymorphonuclear leukocytes and macrophages take place after antigen binding occurs.
Initially, antibody of the IgM class is produced in response to an antigenic stimulus. A second exposure to the same antigen, or a cross-reactive antigen, leads to the so-called second set response, in which antibody of the IgG class with two binding sites is produced more rapidly and in larger quantity compared to the initial IgM primary response. Immunoglobulin of the IgA class is secreted by gut-associated lymphoid tissue and is combined with secretory components of protein to form a dimer termed secretory IgA. This antibody acts at a variety of epithelial sites to prevent microbial adherence and invasion. IgD and IgE exist in smaller amounts in the circulation and do not appear to play a major role as host defense components.

191 Increasing evidence has implicated gram-negative bacterial lipopolysaccharide (LPS endotoxin) as the portion of the gram-negative bacterial cell membrane responsible for many, if not all the toxic effects that occur during gram-negative bacterial sepsis. The following statement(s) is/are true concerning LPS and the host response.
a. The LPS molecule can in itself cause physiologic responses similar to that seen during gram-negative bacterial sepsis
b. LPS triggers host macrophages to release a variety of cytokines including TNF-a, IL-1a, and IL-1b, IL-6, and IFN-a
c. Excessive cytokine production is not associated with detrimental consequences
d. TNFa and IL-1b appear to be the primary mediators within the host, exerting deleterious effects on the host when excessive amounts reach the systemic circulation
Answer: a, b, d

The LPS molecule exerts diverse effects on the mammalian host. Immunologic responses to LPS include nonspecific polyclonal B-cell proliferation, macrophage activation and cytokine secretion, tolerance to subsequent LPS or bacterial challenge, and production of antibody directed against various portions of the LPS molecule after repeated challenge. Physiologic responses similar to those seen during gram-negative bacterial sepsis occur during LPS administration alone and include hypotension, hypoxemia, acidosis, bacterial translocation across the gut, complement and coagulation cascade activation, white blood cell and platelet margination, and death. Indirect effects result from LPS-triggering of host macrophages. Activated macrophages secrete a wide array of cytokines that include TNF-a, IL-1a, and IL-1b, IL-6 and interferon-a (IFNa). Excessive secretion of cytokines produce substantial systemic effects in the mammalian host. TNFa and IL-1b appear to be the primary mediators within the local host milieu, exerting deleterious effects on the host only after large amounts are secreted and reach the systemic circulation.

192 Which of the following statement(s) concerning the gut microflora is/are correct?
a. Gut microflora evolves constantly throughout development
b. The gut microflora can contribute to the physical and chemical barriers at the mucus membrane level
c. Most of the microorganisms found in the oropharynx eventually pass into the intestine
d. In the colon, anaerobic organisms outnumber aerobic organisms in a ratio in excess of 100:1
Answer: b, d

The composition of the gut microflora is established in neonates after ingestion of microbes that are acquired during contamination from the birth canal and during initial feeding, and remain relatively constant thereafter. Although this flora acts to promote development of the immune system, the specific interactions that produce this effect have not been fully elucidated. The microflora also contributes to physical and chemical barriers at the mucus membrane level, in that many autochthonous microbes possess adhesion proteins by which they can bind to certain areas of the mucosal cell or to specific types of bacteria, occupying potential binding sites for pathogenic organisms and producing a substantial physical mucobacterial layer. The oropharynx contains a number of aerobic and anaerobic microorganisms, however, these microbial inhabitants do not usually pass into the intestine, because the stomach itself represents a significant barrier to invading microorganisms by virtue of its low pH, which kills most microbes. The upper small intestine contains few organisms, mainly gram-positive aerobes and lactobacilli. Conversely, the lower small intestine contains a large number of aerobes and anaerobic forms, especially in patients in whom the ileocecal valve allows free backwash of cecal contents into the terminal ileum. Within the colon, a wide diversity and a large number of facultative and strict anaerobic isolates are present. Only a small number of aerobes are present, these microbes being outnumbered 100–300 to 1 by anaerobes.

193 The use of antibiotics can be based on either the clinical course of a patient without the benefit of well-defined microbiologic data (empiric therapy), or targeted at specific identified pathogens once sensitivity reports are available (directed therapy). The following statement(s) is/are true concerning these therapies.
a. The issue of toxic side effects of antibiotics is only important in dealing with emperic therapy
c. With the empiric use of antibiotics, a diligent search for the septic source should be undertaken and continued until identified
d. In clinical situations in which polymicrobial infection is identified, specifically-directed treatment for the predominant organism is satisfactory
Answer: c

The use of empiric therapy without the benefit of well-defined microbiologic data is appropriate when there is sufficient clinical evidence to support the diagnosis such that it would be imprudent to withhold antimicrobial therapy. In this setting, however, a diligent search for the septic focus source should be undertaken and continued (cultures, radiographic procedures, etc.), and initial limits should be placed in the course of empiric therapy with continued reevaluation based on the clinical course of the patient. The choice of antibiotic agents should be based on the clinical situation and known activity patterns within the given institution. Single broad-spectrum agents, although suffering slightly from a lack of individual pathogen specificity, are useful in this setting in that they provide a broad coverage against several groups of pathogens and may avoid some of the toxic effects with specific combined modality regimens. Similarly, for directed therapy, single-agent therapy has been demonstrated to be equivalent to combined therapy and should be chosen in an attempt to select agents with appropriate sensitivities which retain suitable clinical efficacy but exhibit minimal toxicity. After review of cultural reports, many patients have demonstrated polymicrobial infection. Because experimental clinical evidence supports the concept of aerobic-anaerobic synergy, therapy should be directed against all potential components of the infection if the body site is such that these microorganisms may be present.

194 The following statement(s) is/are true concerning newer detection methods of systemic infection.
a. Enzyme-linked immunosorbent assay (ELISA) is a rapid immunologic assay used for both antigen and antibody detection
b. Southern, northern, and western immunoblot techniques are used to detect DNA, RNA, or proteins, respectively
c. Polymerase chain reaction (PCR) is a sensitive assay used to detect small amounts of microbial DNA, thus detecting infection at its early stages
d. Infectious agents currently detected using advanced molecular techniques include cytomegalovirus (CMV) and human immunodeficiency virus (HIV)
Answer: a, b, c, d

Although the classic detection of infection based on clinical signs of infection and bacterial culture remain the most common clinical tools, increasing reliance has been placed on assays that do not employ cultural data. Specifically, the antibody and cytokine host responses are being intensely examined and extremely sensitive amplified assays that rely on antigen, antibody or microbial DNA detection are employed in the clinical setting. Enzyme-linked immunosorbent assay (ELISA) is a rapid, antigen-based, immunologic assay that can be used for both antigen and antibody detection, for determination of antibody titer, as well as for screening for monoclonal antibody production. Transblot techniques are being used increasingly in the clinical setting. These include southern, northern, and western immunotransblot techniques used to detect DNA, RNA, or proteins, respectively. The polymerase chain reaction (PCR) is being used in some centers as a sensitive assay to detect small amounts of microbial DNA. This technique involves extraction of the DNA from the test sample with in vitro amplification through repeated nucleic acid denaturing and polymerization so that the gene copy number increases exponentially. This marked amplification of the gene copy number results in extremely sensitive tests which can detect infection at its early stages.
Clinically, these detection methods are being used to detect a wide variety of infectious agents including CMV and HIV. Furthermore, preliminary investigations into possible detection of fungal pathogens are underway.

195 Cytokines are low-molecular-weight polypeptides exerting a wide variety of biologic effects at both local and systemic levels. Which of the following statement(s) is/are true concerning the production and actions of cytokines?
a. Cytokines are produced solely by macrophages
b. Cytokines act only on other cells within the same local environment
c. Cytokines may have both protective and deleterious effects on the host
d. Each specific cytokine is produced by a single cell type
Answer: c

Macrophages, endothelial cells, lymphocytes, and other cells secrete a large number of different compounds, termed cytokines, that are most probably evolved for the purpose of local intercellular and intracellular signaling. Cytokines frequently are secreted after initial lymphocyte or macrophage activation, and may act on the secreting cell itself (autocrine activation) or on other cells within the same local environment (paracrine activation) to cause increased secretion of the same cytokine or other cytokines, respectively. Some cytokines are produced by several cell types, and most produce a wide array of effects. The duality of the effects of the cytokine component of host defenses, exerting both salutatory and deleterious effects on the host, has become increasingly evident.

196 The following statement(s) is/are true concerning cellular defense mechanisms.
a. Macrophages function solely as antigen processing cells in the initial reaction to exposure to an antigen
b. Macrophages may become activated and secrete cytokines
c. Macrophages serve as phagocytic cells in the tissues but not within the bloodstream
d. Polymorphonucleocytes (PMNS) are normally present in only small numbers within the tissue and enter an area of infection through diapedesis
Answer: b, d

A wide variety of cell types serve to provide host defense at several levels. Macrophages act as the initial antigen processing cell that serves to present antigen to help T cells, thus initiating the immune response. Macrophages, however, are pluripotent cells that, in the process of engulfing and processing antigen, may become activated. Activated macrophages secrete a variety of cytokines. Macrophages also act as phagocytic cells in the tissues and within the bloodstream, and because of their resident nature in many tissues, also represent the first line of host defenses in many areas of the body, even before activation. PMNS are present within the bloodstream, but only in small numbers within the tissue, and enter an area of infection through diapedesis after chemotactic stimuli are excluded by macrophages, bacterial breakdown products, and complement activation.

197 A diabetic develops a severe perineal infection with skin necrosis, subcutaneous crepitance, and drainage of a thin, watery, grayish and foul-smelling fluid. Management should consist of:
a. Gram stain of the fluid, which will likely demonstrate multiple bacteria including predominantly gram-positive rods
b. A CT scan is indicated in a stable patient to define the extent of the disease
c. Broad spectrum antibiotics followed with prompt extensive debridement is indicated
d. A safe guideline is to resect infected necrotic tissue so that a several centimeter margin of grossly normal, healthy tissue can be achieved
e. A colostomy is of little benefit in this situation
Answer: a, b, c, d

The presence of severe perineal infection (referred to as Fournier gangrene when this process involves the perineum and scrotum in males) is associated with a continued high mortality despite aggressive and appropriate therapy. The clinical description provided would suggest an underlying soft tissue necrosis. In a stable patient radiologic studies including a CT scan to define the extent of the disease and the presence of pelvic infection is indicated. Gram stain will likely show evidence of polymicrobial organisms but the presence of Clostridia marked by gram-positive rods would suggest involvement with this organism. Prompt, aggressive and extensive debridement to remove all devitalized and affected tissue and the addition of broad spectrum antibiotics, fluid resuscitation, hemodynamic monitoring, and nutritional support would appear to afford the patient the best chance of survival. The clearest guidelines to determine the limits of resection involve removal of clearly infected, necrotic tissue so that margins several centimeters into grossly normal, healthy tissue are achieved. Because the entire perineal region and buttocks are frequently involved in these patients, performance of a fecal stream diversion by means of a colostomy often provides improved wound care and patient management, although it is not invariably a positive outcome.

198 The use of prophylactic antibiotics has become commonplace. Which of the following statement(s) is/are true concerning the prophylactic use of antibiotics?
a. The appropriate use of prophylactic antibiotics must include the initiation of the agent prior to the surgical procedure
b. Continuing the antibiotic into the postoperative period has led to improved results in antibiotic prophylaxis
c. The prophylactic administration of broad-spectrum agents (third-generation cephalosporins) has been shown to be particularly advantageous
d. The topical use of antimicrobial agents is of no advantage in the prophylactic setting
Answer: a

Intravenous administration of an antibiotic is clearly indicated for patients undergoing clean contaminated operations. These antibiotics should be administered prior to surgery to obtain adequate tissue levels at the time of potential contamination. However, there has been no added benefit demonstrated for the postoperative use of antibiotics with regard to prophylaxis. The choice of antibiotic is a complex issue which remains unresolved largely because both superficial and deep wound infections can occur as a result of either or both skin (superficial wound) flora (e.g., Staphylococcus aureus) and body site (deep wound) infection. For this reason, the administration of agents which possess activity directed against these expected agents is reasonable. Although administration of a first-generation cephalosporin is acceptable, second-generation cephalosporins or extended-spectrum penicillins with gram-positive and gram-negative activity and biliary tract excretion may be more suitable for patients undergoing gastrointestinal or biliary tract procedures. Similarly, the use of agents with additional anaerobic activity for patients undergoing gastrointestinal procedures involving the small bowel or colon should be considered. The administration of broad-spectrum agents such as third-generation cephalosporins for prophylaxis does not seem to provide additional benefit in comparison to the above-mentioned type antibiotics and may foster the development of resistant organisms within a given institution or superinfection within a given patient. There is evidence that in some cases the topical use of antimicrobial agents is equivalent to the administration of intravenous antimicrobial agent antibiotics.

199 If a necrotizing soft tissue infection is considered, therapy mandates:
a. Empiric administration of antibiotics active against gram-positive, gram-negative, and anaerobic bacteria
b. Due to usually resistant species, penicillin is not indicated
c. Immediate operative intervention and aggressive resection of all involved tissues is mandatory
d. The use of hyperbaric oxygen has been demonstrated to be clearly advantageous
Answer: a, c

Identification of a necrotizing, soft tissue infection mandates immediate operative intervention with aggressive resection of all involved tissues and empiric administration of antibiotics active against gram-positive, gram-negative, and anaerobic bacteria. In most cases, this involves the use of several antimicrobial antibiotics in combination. Because of concern in all cases for the presence of Clostridia infection, high doses of aqueous penicillin G are administered. Gram-positive organisms are treated with vancomycin or a semisynthetic penicillin and gram-negative organisms are treated with an aminoglycoside or a monobactam. Anaerobic coverage is typically achieved by use of metronidazole of clindamycin. The use of hyperbaric oxygen therapy is controversial and unfortunately due to the rarity of the disease, prospective randomized data is not available so that the literature remains without controlled trials demonstrating any additional benefits derived from hyperbaric oxygen therapy.

200 Wounds are classified according to the likelihood of bacterial contamination. Which of the following statement(s) is/are true concerning wound classifications?
a. A clean-contaminated wound would be that associated with an elective colon resection with adequate mechanical and antibiotic bowel preparation
b. A contaminated wound would include a resection of obstructed bowel with gross spillage of intestinal contents
c. In a clean wound, no viscus is entered
d. Antibiotic prophylaxis should be administered for all clean-contaminated and contaminated wounds and selectively in patients involving a clean wound
Answer: a, b, c, d

Wounds are classified under three classes according to the likelihood of bacterial contamination: 1) clean (no viscus is entered; e.g., herniorrhaphy); 2) clean-contaminated (minimal contamination; e.g., elective colon resection with adequate mechanical and antibiotic bowel preparation, and 3) contaminated (heavily contaminated surgery; e.g., resection of unprepared, obstructed bowel with gross spillage of intestinal contents or stool, drainage of abscesses, debridement of traumatic neglected wounds). Antibiotic prophylaxis generally should be administered for class 2 and 3 types of wounds, but patients undergoing clean surgery do not always require antimicrobial antibiotic prophylaxis. An exception to this tenet involves cases in which a prosthetic material may be used (artificial joint, heart valve, tissue patch).

201 The following statement(s) is/are true concerning HIV infection.
a. Initial screening with ELISA is highly sensitive but can be associated with a false positive rate of 25%
b. Treatment with azidothymidine (AZD) appears to prolong survival when administered early in the disease
c. Predisposition to infection in HIV infection is primarily due to reduction in the number of helper T cells
d. Common infections in patients with AIDS are Pneumocystis, carinii pneumonia, CMV pneumonitis, Cryptococcus meningitis, and disseminated infection due to atypical mycobacteria
Answer: b, c, d

Acquired immunodeficiency syndrome (AIDS) is a syndrome caused by the human retrovirus (HIV-1) that infects T lymphocytes and causes severe immunosuppression. Individuals who become infected with HIV are prone to a variety of infections and different types of malignancy. A spectrum exists in which patients regress from asymptomatic infection, to development of AIDS-related complex (ARC) of diseases to AIDS itself. Common infections occurring in patients with AIDS are Pneumocystis carinii pneumonia; CMV pneumonitis; gastritis, hepatitis and meningitis due to Cryptococcus neoformans; and pneumonia and disseminated infection due to atypical mycobacteria. Predisposition to these infections is due, in part, to the lymphotrophic nature of HIV, which markedly reduces the number of helper T cells as well as the absolute number of T cells.
HIV detection typically consists of initial ELISA screening, but this test has about a 1–3% false-positive rate, thus mandating all positive tests be confirmed by the western immunoblot analysis.
Treatment of ARC and AIDS consists of aggressive antiinfective therapy once a specific infection occurs and the use of AZT. AZT has been shown to prolong survival when administered early in the course of disease and is considered routine therapy.

202 The following statement(s) is/are true concerning initial microbiologic diagnostic techniques.
a. Appropriate expeditious transport of specimens to the microbiology laboratory is essential for obtaining accurate clinical information
b. The use of potassium hydroxide in preparing a specimen slide for light microscopy will be useful in identification of anaerobic bacteria
c. Antibiotic sensitivity is determined by exposing the specific microorganism to varying amounts of antibiotic with the concentration of the antibiotic inhibiting growth referred to as the MIC (minimal inhibitory concentration)
d. Serum levels of antimicrobial agents should achieve in excess of a 4-to 8-fold increase over the MIC to be considered clinically efficacious.
Answer: a, c, d

Because most surgical infections are polymicrobial, specimens should be cultured for aerobic and anaerobic bacteria, as well as fungi. Although aerobic and aerotolerant microorganisms often do not require special transport media, a delay in specimen processing may markedly reduce the yield, and anaerobic transport media have been demonstrated to markedly increase the cultural yield of this type of organism. The initial piece of information gained concerning potential infection may come from simple staining of a specimen. Gram stain, which will identify the staining characteristics of the organisms, as well as their number should be performed on all specimens. Potassium hydroxide is useful in that it will lyse bacteria and other cellular elements within a preparation and allow observation of yeast or mycelial elements.
Initial culture results may solely indicate that microorganisms are growing and full characterization may take two to three days. Once a specific microorganism is identified, a sample is inoculated during the log phase into broth containing varying amounts of an antibiotic. After an 18-to 24-hour period, the tube or well that exhibits no visible growth is then noted, and the reciprocal of this dilution is termed the minimal inhibitory concentration (MIC). This value may be compared to either measured or known achievable serum levels for a particular antibiotic. In general, antimicrobial agents that achieve in excess of a 4-to 8-fold increase over MIC during the peak serum level have been demonstrated to be clinically efficacious.

203 The complement system consists of a series of serum proteins that exist in a quiescent or very low-level state of activation in the uninfected host. Which of the following statement(s) is/are true concerning complement activation?
a. The alternate (properdin) pathway of complement activation can occur directly through contact with fungal or bacterial cell wall compounds
b. Complement component fragments may serve to decrease vascular permeability
c. Excessive complement activation can produce deleterious effects
d. Fragments of certain complement components serve as chemoattractants to additional cellular components of the host defense mechanism
Answer: a, c, d

Complement activation can occur through either classic or alternate (properdin) pathways, both of which eventuate in deposition of terminal complement pathway components on the antigenic cell surface. The classic pathway of complement activation usually begins with immunoglobulin G-binding which has also bound the antigen. The alternate pathway activation occurs in response to activation of direct binding of the antigen or directly through contact with fungal and bacterial cell wall compounds such as zymosan and gram-negative bacterial lipopolysaccharide (LPS endotoxin). Several complement components represent important host defenses acting to recruit or augment cellular host defenses or to directly inactivate invading microbes through lytic activity. The production of complement component fractions C3a and C5a during activation of this cascade serve primarily to markedly increase vascular permeability, and C5a functions as a PMN and macrophage chemoattractant. This process leads to the recruitment of additional humoral and cellular defenses to the specific area of infection. Excessive complement activation can produce deleterious effects in some instances. Complement activation causes enhanced PMN adhesion, margination, and release of lysosomal enzymes that can directly damage certain target tissues, such as the lung.

204 A 55-year-old renal transplant patient has been hospitalized in a Surgical Intensive Care Unit, receiving a prolonged course of antibiotics following an attack of acute cholecystitis. The following statement(s) is/are true concerning his management.
a. Due to the potential risk of Candida infection, prophylaxis with oral nystatin should be instituted early in the patient’s course
b. A Candida urinary tract infection should be treated with systemic amphotericin B
c. Changes of Candida retinitis are of little significance
d. The presence of a virulent Candida bacteremia should suggest a dosage reduction in immunosuppressive agents until the infection can be adequately controlled
Answer: a, d

Infections due to fungal pathogens have become increasingly common during the past decade, frequently occurring in patients undergoing prolonged hospitalization in the Surgical Intensive Care Unit and in immunocompromised individuals. Prophylaxis with oral antifungal agents (nystatin) is warranted, especially during periods of maximal immunosuppression in transplant patients, in patients with uncontrolled diabetes, or during some cases of prolonged antibacterial microbial therapy. In general, local, apparently noninvasive Candida infections involving the integument and mucus membranes are treated with oral decontamination and topical antifungal therapy using topical agents such as nystatin. Candida urinary tract infections can be treated with either an oral antifungal agent or with topical amphotericin B as a continuous bladder irrigation. Several studies have demonstrated that those patients with three positive sites of Candida infection, or with peritoneal or blood cultures positive for Candida exhibit higher survival rates when amphotericin B therapy is instituted earlier in the course of infection. The presence of retinal changes compatible with Candida retinitis or Candida present within the peritoneal cavity are considered indications for a limited course of amphotericin B therapy (300% to 500 mg). Patients receiving exogenous immunosuppressive agents should undergo a marked dose reduction, and some agents should be discontinued until evidence of infection is absolutely controlled or is eradicated.

205 The initiation of a humoral immune response involves a complex interaction of the antigen, cells and intercellular messengers. Which of the following statement(s) concerning the initiation of the humoral immune response is/are correct?
a. Helper T lymphocytes stimulate B lymphocytes through secretion of cytokines such as interleukin 4 and 6
b. A number of cells can aid in presenting the antigen to the helper T cell including B lymphocytes and macrophages
c. All antigens require coordinated efforts of the various cellular components of the immune system
d. An antigen must be a living microorganism
Answer: a, b

Stimulation of the immune system occurs after a variety of antigen-presenting cells (B lymphocytes, macrophages, dendritic cells, and Langerhans cells) act to engulf, process, and present antigen to T lymphocytes of helper lineage. These T lymphocytes, in turn, act to stimulate B lymphocytes to become mature plasmacytes (through secretion of cytokines such as interleukin 4 and 6) dedicated to the production of antibody directed against the specific antigen. An antigen may be defined as any substance that stimulates the host immune response; that is, that the host immune system recognizes is foreign. Thus, an antigen may be an invading microorganism, an inert particle, or any type of chemical compound that triggers the host immune system. Although some antigens are able to directly stimulate B lymphocytes in and of themselves to produce antibody (many polysaccharides), most antigens require coordinated efforts of the various cellular components of the immune system.

206 The following statement(s) is/are true concerning viral infections.
a. The most common post-transplantation viral infections are caused by herpes viruses and include CMV and herpes simplex virus
b. Viral infections occur at equal frequency anytime during the post-transplantation period
c. CMV infection in the post-transplant patient is most likely a pulmonary process
d. Herpes simplex virus (HSV) infection primarily presents with a mononucleosis-type syndrome with fever, lethargy, and cough
Answer: a, c,

Solid organ transplant patients are prone to develop viral infection by virtue of exogenous immunosuppression. The most common post-transplantation viral infections are those caused by herpes viruses (CMV, herpes simplex virus [HSV], Epstein-Barr virus [EBV], and Varicella-Zoster virus [VZV]). All are most common during periods of maximal host immunosuppression that occur immediately post-transplantation and during periods of allograft rejection. CMV is a common cause of fever after solid organ transplantation, and evidence of CMV infection occurs in approximately 30% of patients. The most common presentation for CMV infection is that of a febrile, leukopenic patient with a cough, diffuse interstitial infiltrates on chest x-ray, and hypoxia.
HSV infection causes primarily oral pharyngeal ulcerations in most cases, although sporadic cases of disseminated disease have been reported. EBV causes an occasional case of mononucleosis-type syndrome but has also been clearly indicated in the pathogenesis of post-transplantation lymphomas. VZV infection can present as disseminated and occasionally life-threatening infections in the nonimmune transplant patient or as painful herpes zoster in patients who have previously developed chicken pox.

207 The following statement(s) is/are true concerning necrotizing fascitis.
a. Mortality rates as high as 40% can be expected
b. The infection involves only the superficial fascia, sparing the deep muscular fascia
c. An impaired immune system is a common factor predisposing to this condition
d. The infection is usually polymicrobial
e. Necrotizing fascitis is most likely to develop in the face of impaired fascial blood supply
Answer: a, c, d, e

Necrotizing fascitis is an uncommon infection of the deep and superficial fascia that is associated with mortality as high as 40% in many series. Although many underlying disease processes predispose patients to necrotizing fascitis, three common factors are almost invariably present: 1) impairment of the immune system; 2) compromise of fascial blood supply, and 3) the presence of microorganisms that are able to proliferate within this environment. Infections of this type are usually polymicrobial in nature, with gram-positive organisms such as staphylococci and streptococci, gram-negative enteric bacteria, and gram-negative anaerobic being frequently identified. These polymicrobial cultural results are assuredly indicative of the occurrence of a synergistic process, perhaps in large part accounting for the severity of these infections. Some microorganisms possess virulence factors that, in conjunction with an underlying host predisposition, allow this disease process to occur without dependence on other bacteria. Examples of such bacteria include Clostridium, Pseudomonas, and Aeromonas. In these patients, the process is often fulminant and is frequently associated with cellulitis, myositis, fascitis, and bacteremia with attendant high mortality.

208 New treatment modalities designed to modulate host defense mechanisms that have been demonstrated conclusively to be of benefit include:
a. Gut decontamination
b. Anti-LPS antibody
c. Anti-TNF antibody
d. Thymopentin
e. None of the above
Answer: e

Selective gut decontamination involves the use of orally administered antibiotics that achieve a high intraluminal level directed against gram-negative aerobes and yeast, leaving the host anaerobic intestinal microflora relatively undisrupted. Although a reduction and alteration of the microorganisms responsible for infectious episodes have been demonstrated in certain groups of patients, a clear-cut impact on host mortality has not been shown. Because LPS may be responsible for toxicity both directly and through host mediator systems, the availability of agents to bind against this portion of the gram-negative bacteria to reduce mortality has been intensively examined. Unfortunately, large multicenter randomized trials provide no evidence of benefit for this treatment. Similarly, since many of the systemic manifestations of gram-negative bacteremia are mediated by cytokines, the effect of an anti-TNF antibody preparation is currently in clinical trial. No proven benefits have yet been identified. Finally, the use of immunostimulants to enhance the state of activation of host defenses has been proposed. Thymopentin is a peptide that contains active thymopoetin, a thymic molecule that acts to stimulate T-lymphocyte activity. Preliminary trials indicate that this agent ameliorates host septic response after major operations and trauma but conclusive evidence that concurrent reduction of infection-related mortality occurs is not available.

209 Antibacterial agents can be classified with regard to their structure, mechanism of action, and activity pattern against various types of bacterial pathogens. Which of the following statement(s) is/are true concerning antimicrobial classes?
a. Penicillins and cephalosporins share the compound structure of a b-lactam ring which binds to bacterial division plate proteins
b. Tetracyclines and macrolides such as erythromycin inhibit bacterial ribosomal activity and therefore protein synthesis
c. Aminoglycosides act in a similar fashion to tetracyclines and therefore are both bacteriostatic
d. Sulfonamides and trimethoprim act synergistically to inhibit purine synthesis
Answer: a, b, d

Penicillins, cephalosporins, and monobactams possess a b-lactam ring of some type and act to bind bacterial division plate proteins, thus inhibiting cell wall peptidoglycan synthesis and either causing or inducing autolytic bacteriolysis. Because gram-positive and gram-negative bacteria possess different types of division plate proteins, many of these agents exhibit differential activity between these two types of microorganisms. Tetracyclines, chloramphenicol, and macrolides inhibit bacterial ribosomal activity, and thus overall protein synthesis by a variety of different mechanisms. Aminoglycosides act to inhibit protein synthesis and also presumably act on a different target site, a supposition based on the fact that aminoglycosides are bacteriolytic and the other agents are bacteriostatic. Vancomycin inhibits assembly of peptidoglycan polymers, whereas quinolones bind to DNA helicase proteins and inhibit bacterial DNA synthesis. Sulfonamides and trimethoprim act in different mechanisms to inhibit protein synthesis, therefore two agents in combination act synergistically.

210 The treatment of the following patient should include:
a. Initial empiric therapy directed against both aerobes and anaerobes
b. The addition of anti-fungal therapy in an elderly patient
c. A minimum of two weeks of antibiotic therapy is indicated
d. The addition of appropriate antibiotic therapy has made surgical therapy unnecessary in such cases
e. Either a single agent or combination therapy is appropriate if the agents selected possess activity against both aerobic and anaerobic bacteria
Answer: a, e

The primary treatment for a perforated viscus is surgical, however antimicrobial therapy is an extremely important adjunct. Empiric antibiotic therapy for secondary bacterial peritonitis and intraabdominal abscess should be directed against both aerobes and anaerobes. Administration of an agent directed against only one component of the infection or the other is inferior to combined therapy. Several studies indicate that the results of using several agents in combination is equivalent to the use of a single agent therapy as long as the agents selected possess activity against both components of the infection. The addition of antientercoccal or antifungal agents as initial therapy has not been substantiated. The most beneficial duration of antibiotic therapy must be based on the setting for the specific patient. Minimal peritoneal contamination with adequate surgical treatment may be treated with a three-to five-day course of antibiotics, whereas longer periods are indicated for immunosuppressed patients and with patients with extensive contamination.

211 The following statement(s) is/are true concerning host defense mechanisms to intraabdominal infection.
a. Bacterial clearance can occur via translymphatic absorption
b. Phagocytic activity and bacterial killing can occur via resident phagocytic cells and an influx of PMNs
c. A fibrinogen-rich inflammatory exudate is released into the peritoneal cavity, trapping large numbers of bacteria and other particulate matter
d. Perforations of a bowel may be walled off but are seldom sealed by the omentum and other mobile viscera
Answer: a, b, c

The introduction of microorganisms into the normally sterile peritoneal environment invoke several potent specialized host antimicrobial defense mechanisms. Bacterial clearance, also termed translymphatic absorption, occurs through specialized structures found only on the peritoneal mesothelium on the underside of the diaphragm that act as conduits for both fluid and particulate matter. Lymphatic channels eventually form which drain into the venous circulation via the thoracic duct. Bacteria not cleared via translymphatic absorption are rapidly engulfed by resident and recruited phagocytic cells including resident macrophages on the peritoneal surface and omentum and attracted PMNs. The final primitive host defense mechanism is sequestration by which a fibrinogen-rich exudate containing plasma oposonins appears during peritoneal infection and fibrin polymerization occurs. Fibrin has the capacity to trap large numbers of bacteria and other particulate matter. Acting in conjunction with omentum and other mobile viscera, perforations are sealed and the contaminated enteric contents walled off, preventing continued soilage of the peritoneal cavity.

212 A 67-year-old male presents with an intraabdominal abscess secondary to perforated sigmoid diverticulitis. The following statement(s) is/are true concerning his intraabdominal abscess.
a. Culture will likely reveal a solitary organism
b. Both aerobic and anaerobic islets are encountered in 50% of specimens
c. The most common aerobic islet will be likely E. coli and other gram-negative enteric bacilli
d. The most common anaerobic islet will be a Bacteroides species
Answer: b, c, d

Typically an intraabdominal infection results in perforation of a hollow viscus and the ensuing contamination of a normally sterile peritoneal cavity. The normal bacterial flora found in that particular location of the alimentary tract thus determines the initial inoculum. In parallel with the overall quantity of microorganisms, (both aerobes but predominantly anaerobes) perforations of the lower small bowel and colon produce a high frequency of infections that contain anaerobic microorganisms. Certain predictable patterns of bacterial islets are found, but on average four to five islets occur in patients with established intraabdominal infection, more than half of which are anaerobes. Both aerobes and anaerobes are encountered in 80% to 90% of specimens. Commonly encountered aerobes isolated are E. coli and other gram-negative enteric bacilli such as Enterobacter, Klebsiella. Among the anaerobes, Bacteroides species (especially B. fragilis, Clostridium), and anaerobic cocci are most consistently isolated.

213 The following statement(s) is/are true concerning gram-negative bacterial sepsis.
a. Mortality due to this condition has almost been eliminated due to therapeutic intervention with antibiotics, aggressive hemodynamic monitoring and fluid resuscitation
b. Recent series have noted a decrease in the incidence of this condition
c. Predisposing factors include old age, malnutrition, and immunosuppression
d. Pseudomonas bacteremia is the most common cause of gram-negative sepsis
e. Polymicrobial sepsis is generally considered a more serious problem than sepsis due to a single organism
Answer: c, e

Gram-negative bacterial sepsis is a serious disease process that produces substantial morbidity and mortality in both normal and immunocompromised patients (10% to 20% and 30% lethality, respectively), despite therapeutic intervention with antimicrobial agents, aggressive hemodynamic monitoring, fluid resuscitation, and metabolic support. During the past several decades, nosocomial infections due to gram-negative pathogens have increased in frequency with resultant increase in the incidence of gram-negative bacteremia to between 3 and 13 cases per 1000 hospital admissions. Factors that predispose to these infections include: 1) underlying host disease processes such as malignancy, diabetes; 2) old age and disability; 3) malnutrition; 4) previous or concurrent antimicrobial antibiotic therapy; 5) major operations; 6) respiratory or urinary manipulation or intubation; and 7) immunosuppression.
Although many different organisms cause this form of sepsis, E. coli predominates in overall frequency. Also common are isolates of Klebsiella, Enterobacter and Serratia; Pseudomonas bacteremia is less common. Some studies, however, have suggested that Pseudomonas sepsis is associated with the highest lethality. In several series, 10% to 20% of patients have had polymicrobial series, and most investigators agree that polymicrobial sepsis is more lethal than infection with a single organism.

214 Which of the following statement(s) is/are true concerning the various types of shock?
a. Traumatic shock is more commonly associated with subsequent organ injury and multiorgan failure syndrome than hemorrhagic shock
b. Cardiogenic shock can be of either an intrinsic or compressive nature
c. Hypodynamic septic shock is associated with a decreased mortality risk when compared with hyperdynamic septic shock
d. Hypoadrenal shock usually responds quickly to resuscitation
e. Neurogenic shock occurs with the absence of sympathetic activity
Answer: a, b, d, e

Classification schemes of shock based on cause have been developed for the seemingly dissimilar processes leading to circulatory collapse and the shock state. Hypovolemic shock, the most common, is the result of intravascular volume depletion through loss of red blood cell mass or plasma volume. Microvascular hypotension results from a combination of low intravascular blood volume, diminished cardiac output, and compensatory sympathetic peripheral vasoconstriction. Shock associated with trauma (traumatic shock) arises from the consequences of hypovolemia due to hemorrhage in conjunction with direct soft tissue injury and bone fracture. Hypovolemia caused by blood loss and fluid extravasation into injured tissues is compounded by activation of maladaptive inflammatory cascades initiated by the tissue injury. In contrast to pure hemorragic shock, subsequent organ injury and multiorgan failure syndrome (MOFS) occurs much more frequently following traumatic shock due to the over-expression of these immuno-inflammatory cascades. Cardiogenic shock is the result of failure of the heart as an effective pump, resulting in inadequate cardiac output, tissue perfusion and oxygen delivery. Intrinsic causes include myocardial infarction, cardiomyopathy, valvular heart disease, or rhythm disturbances. Compressive cardiogenic shock is a discrete entity that results when extrinsic compression of the heart limits diastolic filling and thus systolic ejection and cardiac output. Septic shock refers to hypotension and circulatory insufficiency developing as a consequence of infection and the systemic response to that infection. In its hyperdynamic form, septic shock is marked by diminished peripheral vascular resistance and generalized vasodilatation causing relative hypovolemia. In contrast, hypodynamic septic shock occurs in situations of inadequate resuscitation or preterminal cardiovascular decompensation, and is associated with vasoconstriction and a greatly increased mortality risk. Sympathetic denervation through spinal cord injury, spinal anesthesia, or severe head injury produces generalized arterial vasodilatation and venodilation. Shock occurs when the normal blood volume fails to fill the available intravascular space and severe relative hypovolemia exists. Despite hypotension, there is a noteworthy absence of sympathetic activity, as occurs in hypovolemia or cardiogenic shock. Profound shock can occur in surgical patients following stress due to the loss of the homeostatic corticosteroid response. Hemodynamic instability may develop after an operative procedure or coincident with an unrelated illness. The profound circulatory collapse is often refractory to vigorous resuscitation with fluids and pressor agents. The response to exogenous corticosteroids is usually dramatic and potentially life-saving.

215 Which of the following statement(s) is/are true concerning metabolic derangements in sepsis and the systemic inflammatory response syndrome which may follow progressive shock?
a. Alterations in glucose metabolism lead to the development of efficient substrate utilization
b. A progressive rise in serum triglyceride levels result from less efficient clearance and increased hepatic lipogenesis
c. A net negative nitrogen balance occurs due to the oxidative metabolism of proteins to meet energy needs
d. The serum aromatic amino acids fall rapidly as they are actively used in oxidative metabolism
Answer: b, c

A broad spectrum of metabolic abnormalities become apparent in sepsis and the systemic inflammatory response syndrome following shock. Disruption of the normal cycles of carbohydrate, lipid, protein, and oxygen metabolism occur as hypermetabolism develops. Through the Cori cycle, lactate from the periphery is shuttled back to the liver, where it is used in the production of glucose. Because pyruvate is converted to alanine in the periphery, flux of alanine also contributes to hepatic gluconeogenesis. The glycolytic oxidation of glucose to pyruvate and its subsequent glugoneogenic regeneration from lactate is an inefficient cycling of substrate. There is no net energy production, but heat is released in significant quantities. Alterations in lipid metabolism cause a progressive rise in the serum triglyceride level as a result of less efficient clearance of exogenous triglycerides coupled with increased hepatic lipogenesis. Profound alterations in protein and amino acid metabolism develop with characteristic changes in amino acid levels, nitrogen balance, and skeletal muscle mass. Initially levels of the branch chain amino acids are reduced, whereas those of the aromatic amino acids are elevated. There is an increase in the oxidative metabolism of protein to meet energy needs and a tremendous mobilization of nitrogen with net negative nitrogen balance. The branch chain amino acids are preferentially utilized in the TCA cycle to maintain an activity that otherwise would be lost from the diminished entry of carbohydrate-and fatty acid-generated acetyl coenzyme A. This results in reduced serum level of leucine, isoleucine and valine.

216 Which of the following statement(s) is/are true concerning the microvascular and cellular response to shock?
a. Osmodically induced mobilization of intracellular fluid is the initial response to restore intravascular volume
b. With larger volume hemorrhagic shock deterioration of normal cellular transmembrane potential occurs resulting in an increase in extracellular sodium and water
c. The accumulation of anaerobic metabolites override normal homeostatic vasomotor tone and contribute to the maladaptive vasodilatation
d. Abnormal intracellular calcium homeostasis may contribute to the cellular dysfunction of shock
Answer: c, d

Moderate hypovolemia results in a relatively rapid spontaneous restitution of intravascular volume through expansion of the plasma space. This plasma expansion by erythrocyte free fluid occurs within one hour as a result of alterations in pressure and osmolarity and produces an associated hemodilution. Sympathetic discharge, associated arteriolar constriction, and induced metabolic changes in osmolarity initiate the compensatory events at the microcirculatory level. The initial pressure-related phase of restitution of blood volume in shock is overlapped by a second phase involving osmotically induced mobilization of intracellular fluid. Osmotic mechanisms contributing to the restitution of blood volume after moderate hemorrhage are not adequate in hemorrhage of greater magnitude. In larger hemorrhages (over 25% blood volume), there is also deterioration of the normal cellular transmembrane potential, an increase in intracellular sodium and water, and a concomitant decrease in extracellular fluid volume. Tissue hypoxia results, anaerobic metabolites accumulate, and the cell cannot maintain the normal cell membrane potential. Accumulation of hydrogen ion, lactate, and other products of anaerobic metabolism override homeostatic vasomotor tone and contribute to a maladaptive vasodilatation, further augmenting hypotension and hypoperfusion. The uptake of fluid by the “failing” cell is a major source of food sequestration following shock. Loss of membrane function is proportional to both the extent and duration of shock or degrees of sepsis. The etiology of membrane failure is unclear but appears multifactorial. Loss of intracellular ATP energy stores during hypoperfusion or direct toxicity during sepsis may inhibit the membrane sodium-potassium pump. Cellular dysfunction also appears to be related to abnormal intracellular calcium homeostasis.

217 Which of the following statement(s) is/are true concerning the pulmonary response to shock?
a. The acute pulmonary vascular response to shock differs markedly from that of systemic vasculature
b. The pulmonary edema of ARDS occurs in the face of elevated left heart pressures
c. The initial physiologic changes of ARDS involve the capillary endothelial cells and the type I pneumocyte
d. Mechanisms proposed in the pathogenesis of ARDS include injury from mediators of inflammation elsewhere and from activated cellular elements
e. A decrease in lung compliance may result from the loss of type I pneumocytes
Answer: c, d, e

Contributing pathophysiologic processes to the pulmonary manifestations of shock include the pulmonary component of the cardiovascular response, disruption of the normal lung mechanics, and acute lung injury or ARDS due to sepsis. Pulmonary function may be further compromised by pathology intrinsic to the lung itself, including pulmonary contusion, aspiration, airway obstruction, pneumonia, pneumothorax, hemothorax, and atelectasis. The acute pulmonary vascular response to shock largely parallels that of the systemic vasculature. The increase in pulmonary vascular resistance, which may proportionally exceed that of the systemic circulation, transiently accompanies the systemic adrenergic response. ARDS is a syndrome of progressive lung injury that may arise as a direct consequence of shock or other disease processes. The characteristic findings of ARDS are the presence of pulmonary edema, hypoxemia, and significantly decreased lung compliance. The pulmonary edema is noncardiac in origin and occurs in the face of normal left heart pressures. The hypoxemia results from the development of intrapulmonary shunting and perfusion of under and nonventilated alveoli. The decrease in lung compliance results from the loss of surfactant and lung volume in combination with the presence of interstitial fluid and alveolar edema. Progressive histologic changes of ARDS become apparent in pulmonary capillaries, interstitium, and alveoli. Initially, interstitial edema develops with swelling of the capillary endothelial cells and the type I pneumocytes. The type I pneumocytes subsequently slough, and alveolar edema ensues. Functional surfactant is lost with a significant increase in alveoli opening pressure and decrease in alveolar surface tension. Mechanisms proposed in the pathogenesis of ARDS include injury from mediators of inflammation elaborated elsewhere, and from activated cellular blood elements.

218 Which of the following statement(s) is/are true concerning the diagnosis and management of hypovolemic shock?
a. A fall in hematocrit or hemoglobin always accompanies hemorrhagic shock
b. The treatment of shock is generic regardless of the etiology
c. Pharmacologic intervention to increase myocardial contractility in hypovolemic shock is an important part the early management
d. Complications are less frequent after treatment of hemorrhagic shock than septic or traumatic shock
Answer: d

Hypovolemic shock is readily diagnosed when there is an obvious source of volume loss and overt signs of hemodynamic instability and increased adrenergic output are present. After acute hemorrhage, hemoglobin and hematocrit values do not change until compensatory fluid shifts have occurred or exogenous fluid is administered. These values decrease once transcapillary refill, osmotic-induced shifts, or non-RBC volume resuscitation expands the blood volume. It is imperative that the distinction be made between hypovolemic and cardiogenic forms of shock, because appropriate therapy differs dramatically. Restoration of perfusion in hypovolemic shock requires reexpansion of circulating blood volume in conjunction with necessary interventions to control ongoing volume loss. Continued hemodynamic instability after fluid resuscitation implies that shock has not been reversed or that there is ongoing blood or volume loss. In severe, prolonged hypovolemia, ventricular contractile function may itself become depressed and require inotropic support to maintain ventricular performance, but in general, pharmacologic interventions directed toward increased contractility in situations of inadequate preload are ineffective, further complicate metabolic derangements, and are not indicated until adequate volume replacement has been completed. Complications are less frequent after treatment of hemorrhagic shock than in situations of septic or traumatic shock. In the later circumstances, the massive activation of inflammatory mediator response systems and consequences of their disseminated, indiscriminate cellular injury can be quite profound.

219 Which of the following statement(s) is/are true concerning the neuroendocrine responses to shock?
a. Sympathetic nerve endings release epinephrine which is responsible for greater than 80% of systemic vascular resistance
b. Endogenous epinephrine is the primary contributor to systemic vascular resistance
c. Increased pancreatic secretion of glucagon contributes to glucose intolerance associated with injury and sepsis
d. The renin-angiotensin axis further augments the sympathetic-mediated vasoconstriction
Answer: c, d

The neuroendocrine response to shock attempts to achieve restoration of effective blood volume, mobilization of metabolic substrates, and maintenance of central profusion. Both peripheral and central afferent stimuli to the central nervous system are involved in inducing this response. Hypotension, associated with a decrease in impulses from the aortic and carotid baroreceptors, disinhibits the vasomotor center. This disinhibition results in increased adrenergic output and decreased vagal activity. Sympathetic nerve endings release norepinephrine, inducing peripheral and splanchnic vasoconstriction which is responsible for greater than 80% of systemic vascular resistance and is a major contributor to maintenance of central organ perfusion and venous return. Plasma levels of both epinephrine and norepinephrine are elevated with injury, and the degree of the catecholamine elevation corresponds to the magnitude of injury. In shock the effects of endogenous epinephrine are largely metabolic. In addition to initiating autonomic nervous activity, the hypothalamus secretes releasing hormones, which induce the stress hormone release of the pituitary. As part of this response, adrenocorticotropic hormone (ACTH) secretion by the anterior pituitary is increased stimulating cortisol secretion by the adrenal cortex. In conjunction with elevated plasma levels of cortisol and epinephrine, increased pancreatic secretion of glucagon accelerates hepatic gluconeogenesis and further aggravates the glucose intolerance that follows injury and sepsis. The secretion of renin is increased in responses to adrenergic discharge and decreased perfusion of the juxtaglomerular apparatus in the kidney. Renin allows formation of angiotensin I in the liver, which is then converted to angiotensin II in the lungs. Angiotensin II is an extremely effective vasoconstrictor that further augments sympathetic-mediated vasoconstriction.

220 A 22-year-old man sustains a single stab wound to the left chest and presents to the emergency room with hypotension. Which of the following statement(s) is/are true concerning his diagnosis and management?
a. The patient likely is suffering from hypovolemic shock and should respond quickly to fluid resuscitation
b. Beck’s triad will likely be an obvious indication of compressive cardiogenic shock due to pericardial tamponade
c. Echocardiography is the most sensitive noninvasive approach for diagnosis of pericardial tamponade
d. The placement of bilateral chest tubes will likely resolve the problem
Answer: c

Shock from cardiac compression occurs when external pressure on the heart impairs ventricular filling. Because ventricular filling is a function of venous return and myocardial compliance, any process that places pressure on the heart can cause compressive cardiogenic shock. Included among these are pericardial tamponade, tension pneumothorax, mediastinal hematoma, and positive pressure from mechanical ventilation. Any patient with hypotension after a wound in proximity of the heart should be considered to have compressive cardiogenic shock until proven otherwise. The classical clinical findings of pericardial tamponade include Beck’s triad of hypotension, neck vein distention and muffled heart sounds. Pulses paradoxus may be noted (this involves a decrease rather than the normal increase of systolic blood pressure with inspiration; values 10mmHg are significant). These findings, however, may be obscured in a noisy emergency room environment by positive pressure ventilation or by associated injuries. Placement of a CVP catheter confirms the elevation of right-sided filling pressure. If a pulmonary artery catheter has been placed, findings consistent with tamponade or other forms of cardiac compression are a trend toward equalization of chamber pressures as hypotension progresses. In the patient at risk, echocardiography is an extremely sensitive and noninvasive approach to demonstrate pericardial fluid and the need for operation. Pericardial tamponade must be relieved urgently and cardiac injuries require emergent sternotomy. Chest tube placement would not be appropriate as the sole treatment in this patient.

221 A 32-year-old man suffers a spinal cord injury with a resultant paraplegia in a motorcycle accident. He presents to the emergency room with hypotension. Which of the following statement(s) is/are true concerning his diagnosis and management?
a. The low blood pressure can be assumed to be due to neurogenic shock
b. The sole cause of hypotension is the loss of sympathetic input to the venous system
c. Despite significant hypotension, secondary organ injury will be uncommon
d. There is no role for pharmacologic intervention to maintain blood pressure
Answer: c

222 Which of the following statement(s) is/are true concerning septic shock?
a. The clinical picture of gram negative septic shock is specifically different than shock associated with other infectious agents
b. The circulatory derangements of septic shock precede the development of metabolic abnormalities
c. Splanchnic vascular resistance falls in similar fashion to overall systemic vascular resistance
d. Despite normal mechanisms of intrinsic expansion of the circulating blood volume, exogenous volume resuscitation is necessary
Answer: d

The clinical findings in sepsis and septic shock represent the host response to infection. Gram-positive and gram-negative bacteria, viruses, fungi, rickettsiae, and protozoa have all been reported to produce a clinical picture of septic shock, but the overall response is independent of the specific type of invading organism. Septic shock develops as a consequence of the combination metabolic and circulatory derangements accompanying the systemic infection. It appears that the circulatory deficits are preceded by the metabolic abnormalities induced by infection. In fact, the circulatory changes in hyperdynamic sepsis appear to be an adaptive response to the underlying metabolic dysfunction. Cardiac output is high and systemic vascular resistance low in hyperdynamic septic shock. However, splanchnic vasoconstriction is pronounced even in the absence of systemic hypotension and even though systemic vascular resistance is reduced. Expansion of circulating blood volume can occur through either transcapillary refill or fluid resuscitation. Due to the ongoing inflammatory mediator-induced increases in capillary permeability and continued loss of intravascular volume, exogenous volume resuscitation must be provided to restore venous return and ventricular filling.

223 Which of the following statement(s) is/are true concerning tumor necrosis factor (TNF)?
a. TNF is a product of activated macrophages secreted in response to contact with endotoxin or lipopolysaccharide, antibody complexes, or inflammatory stimuli
b. The liver and gut appear to be a major source of TNF following hypoperfusion
c. Circulating levels of TNF correlate well with severity of tissue injury in shock
d. Recently completed clinical trials of anti-TNF antibody in septic patients shows a marked improvement in survival
Answer: a, b

Tumor necrosis factor (TNF), a protein product of activated macrophages, is secreted in response to contact with endotoxin or lipopolysaccharide, antibody complexes, or other inflammatory stimuli. Elevation of serum levels of TNF have been reported shortly after experimental trauma and shock, however, documentation of elevated circulating levels of TNF in human shock is less clear. Furthermore, circulating levels of TNF cannot be correlated with severity of tissue injury or shock. This variability is thought to be due to rapid clearance and uptake by membrane receptors and by soluble membrane receptors that are released from multiple cells following stress and injury. Following hypoperfusion the liver and gut appear to be the major source of TNF that is rapidly cleared but responsible for inducing hepatocyte changes following shock. The release of breakdown products and escape of bacterial and endotoxin through the damaged mucosal barrier of the gut following shock allows or induces activation of tissue-fixed macrophage (Kupffer cell) of the liver which then produces secondary inflammatory mediators contributing to the post-resuscitation clinical response and inflammatory mediator activation seen in the systemic inflammatory response syndrome.
TNF is central to inflammatory response, particularly in sepsis and following endotoxemia or bacteremia. TNF also induces secondary inflammatory responses through direct interaction with specific membrane receptors, TNF-r. Treatment with anti-TNF antibody in the experimental setting protects animals from the deleterious effects of lethal bacteremia and endotoxemia. However, recently completed clinical trials in septic patients utilizing infusion of monoclonal antibodies to the TNF molecule have shown no overall survival benefit.

224 Which of the following statement(s) is/are correct concerning the immunoinflammatory response to shock?
a. The anaphylactoxins, C3a and C5a, are products of activation of only the classical pathway of the compliment cascade
b. Eicosanoids, such as prostaglandins are stored in platelets and endothelial cells and released in response to inflammatory stimuli
c. Thromboxane and PGI2 have similar effects
d. Platelet-activating factor can be released by both circulating and fixed tissue cells
Answer: d

Inflammatory mediators have recently been recognized as playing a significant role in the clinical manifestations and progression of shock and the development of subsequent complications. These mediator systems function primarily as parcrine and autocrine agents in the local environment and are not usually detectable systemically. The over-expression and systemic dissemination of these mediators produces the toxic autodestructive processes underlying multiorgan failure syndrome with attendant high mortality. The compliment cascade is activated in shock and tissue injury through both the classical and alternative pathways. Activation of either pathway results in generation of the anaphylatoxin, C3a and C5a, soluble products with potent systemic hemodynamic effects. The eicosanoids, which include the prostaglandins and leukotrienes are formed acutely from arachidonic acid released from the membrane phospholipid by phospholipase A2. Eicosanoids are not stored in any measurable level and are generated as needed from readily available arachidonic acid in response to various inflammatory phenomena. Platelets, white cells, and endothelial cells are a rich source of these compounds. Thromboxane (TXA2) is the major arachidonic acid metabolite elaborated by platelets. TXA2 induces intense vasoconstriction, platelet aggregation and degranulation, neutrophil margination in the microcirculation and bronchial constriction. PGI2, the major arachidonic acid metabolite formed by endothelial cells, serves a check against actions of TXA2. PGI2 is a vasodilator and a potent inhibitor of platelet aggregation. Platelet aggravating factor is a potent phospholipid mediator released by neutrophils, platelets, macrophages and endothelial cells in response to ischemia, tissue injury and sepsis. Its effects include decreased cardiac function, increased pulmonary vascular resistance, bronchoconstriction, peripheral vasodilatation, and increased vascular permeability.

225 Which of the following physical findings are associated with the various classes of hemorrhagic shock?
a. Mild shock (< 20% blood volume): Pallor, cool extremities, diminished capillary refill and diaphoresis b. Moderate shock (20%–40% blood volume): All of the above plus tachycardia and hypotension c. Severe shock (> 40% blood volume): Systemic hypotension, changes in mental status, tachycardia, oliguria
d. All of the above
Answer: a, c
PHYSICAL FINDINGS IN HEMORRHAGIC SHOCK*

Moderate
Mild (<20% (20%-40% Severe(>40%
Blood Volume) Blood Volume) Blood Volume)
Pallor Pallor Pallor
Cool extremities Cool extremities Cool extremities
Diminished capillary Diminished capillary Diminished capillary
refill refill refill
Diaphoresis Diaphoresis Diaphoresis
Collapsed Collapsed Collapsed
subcutaneous subcutaneous subcutaneous
veins veins veins
Tachycardia Tachycardia
Oliguria Oliguria
Postural Hypotension
hypotension Mental status
changes

226 A 68-year-old male who underwent a repair of an abdominal aortic aneurysm 5 days ago, develops tachycardia, tachypnea, hypotension with cool, pale, mottled cyanotic extremities. He is agitated and complains of shortness of breath. Which of the following statement(s) is/ are correct concerning his diagnosis and management?
a. Myocardial ischemia secondary to preexisting coronary artery disease is most likely the underlying cause of this problem
b. Invasive hemodynamic monitoring with a Swan-Gantz catheter will demonstrate a low cardiac output, a high systemic vascular resistance, and elevated cardiac filling pressures
c. The use of morphine sulphate and nitrates should be part of the initial management
d. The primary pharmacologic treatment involves the use of moderate doses of inotropic agents
e. Afterload reduction with nitroprusside is absolutely contraindicated
Answer: a, b, d

227 Which of the following statement(s) is/are true concerning ischemia reperfusion injury?
a. During ischemia, ATP degradation results in increased plasma and intracellular levels of hypoxanthine and xanthene
b. Oxygen free radicals such as the superoxide radical are involved in the expression of the proinflammatory phenotype of endothelial cells, macrophages and neutrophils
c. The intracellular adhesion molecule-1 (ICAN-1) contributes to injury and disruption of the endothelial lining, with extensive capillary leak and resultant interstitial edema
d. Animal models have demonstrated that passive immunization with antibodies to neutrophil adhesive complex lessen the ischemic/reperfusion microvascular injury
Answer: a, b, c, d

During the ischemia and hypoperfusion phase, degradation of ATP stores essential to maintain cell integrity and significant loss of diffusible intracellular adenine neuclotides occurs. As ATP further degrades there is an elevation in plasma and intracellular levels of hypoxanthine and xanthene which upon restoration of perfusion and reoxygenation are catalyzed by xanthine oxidase resulting in the formation of superoxide radicals. These radicals plus others such as hydrogen peroxide and hydroxyl radical are generated and lead to endothelial and parenchymal cell injury through membrane lipid peroxidation and activation of critical enzymes. These radicals have also been shown to be involved in the expression of proinflammatory phenotype endothelial cells and on macraphages and neutrophils. The proinflammatory phenotype of the endothelium includes procoagulant activity and the expression of adhesion molecules on the membrane surface, including the intercellular adhesion molecule-1 (ICAN-1) and the selectins. The subsequent adhesion of activated neutrophils to the endothelial leads to an explosive oxidative burst producing additional radicals and extensive release of proteolytic enzymes leading to injury and disruption of the endothelial lining, extensive capillary leak, and massive interstitial edema. Passive immunization of animals with monoclonal antibodies to either the neutrophil adhesive complex or the endothelial selectins dramatically lessens ischemia/reperfusion microvascular injury.

228 Which of the following statement(s) is/are true concerning the physiology of the microvascular system?
a. Filtration of capillary fluid into the interstitial and the subsequent reabsorption is influenced by Starling’s law of ultrafiltration
b. The most important variable controlling blood to a capillary bed is the length of the vessel
c. Most of the resistance to systemic blood flow occurs at the arteriolar level
d. Adrenergic vasoconstriction can arrest blood flow to an entire capillary bed
Answer: a, c, d

Exchange of material between the vascular space and the cell of various tissues via the interstitial space is essential for organ viability and occurs at the capillary level. The filtration of capillary fluid into the interstitium and its subsequent reabsorption into the post capillary venule is governed by microvascular permeability in conjunction with the balance between hydrostatic and oncotic pressures. The relation of these forces to one another (and their net effects) are illustrated by what is termed Starling’s law of ultrafiltration. In normal circumstances, a net filtration from capillary to interstitium is effected by a relatively higher capillary hydrostatic pressure, whereas net reabsorption from the interstitium back into the post capillary venule occurs as hydrostatic pressure falls and oncotic forces predominate. Although the mechanisms controlling blood flow to the capillary bed are complicated and vary among the different tissues, certain concepts are useful. Poiseuille’s law describes the relation between flow of fluid through a tube and the tube length and radius, the fluid viscosity, and the pressure gradient between ends of the tube. The radius of the tube (or vessel) is the single most important variable, because flow is proportional to the radius to the fourth power. Vasoconstrictive and vasodilatory influences directly impact local blood flow, as well as flow to other tissues through secondary effects on the systemic pressure. This secondary effect of peripheral vasoconstriction maintains the pressure gradient for central perfusion of the heart and brain. Systemic blood flow meets most of its resistance at the arteriolar level. While the individual capillary radius is significantly smaller, the vast number of capillaries offers less total resistance. The vascular smooth muscle in arterioles has both a-and b- adrenergic receptors. Alpha stimulation affects vasoconstriction where beta stimulation affects vasodilatation. The efferent sympathetic fibers innervating the precapillary resistance vessels and the venous capacitance vessels release norepinephrine on stimulation, which induces smooth muscle contraction and narrowing of the caliber of the vessels. These contractions are potent enough that blood flow to entire capillary beds can be arrested by adrenergic vasoconstriction.

229 Which of the following statement(s) is/are true concerning the effects of MOFS?
a. Pulmonary dysfunction tends to arise early and may resolve within 7 to 10 days
b. Unless the precipitating insult has prompted oliguric acute tubular necrosis, renal function tends to be maintained early in the course of MOFS
c. Although hepatic dysfunction is common with MOFS, the GI tract plays little role in this process
d. Intercurrent nosocomial infection, most commonly pulmonary, is a common complication providing a “second hit” to the patient
Answer: a, b, d

Pulmonary dysfunction typically arises early in the development of systemic inflammation and may represent mild relatively localized acute lung injury or it may be a prelude to fulminant ARDS. The lung injury, and associated dysfunction, may resolve over the initial 7 to 10 days or persist, depending on the ongoing pathologic process. Many times a “second hit” such as a nosocomial infection, which is most commonly pulmonary, is a complication which can frequently worsen the pulmonary condition. Renal function tends to be maintained early in the course unless the precipitating insult has been prompted by a sudden oliguric acute tubular necrosis. With persistent activation and inflammatory mediators, glomerular filtration falls and the development of oliguric or polyuric renal failure marks the gradual transition into MOFS. Gastrointestinal abnormalities include ileus, stress ulceration, diarrhea, and mucosal atrophy. Breakdown of the mucosal barrier allows translocation of bacteria and endotoxin. Hepatic dysfunction is marked by progressive rise in serum bilirubin levels after a latent period of several days.

230 Which of the following statement(s) is/are true concerning hypoadrenal shock?
a. In the United States, idiopathic adrenal atrophy (Addison’s disease) is the most common cause
b. Laboratory abnormalities include hyponatremia, hypochloremia, and hyperkalemia
c. Fever may be seen with hypoadrenal shock
d. ACTH stimulation test is the diagnostic test of choice to confirm hypoadrenal shock
Answer: b, c, d

Shock of a dramatic nature, poorly responsive to resuscitation, may develop as a consequence of adrenal insufficiency. In this country, adrenal insufficiency most commonly arises as a consequence of the chronic therapeutic administration of high doses exogenous corticosteroids causing adrenal suppression. Other causes include idiopathic adrenal atrophy (Addison’s disease), tuberculosis, metastatic disease, bilateral hemorrhage, and amyloidosis. The stress of illness, operation, or trauma typically requires that the adrenal glands secrete cortisol in excess of that required in the nonstressed state (approximately 3–4 fold). Insufficiency not otherwise apparent may manifest itself only after major physical stress. Findings associated with adrenal insufficiency include weakness, fatigue, anorexia, abdominal pain, nausea, vomiting, and weight loss. Surgical patients with significant adrenal insufficiency need not present with the above findings. More typical is the development of refractory shock, frequently with hyperthermia, in the course of injury or illness. Hypotension may be dramatic despite massive volume resuscitation and pressor support. Laboratory findings suggesting hypoadrenalism include hyponatremia, hypochloremia and hyperkalemia. The diagnosis of adrenal insufficiency may be confirmed or excluded by means of an ACTH stimulation test. A significant major cortisol response should be elicited by ACTH administration.

231 Which of the following statement(s) is/are correct concerning the cardiovascular response to shock?
a. Changes in cardiac contractile function shift the Frank Starling curve up and down
b. Venoconstriction from skeletal muscle is a significant contributor to the restoration of blood volume with shock
c. Arterial vasoconstriction affects all vascular beds equally
d. The total circulating blood volume is equally split between the arterial and venous system
Answer: a

Central in the general cardiovascular response to shock is the action of the heart itself. The principle determinants of cardiac function in the normal heart are the volume of blood available for the heart to pump (preload), the systolic contractile capability, and the diastolic filling of the ventricles. In hypovolemia, the two dynamic variables of cardiac function, ventricular filling and myocardial contractility remain paramount and determine the stroke volume. The product of stroke volume and heart rate in turn determines the cardiac output. Increases in ventricular end-diastolic volume, reflecting venous return, cause ventricular distention. Ventricular distention in turn produces increased volume output with each stroke, the Frank Starling mechanism. Contractile function may vary independent of volume status. Changes in the contractile function shift the Starling curve up and down, producing increases or decreases in stroke volume for any given end-diastolic volume. A fundamental requirement for cardiovascular function is adequate cardiac filling, and cardiac output cannot exceed venous return. The venous system contains nearly two-thirds of the total circulating blood volume, including 20% to 30% within the splanchnic venous system. Most of this volume resides in small veins, which comprise the bulk of venous capacitance. The venous system, especially that of the splanchnic circulation, becomes important in the physiologic compensation to hypoperfusion because it serves as a dynamic reservoir for the autoinfusion of blood volume involving both active and passive mechanisms. The splanchnic circulation makes major contributions to the maintenance of venous return, therefore, it is likely that sympathetic venoconstriction is responsible for a portion of the blood mobilized from the splanchnic venous circulation. Sympathetic mediated venoconstriction in skin and skeletal muscle is probably not as significant as a source of blood volume. Selective vasoconstriction occurs in response to alpha adrenergic receptor stimulation with increased sympathetic activity in shock. Sympathetic stimulation does not cause significant vasoconstriction of either cerebral or coronary vessels, with normal blood flow maintained in these circulations. Blood flow to the skin is sacrificed early, followed by that to the kidneys and splanchnic viscera.

232 Which of the following statement(s) is/are true concerning pharmacologic agents used in the treatment of shock?
a. The primary difference between dopamine and dobutamine is the absence of significant a adrenergic activity
b. The renal and mesenteric vasoconstrictive effects of norepinephrine complicate and sometimes restrict its use
c. The apparent paradoxical use of vasodilators, such as nitroprusside, in shock is indicated as a means to augment cardiac function
d. Isoproterenol with its potent b-adrenergic effect, is a particularly useful agent in the treatment of all forms of shock
Answer: a, b, c

Therapeutic adjustments of intravascular volume (preload) and systemic vascular resistance (afterload) form the basis of the treatment strategies for all forms of shock. Optimal volume resuscitation should precede measures to augment to contractile function of the heart. Inotropic agents are used in shock when there is inadequate cardiac output despite adequate circulating blood volume. Dopamine and dobutamine are often times first line agents in the pharmacologic treatment of shock. Dopamine, at low doses, stimulates dopaminergic receptors producing renal arteriolar vasodilatation with associated increases in renal blood flow, urine output, and sodium excretion. At moderate doses, stimulation of cardiac b-receptors produces increases in contractility and cardiac output with little effect on heart rate or blood pressure. At higher doses, peripheral vasoconstriction from increasing a activity becomes more pronounced, prompting significant increases in vascular resistance and blood pressure. Dobutamine’s predominant effect is an increasing cardiac contractility with lesser increases in heart rate. Some reduction of peripheral vascular resistance may also occur. When compared to dopamine, dobutamine produces less peripheral vasoconstriction and less chronotropic response. Norepinephrine exerts both a and b-adrenergic effects, with a effects being evident at lower infusion rates and a effects more prominent at high doses. The major use of norepinephrine in current practice is in the patient with hypotension that persists despite appropriate volume resuscitation and the use of inotropic agents. Renal and mesenteric vasoconstrictive effects of norephinepherine complicate its use, especially when support is needed for significant periods of time. Isoproterenol is a potent b-adrenergic agent. With isoproterenol, myocardial oxygen demand is increased and diastolic coronary feeling is limited by tachycardia or diminished diastolic pressure. Indications for the use of isoproterenol are fairly limited, because agents with fewer adverse effects have become available.
Vasodilators are used to augment cardiac function through optimization of ventricular filling pressures (preload) and systemic vascular resistance (afterload) both of which reduce demands on the myocardium. Decreases in afterload prompt increases in cardiac output and venodilatation contributes to decreases in pulmonary venous pressure and central venous pressure. Hypotension, however, may develop therefore patients must have careful constant monitoring of arterial pressure and repeated hemodynamic measurements with a pulmonary artery catheter.

233 Which of the following statement(s) is/are true concerning the treatment of MOFS?
a. Prevention and therapy of MOFS requires control of the infectious or inflammatory source
b. Restoration of normal clinical parameters such as blood pressure, pulse rate, and urine output ensures optimal resuscitation in most patients
c. Branch chain amino acids play and important role in the nutritional support of the patient
d. Because of the nature of gut injury, total parenteral nutrition is preferred for most patients with MOFS
Answer: a, c

The therapy of MOFS is directed towards interrupting the involving pathophysiologic process and providing an optimal physiologic environment for healing and recovery. Fundamental concerns are control of the source of infection, inflammation or instability; restoration of microcirculatory blood flow and oxygen transfer, and the institution of optimal supportive care. Both the prevention and therapy of MOFS, therefore, requires source control and restoration of adequate profusion. Resuscitation efforts are directed toward restoration of adequate microcirculatory blood flow in all organ systems. Restoration of normal clinical parameters such as blood pressure, pulse rate, urine output, and acid-base balance does not ensure optimal resuscitation. The physiologic endpoint that most closely corresponds with adequate microcirculatory flow is the level of cardiac output and the oxygen delivery at which oxygen consumption and lactate production remain independent of flow.
The importance of metabolic support in the patient with MOFS cannot be overemphasized. The malnutrition of MOFS is markedly different than that of starvation and the nutritional requirements also differ. If optimal quantities of appropriately formulated amino acid solutions are given, protein synthetic rates can approach catabolic rates and the goal of nitrogen balance can be achieved. Formulas rich in branch chain amino acids appear to be more efficient in promoting nitrogen retention and minimizing urea production. Whenever feasible, enteral feeding is preferred over TPN because evidence suggests that bacterial translocation from the gut can be limited through the use of enteral feeds. Enteral absorption and processing of nutrients appears superior to TPN and lessens overall complications.

234 Which of the following statement(s) is/are true concerning the multiorgan failure syndrome (MOFS)?
a. Changes in the splanchnic and pulmonary microcirculation are critical to the development of MOFS
b. Tissue fixed microphages, including the liver Kupffer cell, have little role in the development of MOFS
c. MOFS represents systemic consequences of loss of homeostatic control of local inflammation and microcirculatory hypoperfusion
d. The nature of MOFS is highly dependent upon the etiology of the underlying problem
Answer: a, c

The nature of multiorgan failure syndrome (MOFS) is that of a diffuse cellular injury, developing systemically as a consequence of losing homeostatic control of local inflammation and microcirculatory hypoperfusion. Endothelial injury, platelet aggregation and activation of macrophages and neutrophils occur, and the clotting, fibrinolytic, kinin, and complement cascades are activated, along with the release of potent inflammatory cytokines. The effects of shock, resuscitation, and reperfusion, and the subsequent development of MOFS appear to be critically dependent on changes in the splanchnic and pulmonary microcirculations. These vascular beds appear to be major sites of activation of subsequent inflammatory mediator production that underlies the diffuse systemic inflammatory response. Extensive activation of the liver Kupffer cell and release of inflammatory mediators coupled with the ongoing release of activated neutrophils and by-products of activated gut macrophages is responsible for the injury to the pulmonary microcirculation and secondary induction of alveolar macrophage and additional inflammatory mediator systems. Excessive and persistent macrophage activation plays an essential role in MOFS and is hypothesized to represent the penultimate step in a series of continuous immuno-inflammatory stimulatory events, including local hypoxia, exposure to bacteria and toxins, and mediator release from localized areas of inflammation. When infection is the underlying or major contributing process, the diffuse inflammatory response develops independently of the specific type of microorganism. In noninfectious cases, the response also appears independent of the specific underlying cause.

235 Invasive hemodynamic monitoring using a Swan-Gantz catheter is essential in the optimal management of patients in shock or those suffering post-shock sequelae. Which of the following physiologic characteristics are associated with the various forms of shock?
a. Hypovolemic shock: Decreased pulmonary capillary wedge pressure (PCWP), decreased cardiac output, increased systemic vascular resistance (SVR)
b. Cardiogenic shock: Increased PCWP, decreased cardiac output, decreased SVR
c. Septic shock (hypodynamic): Decreased cardiac output, increased SVR
d. Neurogenic shock: Decreased PCWP, increased cardiac output, decreased SVR
Answer: a, c

236 Which of the following statement(s) is/are true concerning the relationship between cardiac function and effective blood volume?

a. A pulmonary capillary wedge pressure of 5–10 rules out fluid overload as a cause of pulmonary edema
b. A shift to the right in the Frank-Starling curve is associated with compromised cardiac function
c. Dilutional anemia may contribute to tachycardia even though blood volume and filling pressures are normal
d. The sole purpose of a pulmonary artery catheter is to measure pulmonary artery pressure and cardiac output
Answer: b, c

Although physical findings are often adequate to establish a diagnosis and institute management of cardiac failure, direct measurement of filling pressures of the right heart (central venous pressure) or the left heart (pulmonary artery pressure) may be required. Placement of a pulmonary artery catheter allows us to measure cardiac output by thermodilution and, more importantly, to sample mixed venous blood for saturation measurements which tell us the ratio between systemic oxygen delivery and oxygen consumption. From all of these measurements we can determine if cardiac output is normal for the level of filling pressure of the left ventricle, or if contractility is decreased. In the latter case, cardiac output will be lower than predicted for a given level of filling pressure. In the Frank-Starling curve, if the patient is to the right of the normal range, then cardiac function is compromised either because of valvular disease, extrinsic pressure such as pericardial tamponade, or more commonly, a decrease in contractility. If cardiac function and anatomy are normal, then blood volume, filling pressure and cardiac function are related to the Starling curve. The intake and output of fluid and salt is autoregulated to maintain the filling pressure of the left ventricle around 10 mm Hg. Extracellular fluid expansion is usually associated with normal blood volume. Gross expansion of extracelluar space results in deleterious effects if tissue edema can and often do exist with perfectly normal blood volume. In other words, a pulmonary capillary wedge pressure of 5–10 does not rule out fluid overload as a cause of pulmonary or GI dysfunction. In critically ill patients, the fear of hypotension and effect of perfusion usually results in infusion of intravenous salt and water in quantities which exceed losses. Consequently, most patients in the Intensive Care Unit have anemia, dilutional hypoproteinemia, and a compensatory increase in cardiac output. In response to anemia, these patients are tachycardic, even though blood volume is normal, filling pressures are normal, and total body extracellular fluid is excessive.

237 Which of the following statement(s) is/are true concerning methods of nutritional support?

a. Optimal results for enteral feedings are achieved with approximately half of calories supplied as carbohydrate and half as fat
b. Diarrhea is the most common complication of enteral feeds and is due to the high osmolarity of the carbohydrate components
c. The hyperosmolar nature of parenteral fat solutions requires central venous administration
d. Approximately 25–50% of calories should be provided as fat emulsion in patients receiving total parenteral nutrition
Answer: a, b, d

Most formulas for enteral feeding range from 1.0 to 2 cal/ml and include 3 to 7% protein. Most of the calories are supplied as glucose or sucrose, so that the solutions have a very high osmolarity. Cramps or diarrhea can result when these high osmolar solutions are placed into the stomach or intestine. Diarrhea is the major complication with most tube feeding formulas. Diarrhea can be minimized by the use of starch or fat as an energy source in tube feedings. This can be supplied as part of the commercial preparation or added in the form of medium chain triglycerides or other oils. The best results are usually achieved by supplying approximately half the calories as carbohydrate and half as fat. In patients receiving total parenteral nutrition, energy source is provided as carbohydrate, fat, and amino acid solutions. Parenteral feeding with carbohydrate is limited by the sclerotic effect of hyperosmolar solutions on veins. Fat is a more efficient energy source and can be given through peripheral veins in concentrations of either 10 or 20%. Most intensivists favor supplementing standard total parenteral nutrition solution with intravenous fat to provide at least 100 grams of fat emulsion each week to preclude fatty acid deficiency. Giving up to 25 to 50% of calories each day as fat emulsion may optimize the delivery of this caloric delivery.

238 Which of the following statement(s) is/are true concerning the autoregulation necessary to maintain oxygen consumption and oxygen delivery?

a. A change in oxygen consumption is followed by a proportionate change in oxygen delivery
b. A change in oxygen delivery is followed by a change in oxygen consumption
c. Increases in oxygen delivery are due solely to an increase in cardiac output
d. The normal ratio of oxygen delivery to consumption is 2:1
Answer: a

The relationships between oxygen consumption and oxygen delivery represent one of the most interesting regulation systems in homeostasis. First of all, if one of the three components of oxygen delivery is abnormal, endogenous mechanisms regulate the other two until normal oxygen delivery has been restored. The various combinations of compensatory mechanisms supply adequate oxygen for systemic metabolism through a wide range of variations in oxygen delivery. When there is a change in oxygen consumption, there is a proportionate change in oxygen delivery, which occurs almost immediately, mediated completely by a change in cardiac output. Conversely, a primary change in oxygen delivery is not followed by any change in oxygen consumption. The normal ratio of oxygen delivery to consumption is approximately 5:1.

239 Which of the following statement(s) is/are true concerning O2 venous saturation monitoring?

a. The normal saturation of mixed venous blood is 50%
b. Mixed venous blood obtained for saturation monitoring can be obtained from any peripheral vein
c. If arterial blood is fully saturated, the saturation of mixed venous blood is 80%
d. In less than fully saturated blood, the difference between arterial and venous saturation corresponds to oxygen extraction
Answer: c, d

240 Which of the following statement(s) is/are true concerning oxygen kinetics in a critically ill, febrile patient?

a. Oxygen consumption will likely exceed three times normal
b. The high cardiac output and pulse rate are designed to increase oxygen delivery
c. The hyperdynamic response may actually increase oxygen delivery to exceed the increase in oxygen consumption
d. The patient can maintain adequate compensation as long as the oxygen delivery/oxygen consumption rate is greater than 2:1
Answer: b, d

241 Which of the following statement(s) is/are true concerning the treatment of pulmonary interstitial edema?

a. Diuresis and blood transfusion is a valuable step
b. Salt-poor albumin leaks through the capillaries and worsens the condition
c. Mannitol is contraindicated as a diuretic in this clinical situation
d. Isoproterenol is a poor choice as an ionotropic agent
Answer: a

242 Which of the following statement(s) is/are true concerning the pathophysiology of gas exchange?

a. Hypoventilation in relation to perfusion can result in an oxygen saturation of less than 100%
b. Diffusion block and / mismatch can almost completely be overcome by breathing 100% oxygen
c. Transpulmonary shunting does not occur under normal circumstances
d. The normal arterial oxygen saturation should be 100%
Answer: a, b

243 Which of the following statement(s) is/are true concerning CO2 transfer in the lung?

a. Carbon dioxide excretion is a direct function of alveolar ventilation
b. Normally end tidal CO2 should be identical to PaCO2
c. The gradient between end tidal and arterial CO2 can be an indirect measure of nonperfused alveoli
d. Positive pressure ventilation under normal airway pressures creates a significant end tidal PaCO2 gradient
Answer: a, b, c

244 Which of the following statement(s) is/are true concerning the assessment of protein reserve?

a. Conventional serum proteins such as albumin and globulin are early indicators of malnutrition
b. The total lymphocyte count reflects immune status and not nutrition
c. Antigen skin testing reflects patient immunity and not nutrition
d. Measurement of urea excretion in urine can be used as a measurement of protein breakdown
Answer: d

245 Useful steps to optimize systemic oxygen delivery include:

a. Maintaining mean arterial blood pressure between 50 and 90 mm Hg
b. Optimizing PEEP levels by monitoring mixed venous saturation
c. Turning the patient prone
d. Sedation or paralysis
Answer: a, b, c, d

246 Phases of multiorgan failure will include:

a. Generalized increased capillary permeability
b. A hypermetabolic state
c. Organ malfunction
d. All of the above
Answer: d

247 Which of the following statement(s) is/are true concerning oxygen consumption (O2)?

a. O2 is normally 100–120 cc2/m2/min
b. Resting O2 is controlled by the level of thyroid and catecholamine hormones
c. Under steady state conditions the amount of oxygen consumed exceeds the amount of oxygen taken up by the pulmonary capillaries
d. O2 is dependent on the status of pulmonary function
Answer: a, b

248 Which of the following statement(s) is/are true concerning the outcome in patients with acute renal failure?

a. Mortality for ischemic acute tubular necrosis without other organ failure is approximately 6%
b. Multiple organ failure complicated with acute renal failure is associated with mortality ranging from 50% to 90%
c. Recovery of renal function after six weeks is unlikely
d. There is no difference in survival between oliguric and nonoliguric renal failure
Answer: a, b, c

249 Which of the following statement(s) is/are true concerning oxygen delivery?

a. The amount of oxygen delivered to peripheral tissues is dependent upon the oxygen content in arterial blood and cardiac output
b. Oxygen content is commonly measured in arterial blood
c. The normal arterial-venous difference is 4 cc O2/dL
d. Normal systemic oxygen delivery for a typical adult is approximately 1000 cc/min
Answer: a, c, d

250 Which of the following statement(s) is/are true concerning carbon dioxide kinetics?

a. The amount of carbon dioxide produced is equivalent to the amount of oxygen consumed
b. Carbon dioxide levels in blood, present mostly as a bicarbonate ion, can quickly change
c. Normally the amount of carbon dioxide excreted through the lung is exactly equal to the amount of carbon dioxide produced in peripheral tissues
d. The amount of carbon dioxide excreted is a function of ventilation of perfused alveoli
Answer: a, c, d

251 Which of the following result in a decrease in functional residual capacity?

a. Shallow breathing
b. Partial airway occlusion
c. Absorption atelectasis
d. Hemothorax
Answer: a, b, c, d

252 Which of the following statement(s) meet the criteria for organ failure?

a. Bilirubin greater than 5 mg/dl
b. Creatinine greater than 3 mg/dl
c. Alveoloarterial O2 gradient greater than 300 mm Hg
d. Glasgow Coma score less than 10
Answer: a, b, c, d

253 Which of the following statement(s) is/are true concerning pulmonary edema?

a. Pulmonary edema effectively narrows bronchi and increases pulmonary vascular resistance
b. Ventilation and perfusion are decreased equally
c. Positive pressure ventilation improves gas exchange by decreasing lung edema
d. The condition is frequently caused by decreased plasma protein levels
Answer: a

254 Which of the following statement(s) is/are true concerning various causes of acute renal failure?

a. Acute tubular necrosis is the most common pathologic finding of acute renal failure
b. induced renal failure is compounded in situations of hypovolemia
c. Myoglobin-induced renal failure can be prevented using diuretics and alkalization of urine
d. The incidence of radiographic contrast dye-induced renal failure occurs independent of preexisting conditions
Myoglobin is a direct nephrotoxin
Answer: a, b, c

255 The patient requires renal replacement therapy. Which of the following statement(s) is/are true concerning the differences between hemodialysis and continuous arteriovenous hemodialysis (CAVHD)?

a. Anticoagulation is not required for CAVHD
b. Hemodynamic instability will be a particular problem with both techniques
c. Both techniques will decrease serum urea ni+62trogen levels
d. CAVHD will likely result in better removal of excessive volume
Answer: c, d

256 Which of the following statement(s) is/are true concerning continuous arteriovenous hemofiltration (CAVH)?

a. The technique runs continuously
b. It is not associated with the hemodynamic instability
c. Systemic heparin anticoagulation is necessary
d. Fluid balance and correcting electrolyte abnormalities takes several days
Answer: a, b

257 A 64-year-old diabetic patient develops acute renal failure following an aortic aneurysm repair. Which of the following statement(s) is/are true concerning his diagnosis and management?

a. Resting energy expenditure will likely be less than would be expected for a patient with normal renal function
b. Maintenance of positive energy balance reduces protein catabolism and makes the management of renal failure easier
c. Expected metabolic abnormalities include hyperkalemia, hypercalcemia, and metabolic alkalosis
d. A nonoliguric renal failure is usually associated with a better outcome
Answer: b, d

258 Which of the following statement(s) is/are true concerning various energy sources?

a. Carbohydrate is the most efficient source of energy
b. Endogenous fat is the major source of energy during starvation
c. The respiratory quotient of carbohydrate is greater than either fat or protein
d. Ketones can be used as a source of energy during starvation
Answer: b, c, d

259 Which of the following statement(s) is/are true concerning the treatment of multisystem organ failure?

a. Forced diuresis with negative fluid balance may improve survival and acute respiratory failure
c. Nutritional support should be withheld for several days until the patient’s condition stabilizes
d. Continuous arteriovenous hemofiltration is preferred to intermittent hemodialysis for most critically ill patients
e. Hepatic failure should be treated specifically with pharmacologic manipulation
Answer: a, d

260 Which of the following statement(s) is/are true concerning pulmonary mechanics?

a. The standard compliance or volume pressure curve is measured during lung inflation
b. The decreased compliance in acute respiratory failure occurs because the lung is smaller not stiffer
c. In acute respiratory failure, higher pressures are required to achieve the same level of inflation
d. Areas of normal lungs are more vulnerable to overdistention which may lead to progressive lung dysfunction
Answer: b, c, d

261 Which of the following statement(s) is/are true concerning the use of a ventilator in the treatment of respiratory failure?

a. The assist-control mode is appropriate in the paralyzed patient
b. Peak inspiratory pressure should be optimized at a level in excess of 40 cm H2O
c. A patient receiving excessive carbohydrate as a nutritional support may have an elevated minute ventilation and may tire with spontaneous breathing
d. In general, weaning requires an adequate inspiratory force, vital capacity, and a minute ventilation less than 10 L/min
Answer: c, d

262 Which of the following statement(s) is/are true concerning the estimation and measurement of energy requirements in the critically ill patient?

a. One can only estimate energy expenditure with actual measurement not technically possible
b. The amount of oxygen absorbed through the lungs is equal to the amount of oxygen consumed by metabolic processes
c. Metabolic rate, normalized to body surface area, may underestimate metabolism in a fat person
d. To convert cc’s of oxygen per minute to calories per day, a conversion factor of 10 kcal of energy per liter of oxygen should be used
Answer: b, c

263 Which of the following statement(s) is/are true concerning the response to a decrease in functional residual capacity percent (FRC)?

a. Supplying supplemental oxygen will always improve the situation
b. Respiratory alkalosis may occur
c. Decreasing compliance is a common occurrence
d. Respiratory rate and depth of breathing generally decrease
Answer: b, c

264 Which of the following statement(s) is/are correct concerning the body fluid compartments?

a. Both the extracellular and intracellular components of total body water can be directly measured
b. The intravascular space accounts for the majority of extracellular fluid
c. All water in the interstitial space is freely exchangeable
d. Transcellular fluid, separated from other compartments by both endothelial and epithelial barriers, constitute about 4% of total body water
Answer: d

265 Which of the following statements(s) is/are true concerning metabolic alkalosis?

a. Either increased extracellular bicarbonate concentration or inhibited renal excretion of bicarbonate can cause metabolic alkalosis
b. In metabolic alkalosis secondary to prolonged gastric outlet obstruction, the urine pH is usually acidic
c. Hypokalemia can lead to metabolic alkalosis
d. The respiration compensatory mechanisms for metabolic alkalosis are quite ineffective
Answer: b, c, d

266 Which of the following statement(s) is/are true concerning respiratory acidosis?

a. Respiratory acidosis is associated with chronic pulmonary disease far more commonly than is hypoxemia
b. The initial buffering effect occurs at the cellular level
c. Renal compensation occurs within 24 hours
d. Correction of hypoxemia in patients with chronic lung disease may worsen respiratory acidosis
Answer: b, d

267 Which of the following(s) is/are true concerning the control of the volume of body water?

a. Osmoreceptors and baroreceptors work equally to control fluid balance during normal conditions
b. The cardiac atrium regulates volume only by means of its sympathetic and parasympathetic connections
c. The kidney is the primary effector organ in controlling water balance
d. The conversion of angiotensin I to angiotensin II is dependent on the amount of the enzyme, renin, available
e. Nitric oxide plays a number of important roles in regulation of renal hemodynamics
Answer: c, d, e

268 Which of the following statement(s) is/are true concerning maintenance intravenous fluid therapy?

a. The total daily water requirement for a 70-kg man is about 2500 mL/day
b. Normal maintenance IV therapy requires administration of sodium, potassium, calcium, phosphate, and magnesium
c. Fluid volume calculations for elderly patients generally are decreased compared to their younger counterparts
d. A child requires a lesser amount of maintenance fluid per kilogram than a larger individual
Answer: a, c

269 Which of the following statement(s) is/are true concerning abnormalities in calcium concentration?

a. Parathyroid hormone affects calcium homeostasis only at the exchange of calcium between bone and extracellular fluid
b. About 45% of total plasma calcium is in the ionized state and is responsible for most physiologic actions
c. Changes in plasma protein levels or pH can alter the proportion of calcium in the ionized state
d. Intravenous normal saline administration is the first step in treatment of hypercalcemia
e. Classic signs of hypocalcemia include hyperactive deep tendon refluxes, Cvostek’s sign and Trousseau’s sign
Answer: b, c, d, e

270 Which of the following statement(s) is/are true concerning total body water?

a. Total body water in men represents a higher percent body weight than in women
b. In infants, water comprises up to 80% of body weight
c. Total body water content decreases with increasing age
d. Total body water is equally distributed within the intra-and extracellular compartments
Answer: a, b, c

271 Which of the following statement(s) is/are true concerning the clinical presentation and treatment of severe metabolic alkalosis?

a. In most cases clinical signs are obvious
b. Correction of potassium and volume depletion corrects most cases of metabolic alkalosis
c. Acetazolamide can enhance renal excretion of bicarbonate
d. Acid replacement should be provided at a molar equivalent basis for excess serum bicarbonate
Answer: b, c

272 Which of the following statement(s) is/are true concerning abnormalities in serum sodium?

a. The most common cause of hyponatremia is a deficit in total body sodium
b. Hyponatremia can occur in situations of excessive solute
c. Most surgical patients with hyponatremia are best treated by free water restriction
d. Central nervous system effects are the predominant symptom of hypernatremia
e. Hypernatremia should be rapidly corrected with free water administration
Answer: b, c, d

273 Which of the following statement(s) is/are true concerning abnormalities in serum potassium?

a. Hyperkalemia can occur in the otherwise normal surgical patient due to excessive intravenous potassium administration
b. The primary EKG change associated with severe hyperkalemia is peaked T-waves
c. Temporary treatment of hyperkalemia includes administration of calcium, sodium bicarbonate, or glucose and insulin
d. Alterations in membrane potentials reflected in cardiac and skeletal muscle are common results of both hypo-and hyperkalemia
e. A reduction in serum potassium of 1mEq/liter requires replacement of 40mEq of potassium
Answer: c, d

274 Which of the following statement(s) is/are true concerning the derangement of metabolic acidosis?

a. A major source of acid production of the body is sulfuric acid
b. Excessive loss of bicarbonate can occur with intestinal or pancreatic fistulas
c. Ketoacidosis can occur in conditions of either hyper-or hypoglycemia
d. Lactic acidosis is present when serum lactate concentration is > 2 mEq/L
e. Lactic acidosis can be associated with ethanol toxicity
Answer: a, b, c, e

275 Which of the following statement(s) is/are true concerning the osmotic activity of body fluids?

a. Urea contributes to the osmolality of a solution but not its tonicity
b. The osmolality of the body remains fairly constant at approximately 289 mOsm/kg H2O
c. The two primary regulators of water balance are antidiuretic hormone and aldosterone
d. Serum sodium is the most valuable laboratory indicator of abnormal total body water content
Answer: a, b, d

276 Which of the following statement(s) is/are true concerning the compensatory mechanisms and treatment of metabolic acidosis?

a. Maximal renal compensation for metabolic acidosis occurs before full respiratory compensation can occur
b. All patients with lactic acidosis should receive prompt treatment with bicarbonate
c. Potassium replacement is essential even in the face of normal or high serum potassium when treating diabetic ketoacidosis
d. Sodium bicarbonate administration should begin simultaneous with volume resuscitation in patients with hypoxia secondary to shock
Answer: c

277 Which of the following statement(s) is/are true concern renal tubular acidosis?

a. Renal tubular acidosis is primarily caused by reduction in ammonia excretion
b. The renal tubular defect in renal tubular acidosis can either be at the distal or proximal renal tubule
c. In distal renal tubular acidosis associated with hyperkalemia, the defect involves increased tubular permeability with backleak of secreted sodium and potassium into the tubular cell
d. Uremic acidosis occurs independently of protein intake
Answer: a, b

278 Which of the following statement(s) is/are true concerning the postoperative fluid management in a surgical patient?

a. Standard formulas are available that essentially can direct the therapy for all patients
b. Isotonic solutions containing potassium should be used throughout the entire postoperative period
c. Urine output should be maintained at a level greater than 0.5 ml/kg/h
d. A urine specific gravity of greater than 1.012 may indicate that the patient is dehydrated
Answer: c, d

279 Which of the following statement(s) is/are true concerning parenternal electrolyte solutions?

a. Lactated Ringer’s solution contains physiologic concentrations of all important electrolytes
b. Glucose is added to hypotonic saline solutions to increase their tonicity
c. About 1/2 of all exogenously administered albumin ends up in the extravascular space
d. Normal saline solution provides excessive sodium and chloride which may lead to body sodium overload
Answer: b, c, d

280 An 11-year-old boy has experienced severe diarrhea for 10 days. He presents with decreased skin tungor, sunken eyes, orthostatic hypotension, and tachycardia. Which of the following statement(s) may be true concerning his diagnosis and treatment?

a. His hematocrit will likely be elevated
b. His BUN may be elevated out of proportion to serum creatinine
c. His serum sodium will be elevated
d. Fluid resuscitation should begin with D5/.2 normal saline because of the expected high serum sodium associated with excessive fluid loss
Answer: a, b

281 Clinical manifestations of acute metabolic acidosis include:

a. Decreased cardiac contractility
b. Decreased catecholamine secretion
c. Peripheral arteriolar dilitation
d. Shift of the oxygen-hemoglobin disassociation curve to the left
Answer: a, b, c

282 Which of the following statement(s) is/are true concerning respiratory alkalosis?

a. Exposure to high altitudes can result in respiratory alkalosis
b. Renal compensation for respiratory alkalosis is obtained by increasing excretion of bicarbonate
c. Symptoms of respiratory alkalosis may mimic those of hypocalcemia
d. The treatment of acute respiratory alkalosis may involve a brown paper bag
Answer: a, c, d

283 Which of the following statement(s) is/are true concerning alterations in serum magnesium?

a. Renal failure is the primary cause of hypermagnesemia
b. Hypomagnesemia may occur during prolonged periods of intravenous fluid replacement
c. Symptoms of hypomagnesemia may mimic symptoms of hypocalcemia
d. Intravenous administration of magnesium sulfate is usually the most efficient method of correction of magnesium deficiency
Answer: a, b, c, d

284 Muscle relaxants are a class of anesthetic agents used to prevent movement and facilitate surgical exposure. Which of the following statement(s) is/are true concerning the use of muscle relaxants in surgical procedures.

a. Succinylcholine produces rapid obvious muscle fasciculations
b. Pancuronium can be reversed by increasing the acetylcholine concentration using an anticholinesterase inhibitor (neostigmine)
c. Prolonged periods of muscle relaxation in patients requiring prolonged ventilation should be used in conjunction with analgesics and amnesic agents
d. The best clinical test for complete reversal of neuromuscular blockade is the ability of the patient to produce a large negative inspiratory force
Answer: a, b, c

285 Local anesthetics are essential agents used in current surgical practice. Which of the following statement(s) is/are true concerning the use of local anesthetic agents.

a. Complications due to excessive plasma concentration can result only from inadvertent intravascular injection of the agent
b. Bupivacaine is noted for a slow onset but long duration
c. The addition of epinephrine to a local anesthetic agent will both lower the toxicity and increase the duration of local anesthesia
d. Hypotension observed when a local anesthetic is administered in the form of a spinal epidural block, is the result of myocardial depression
Answer: b, c

286 A 65-year-old gentleman with a history of coronary artery disease and a recent myocardial infarction requires an elective colon resection for a nonobstructing neoplasm. Which of the following statement(s) is/are true concerning the risks of general anesthetic in this patient?

a. The age of the previous infarct has no effect on the perioperative reinfarction risk
b. The incidence of reinfarction appears to stabilize after six months
c. Invasive hemodynamic monitoring has no effect on perioperative reinfarction rates
d. Reinfarction has minimal effect on mortality
e. Perioperative infarction most frequently occurs after the first 72 hours from surgery
Answer: b, d

287 General anesthesia is not without risks. Which of the following statement(s) is/are true concerning the risk associated with general anesthesia.

a. Current estimates for mortality due to anesthesia alone are 1:10,000
b. Human error accounts for between 50 and 75% of anesthetic-related deaths
c. Most anesthetic-related deaths are associated with overdose of analgesic agents
d. The most common problems associated with adverse anesthetic outcomes are related to the airway
Answer: b, d

288 Over the last decade, the routine use of both invasive and noninvasive monitoring devices has been instituted for the administration of most anesthetics. The following statement(s) is/are true concerning monitoring of the surgical patient.

a. A pulse oximeter reading will reflect changes in PaO2 only below 80 mm Hg
b. Monitoring of end tidal CO2 will reflect changes in ventilation but not cardiac output
c. Intermittent, noninvasive systemic blood pressure monitoring using an oscillometric blood pressure cuff has essentially replaced clinical measurement by auscultation
d. Pulmonary arterial catheter monitoring is generally reserved for critically ill patients with significant left ventricular dysfunction
Answer: a, c, d

289 Correct statement(s) concerning complications occurring in the post-anesthetic care unit include which of the following?

a. The use of nitrous oxide has been well documented to increase the incidence of postoperative nausea
b. Perioperative myocardial ischemia is usually easily diagnosed in the early postoperative period
c. Hypothermia results in a deleterious effect on metabolism therefore delaying recovery from anesthesia
d. The serotonin antagonist, odansetron, holds promise as the superior antiemetic agent in the perioperative period
Answer: c, d

290 Patient-controlled analgesia ( PCA) is a commonly used technique for postoperative analgesia. The following statement(s) is/are true for the use of PCA.

a. Satisfactory pain relief is provided by the administration of higher narcotic doses
b. The technique is not applicable in the semiconscious or uncooperative patient
c. PCA is as safe as conventional intramuscular administration of pain medication
d. Excessive administration of narcotic medication can be limited by a lockout duration which controls administration of the narcotic
Answer: b, c, d

291 Narcotics are commonly used in the administration of general anesthesia. Which of the following statement(s) is/are true concerning this class of agents.

a. Narcotics have both profound analgesic and amnestic properties
b. Narcotics can cause hypotension by direct myocardial depressive effects
c. Naloxone should be used routinely for the reversal of narcotic analgesia
d. Acutely injured hypovolemic patients are at significant risk for decreased blood pressure with the use of narcotic analgesics
e. Propofol is a new intravenous short-acting narcotic used frequently in the outpatient setting
Answer: d

292 Anesthetic techniques used in the management of patients with significant pulmonary disease include:

a. Intubation at a deep level of anesthesia
b. Choice of an anesthetic agent which produces bronchodilatation
c. The use of epidural analgesia for postoperative pain control
d. Perioperative use of intermittent positive pressure breathing
Answer: a, b, c

293 Which of the factors listed below will adversely affect the risk of perioperative cardiac complications and reinfarction in the patient described above?

a. Greater than five premature ventricular beats per minute on EKG rhythm strip
b. The anesthetic technique used
c. Withdrawal of medical therapy with beta blockers and topical nitrates
d. Length of surgical procedure less than three hours
e. Known three vessel coronary artery disease
Answer: a, c, e

BASIC SURGICAL BACKGROUND Questions and Answers pdf Download

Leave a Reply

Your email address will not be published. Required fields are marked *